You are on page 1of 285

http:/ / www.guo.coursehost.

com

Deeper Understanding, Faster Calculation
--SOA Exam M Insights & Shortcuts

by Yufeng Guo

3
rd
Edition

Exam date: November 7, 2006

Im offering an online course for Exam M. For details, see
http:/ / www.guo.coursehost.com

This electronic book is intended for individual buyer use for the sole purpose of preparing for
Exam M. This book may NOT be resold to others or shared with others. No part of this
publication may be reproduced for resale or multiple copy distribution without the express
written permission of the author.

2006 By Yufeng Guo
Page 1 of 285 Deeper Understanding: Exam M November 7, 2006 2006 Yufeng Guo
http:/ / www.guo.coursehost.com

Table of Contents

Chapter 1 Passing Exam M the First Time .......................................... 6
Origin of the COM method............................................................................................. 6
Pitfalls of the Traditional Study Method ........................................................................ 6
How COM Works........................................................................................................... 7
Why COM Works ........................................................................................................... 8
An example of the COM method.................................................................................... 9
Chapter 2 Eliminate calculation errors .............................................. 21
Solve
2
0 ax bx c + + = . .................................................................................................. 22
Linear Interpolation ...................................................................................................... 27
Calculate mean and variance of a discrete random variable......................................... 31
Find the conditional mean and variance ....................................................................... 39
Find the present value of life insurance or annuity....................................................... 41
How to eliminate errors in complex calculations ......................................................... 44
Chapter 3 Survival distribution........................................................... 51
Age-at-death random variable of a newborn ................................................................ 51
What makes a good survival function........................................................................... 51
Age-at-death random variable of an individual aged x ............................................... 51
Distribution function of ( ) T x ...................................................................................... 52
Relationship between ( ) T x and X ............................................................................. 52
Difficult symbol ............................................................................................................ 53
Curtate future lifetime of ( ) x ....................................................................................... 53
Force of mortality (instantaneous death rate/mortality) - ( ) x and ( )
x
t ................. 55
FAQs............................................................................................................................ 56
Common problems and model solutions....................................................................... 59
Type 1 From ( ) S x to force of mortality
x
........................................................ 59
Type 2 From ( ) S x find ( ) f t , ( ) E T , ( ) Var T ................................................... 60
Type 3 From ( ) S x to
t x
p ,
t x
q ............................................................................ 62
Type 4 From a series of
t x
q find
x x t
q .................................................................. 64
Type 5 What makes a good
x
............................................................................. 65
Type 6 From ( )
x
t to
t x
p ................................................................................... 66
Type 7 Two independent lives, first death after m years but before n years...... 68
Type 8 Two independent lives, second death after m years but before n years . 70
Type 9 Two independent lives, at least one will die in n years ........................... 71
Type 10 Multiple independent lives.................................................................... 74
Chapter 4 Life Table............................................................................. 77
Basic concepts............................................................................................................... 77
Other formulas .............................................................................................................. 78
Complete expectation of future lifetime ....................................................................... 78
Curtate expectation of life............................................................................................. 78
Page 2 of 285 Deeper Understanding: Exam M November 7, 2006 2006 Yufeng Guo
http:/ / www.guo.coursehost.com

Relationship between
0
x e and
x
e ................................................................................... 79
Relationship between
0
x e and
x
e under UDD............................................................... 79
Less common life table functions ................................................................................. 79
Common problems and model solutions....................................................................... 81
Type 1 Build a life table........................................................................................ 81
Type 2 From the Life Table, find
t x
p and
t x
q ..................................................... 82
Type 3 Find mean and variance of the number of survivors................................. 83
Type 4 Recursive formula
x
e ................................................................................ 84
Type 5 From
x
l find
x
........................................................................................ 85
Type 6 Force of mortality doubles ........................................................................ 86
Chapter 5 UDD between integral ages................................................ 88
Shortcut for UDD between [ ] , 1 x x + ............................................................................ 88
UDD shortcut examples................................................................................................ 91
Chapter 6 The heart and soul of Actuarial Mathematics ................. 96
Fundamental law of life insurance................................................................................ 96
Equivalence principle.................................................................................................. 102
n-year Term Insurance Model..................................................................................... 104
n-year term fully discrete........................................................................................ 104
n-year Term Fully Continuous................................................................................ 108
Chapter 7 Fundamentals of life insurance and annuity.................. 112
Memorizing definitions............................................................................................... 112
Term vs. whole life ................................................................................................. 112
Endowment ............................................................................................................. 113
Deferred Life Insurance .......................................................................................... 114
Memorizing symbols .................................................................................................. 114
Common problems and model solutions..................................................................... 120
Type 1 Discrete life insurance, find E(Z), Var(Z) .............................................. 120
Type 2 Continuous life insurance, find E(Z), Var(Z) ......................................... 130
Free internet resources for learning life annuities....................................................... 131
Chapter 8 Reserve............................................................................... 132
Whats reserve all about?............................................................................................ 132
Whats a benefit reserve?............................................................................................ 133
How do I calculate reserve?........................................................................................ 134
Retrospective and prospective method ....................................................................... 142
Steps to calculate reserve at time t for a fully discrete insurance .............................. 146
Recursive formulas for reserve ................................................................................... 159
Reserve for variable death benefit .............................................................................. 164
How to calculate reserve for a fully continuous insurance ......................................... 165
Chapter 9 Asset share......................................................................... 168
Chapter 10 Expense loaded premium................................................. 172
Chapter 11 Multiple decrement model ............................................... 176
Examples of multiple decrements............................................................................... 176
Page 3 of 285 Deeper Understanding: Exam M November 7, 2006 2006 Yufeng Guo
http:/ / www.guo.coursehost.com

Building a multiple decrement model......................................................................... 177
The associated single decrement table........................................................................ 179
Constant Force of Mortality or UDD under the Multiple Decrement Table .............. 182
Construct a multiple decrement table ......................................................................... 186
How to memorize formulas for two decrements......................................................... 186
Common problems and model solutions..................................................................... 188
Type 1 Find
( )
f T J j = and
( )
E T J j = ......................................................... 188
Type 2
( )
f J j T = and ( )
T
f t ........................................................................... 190
Type 3 UDD in the multiple decrement table ..................................................... 192
Type 4 UDD in associated single decrement tables, 2 decrements..................... 193
Type 5 UDD in associated single decrement tables, 3 decrements..................... 195
Type 6 multiple decrements some continuous and some discrete ................... 196
Type 7 PVDB under the two decrement table .................................................... 200
Chapter 12 Markov Chain Problems.................................................. 204
Tackling Markov Chain problems using a business-oriented approach ..................... 204
Learn how to manipulate matrix................................................................................. 213
Chapter 13 Stochastic process, counting process, Poisson process.. 223
Stochastic process ....................................................................................................... 223
Counting process......................................................................................................... 224
Poisson process ........................................................................................................... 225
1
st
definition ............................................................................................................ 225
2
nd
definition ........................................................................................................... 225
3
rd
definition............................................................................................................ 226
Lucky Tom finding coins............................................................................................ 229
Chapter 14 Claim payment per payment ........................................... 245
Chapter 15 Aggregate model, normal approximation ...................... 251
Chapter 16 Double aggregate model, normal approximation.......... 255
Chapter 17 Aggregate loss model ........................................................ 260
Chapter 18 LER (loss elimination ratio)............................................. 262
Chapter 19 Find E(Y-M)
+
.................................................................... 264
Chapter 20 Other commonly used shortcuts...................................... 272
Convert discount rate to interest rate or vice versa..................................................... 272
Memorize the relationship between annuity due and life insurance........................... 272
De Moivres Law........................................................................................................ 273
Recursive formulas life insurance and annuities ..................................................... 273
UDD............................................................................................................................ 274
Insurance and annuity when If and are constant ............................................... 277
Memorize first to die and last to die ........................................................................... 277
Calculate PV of insurance or annuity if mortality is updated..................................... 278
Common shock ........................................................................................................... 278
Poisson/gamma model ................................................................................................ 279
Memorize Weibull Distribution.................................................................................. 279
Stop loss...................................................................................................................... 280
Claim payment under policy limit, deductible, and inflation ..................................... 281
Page 4 of 285 Deeper Understanding: Exam M November 7, 2006 2006 Yufeng Guo
http:/ / www.guo.coursehost.com

Split a Poisson process................................................................................................ 281
About the author....................................................................................... 283
User review of this manual....................................................................... 284
Value of this PDF study manual.............................................................. 285
Page 5 of 285 Deeper Understanding: Exam M November 7, 2006 2006 Yufeng Guo
http:/ / www.guo.coursehost.com

Chapter 1 Passing Exam M the First Time

Origin of the COM method

When I became an actuary, I was in my late 30s with a young family to support. I knew
it was possible to need three or four sittings to pass Course 3 and that it could take five to
eight years to achieve the ASA designation. I wanted to lessen this burden on my family.
After passing Course 1, I set out to develop a study method that would help me pass
Courses 2 and 3 in one sitting. By trial and error, I discovered the COM method:

C= Calculation. Eliminate all calculation errors.

O= Officially Released Exams. Master officially released Exam M problems through
reverse engineering.

M= Margin. Add some margin of error by studying more material than what you expect
will be tested.

Pitfalls of the Traditional Study Method

Under the traditional method, students read a chapter from the required text, solve some
practice problems in a corresponding study manual, and write down some notes. Then the
student moves on to the next chapter and repeats the process until completion several
months later.

Next, candidates solve previous Course 3 and M exam problems. A zealous candidate
might solve 1,000+ practice problems from sources like old SOA or CAS problems or
original practice problems. This traditional study method is how we have all been trained
to learn new information. However, the sad truth is that many candidates who use this
method fail exams repeatedly. What goes wrong?

The traditional study method has two major flaws:

First, the traditional study method casts the net too wide. Candidates indiscriminately
learn everything from the textbooks. After months of hard work, they walk into the exam
room feeling confident only to discover that they know too much about what SOA
doesnt test and too little about what SOA does test.

Second, the traditional study method encourages a candidate to solve vast numbers of
practice problems without a thorough understanding and analysis of the critical concepts.
Days and nights spent solving hundreds of old SOA/CAS problems and memorizing
formulas with only a superficial understanding of the core concepts yield little fruit on
exam day. Candidates are often shocked to discover that they did not pass the exam.
Page 6 of 285 Deeper Understanding: Exam M November 7, 2006 2006 Yufeng Guo
http:/ / www.guo.coursehost.com

After failing the exam, many candidates decide to solve even more practice problems,
leading to repeated failures of Exam M.

How COM Works

The COM method fixes the two major flaws of the traditional study method. First,
candidates find out what concepts are really important for the exam through reverse
engineering. After identifying these core concepts, candidates then learn these concepts
through solving the officially released Exam M problems. By repeatedly testing
themselves with Exam M problems under exam conditions, candidates perfect their
command of the core concepts essential for Exam M.

Calculation

Eliminate all calculation errors.
Develop error free calculation processes
Maximize the use of calculators

Officially released M problems (including Sample M problems and additional sample
problems) from the SOA website are the core for your study plan. Master Exam M
problems through reverse engineering:

Never suggest there arent enough practice problems. The officially released
Exam M problems are sufficient for anyone to pass Exam M.

Resist the temptation to solve 1,000+ old SOA/CAS problems. Dont bother with
old SOA problems. SOA is innovative and consistently develops new ways of
measuring your command of the information.

Resist the temptation to solve problems from a textbook. Textbook problems tend
to be more theoretical; exam problems are more practical.

If you do want to solve additional practice problems (such as problems from the
CAS website or from a seminar), remember to master the officially released Exam
Mproblems first. Only after you have mastered these problems should you
consider solving other practice problems. However, keep in mind that the
officially-released Exam M problems are the best practice problems.

Work problems in an exam like-condition. Then work and rework problems from
Exam M until you can solve all the problems 100% right, 100% of the time.

Do not rely on academic solutions. They look nice on paper but are too lengthy
and complex to use under exam conditions.

Page 7 of 285 Deeper Understanding: Exam M November 7, 2006 2006 Yufeng Guo
http:/ / www.guo.coursehost.com

Read Exam M problems and solutions first. Use reverse engineering to identify
what concepts are tested; then use your textbook and study manuals to learn the
tested concepts.

Build 3-minute solution scripts.

Margin -- Learn additional information in the event that there are subjects tested which
do not appear within the officially released SOA problems. This step should be
completed last, assuming you have achieved all of the previous study goals and that you
have additional time.

There are many topics included in the texts that werent tested in the SOA
officially released practice questions. There is a fair chance some of this material
will appear in the official examination.

If you have the time, prepare for the unexpected.

Take a little time to go over the textbooks and study the core concepts that are
listed in the syllabus but were not tested in the officially released questions.
If you are short on time, skip building margin.
Time Allocation

Spend 95% of your study time on perfecting your calculation skills and on mastering the
officially released M problems. Spend 5% of your time studying extra material.

Why COM Works

Officially released Exam M questions demonstrate the depth of knowledge that SOA
expects a passing candidate to have. If you can solve the officially released Exam M
problems under exam-like conditions with 100% accuracy, you should have the necessary
aptitude to pass Exam M. With the additional preparation of concepts not included in the
officially released questions, you have added some margin in case SOA decides to test
new concepts.
Reverse engineering -- First analyze exams and then, study
Under this method, you analyze what concepts are tested in Exam M. Then you learn
these concepts from the textbooks and study manuals.

1. Get the most recent SOA M exam and its solutions. Start from Problem 1 in the
exam. Read the solution. Identify what concepts are involved in Problem 1. Exam
problems are generally arranged from simple concepts to complex concepts.
Problem 1 most likely contains the easiest concepts.

Page 8 of 285 Deeper Understanding: Exam M November 7, 2006 2006 Yufeng Guo
http:/ / www.guo.coursehost.com

2. Learn the concepts involved in Problem 1 from the textbooks and study manuals
you have. Build a 3-minute solution script to this problem.

3. Work on Problem 2. Do the same: read the solution to the problem, identify what
concepts are tested, look up these concepts from your textbooks and study
manuals. Learn these concepts. Build a 3-minute solution script to Problem 2.

4. Repeat this process till you have analyzed all the test problems, identified all the
concepts involved, and built 3-minute solution scripts to all the problems.

5. Work on the next official Exam M. Do the same: read the solution to the problem,
identify what concepts are tested, look up these concepts from your textbooks and
study manuals. Learn these concepts. Build a 3-minute solution script to each of
the tested problems.

Why Reverse Engineering?

If SOA thinks a concept is important, most likely it tested this concept in Exam
M.

If SOA doesnt think that a concept is important, most likely it didnt test it in
Exam M.

Concepts tested on Exam M are the bare minimum knowledge you have to learn.

Concepts not tested in Exam M are less important and should command less of
your time.

You can add some margin by learning concepts that were not tested in Exam M,
but this should be done only after you have first mastered Exam M problems.

An example of the COM method

Typical problem -- Sample M problem # 66

For a select-and-ultimate mortality table with a 3-year select period:

(i)
x
[ ] x
q
[ ] 1 + x
q
[ ] 2 + x
q
3 + x
q
3 + x
60 0.09 0.11 0.13 0.15 63
61 0.10 0.12 0.14 0.16 64
62 0.11 0.13 0.15 0.17 65
63 0.12 0.14 0.16 0.18 66
64 0.13 0.15 0.17 0.19 67
Page 9 of 285 Deeper Understanding: Exam M November 7, 2006 2006 Yufeng Guo
http:/ / www.guo.coursehost.com

(ii) White was a newly selected life on 01/01/2000.
(iii) Whites age on 01/01/2001 is 61.
(iv) P is the probability on 01/01/2001 that White will be alive on 01/01/2006.

Calculate P .
How to reverse engineer this problem and build a 3-minute solution script:

Step 1 Identify key concepts involved:

Select & Ultimate Mortality Table
Select period
Variable x ,
x
q
Step 2 Understand the above key concepts. Translate the textbook definitions of
these key concepts into your own words.

Select
If John is selected at age 50, this simply means that John bought an insurance
policy at age 50; the insurance company selected John as a customer when John
was 50 years old.

Similarly, if John was selected on 1/1/2000, then John bought an insurance policy
on 1/1/2000; the insurance company selected John as a customer on 1/1/2000.

The word select implies that not everyone wanting a life insurance policy can
actually get one.

Life insurance companies are picky about their customers. For example, they do
not want to sell a life policy to someone on his death bed. If someone on his death
bed is allowed to buy a life insurance policy, he can purchase a big policy (such
as a policy that pays $1,000,000 death benefits if the insured dies). The insured
about to die will pay only a small initial premium (such as $1,000). If he dies in a
few days then the insurance company, having collected only $1,000 premium, has
to pay $1,000,000 to the insureds family. If a life insurance company makes a lot
of deathbed sales, it will soon go bankrupt.

Selection process
When someone applies for an insurance policy, the insurance company often pays
a doctor to give the applicant a free medical exam. Then the insurance company
analyzes the exam results to decide if this applicant is a good risk, and how much
premium to charge him. This process is called underwriting.

Age and death rates x , [ ] x ,
x
q ,
[ ] x
q
Page 10 of 285 Deeper Understanding: Exam M November 7, 2006 2006 Yufeng Guo
http:/ / www.guo.coursehost.com

Once a customers application for a life policy is approved, the insurance
company needs to estimate the likelihood that this customer will die in each of the
future years and charge a premium accordingly.

If the customer is more likely to die (e.g. he is a pilot), he will be charged a higher
premium.

The likelihood that someone who went through medical exam is going to die next
year is called
[ ] x
q or
x
q where x represents the age of the customer. We can
simply call
[ ] x
q or
x
q as the death rate for the next year. Here
[ ] x
q is the
probability that [ ] x dies between x and 1 + x ;
x
q is the probability that x will
die between ages x and 1 + x .
[ ] x and x refer to two people both aged x . The difference is that [ ] x has gone
through a medical exam and was approved to buy a life policy. In contrast, x
merely represents someone aged x who is randomly chosen from the general
population.

So [ ] x and x mean that we have two people both aged x , but [ ] x is less likely to
die than x . So
[ ] x x
q q < and [ ] x should pay less premium than x .

[ ] x
q represents the probability of someone aged x , having gone through medical
underwriting, who will die before reaching age 1 + x .

x
q represents the probability of someone aged x randomly chosen from the
general population who will die before reaching age 1 + x .

[ ] 1 + x
q refers to someone who is now 1 + x years old and who went medical exam
last year at age x .
[ ] 1 + x
q represents the likelihood that this person will die between
age 1 + x and 2 + x .

1 + x
q refers to someone who is now 1 + x years old and who was randomly chosen
from the general population last year at age x . Here
1 + x
q represents the likelihood
that this person will die between age 1 + x and 2 + x .
Select period

If after n years, the effect of the medical exam wears off and
[ ] n x n x
q q
+ +
= , then
n is called the select period. After n years, there are no differences between
[ ] n x + and n x + .
Page 11 of 285 Deeper Understanding: Exam M November 7, 2006 2006 Yufeng Guo
http:/ / www.guo.coursehost.com

So if n m , well no longer use [ ] m x + and
[ ] m x
q
+
; instead well use m x + and
m x
q
+
Select table, ultimate table, select & ultimate table

Select table - A table listing the year-by-year death rate for [ ] x . In a select table,
the effect of the medical exam has not worn off yet.

Ultimate table - A table listing the year-by-year death rate for x . In the ultimate
table, the effect of the medical exam has worn off.

Select & ultimate table If you merge a select table and an ultimate table, youll
get a select & ultimate table. In this table, the medical exam is effective for a
number of years and then wears off.

Step 3 Design a 3-minute script for looking up rates

Look up rates in a select table
To look up a death rate, first identify the issue age --- how old the insured was
when he first bought the policy. This is the age of the insured when the insurance
company issued the policy.
Next, identify how many years elapsed after the issue of the policy (this is called
policy year or duration).
Example #1. Mary is 62 years old now. She bought her insurance when she was
60. Whats her death rate from age 62 to 63?

x (issue age)
[ ] x
q
[ ] 1 + x
q
[ ] 2 + x
q
60 0.09 0.11 0.13
61 0.10 0.12 0.14
62 0.11 0.13 0.15
63 0.12 0.14 0.16
64 0.13 0.15 0.17
We have [ ] [ ] 60 = x (issue age). So well use the row where 60 = x .
The policy year or duration is 3. Policy Year 1 runs from age 60 to 61; Policy
Year 2 runs from age 61 to 62; and Policy Year 3 runs from age 62 to 63. So
Page 12 of 285 Deeper Understanding: Exam M November 7, 2006 2006 Yufeng Guo
http:/ / www.guo.coursehost.com

among the three columns of q values, well need to use the third column of q
values this is the
[ ] 2 + x
q column.

This gives us
[ ] [ ]
13 . 0
2 60 2
= =
+ +
q q
x
Example #2. John is 62 year old. He bought his insurance when he was 61.
Whats his death rate from age 62 to 63? Use the same table above.
Solution: We have [ ] [ ] 61 = x (issue age); well use the second row.

The policy year is 2. Policy Year 1 runs from age 61 to 62; Policy Year 2 runs
from age 62 to 63. This gives us
[ ] [ ]
12 . 0
1 61 1
= =
+ +
q q
x
Mary and John are both 62 years old today. However, Marys probability to die
next year is 0.13, while Johns probability to die next year is 0.12. So under the
select table, people with the same age today have different chances of dying each
year. The death rates depend, among other things, on the issue age and the
duration (i.e. policy year).

Look up rates in an ultimate table

Identify the insureds actual age (called attained age). All the people with the
same attained age have the same death rate.

Example #1. Mary is 62 years old now. She bought her insurance when she was
60. Whats her death rate from age 62 to 63? Solution: we have 62 = x (attained
age); 15 . 0
62
= q
x (attained age)
x
q
60 0.13
61 0.14
62 0.15
63 0.16
64 0.17
Example #2. John is 62 years old now. He bought his insurance when he was 61.
Whats his death rate from age 62 to 63? Solution: we have 62 = x (remember
that x is the actual age, not issue age). So 15 . 0
62
= q
Look up rates in a select and ultimate table

Within the select period, look up the death rate by issue age and by Policy Year.
After the select period, look up rates only by the attained age.

Page 13 of 285 Deeper Understanding: Exam M November 7, 2006 2006 Yufeng Guo
http:/ / www.guo.coursehost.com

Example #1. Mary is 62 years old now. She bought her insurance when she was
60. Identify her death rate by each policy year.

x
[ ] x
q
[ ] 1 + x
q
[ ] 2 + x
q
3 + x
q
3 + x
60 0.09 0.11 0.13 0.15 63
61 0.10 0.12 0.14 0.16 64
62 0.11 0.13 0.15 0.17 65
63 0.12 0.14 0.16 0.18 66
64 0.13 0.15 0.17 0.19 67
Solution: Marys death rates by policy year are listed below in color. The rates in red are
select rates; the rates in blue are ultimate rates.

x
[ ] x
q
[ ] 1 + x
q
[ ] 2 + x
q
3 + x
q
3 + x
60 0.09 0.11 0.13 0.15 63
61 0.10 0.12 0.14 0.16 64
62 0.11 0.13 0.15 0.17 65
63 0.12 0.14 0.16 0.18 66
64 0.13 0.15 0.17 0.19 67
.
Example #2. John is 62 years old now. He bought his insurance when he was 61.
Identify his death rate by each policy year.

Solution: Johns death rates by policy year are listed below in colors. The rates in red are
select rates; the rates in blue are ultimate rates.

x
[ ] x
q
[ ] 1 + x
q
[ ] 2 + x
q
3 + x
q
3 + x
60 0.09 0.11 0.13 0.15 63
61 0.10 0.12 0.14 0.16 64
62 0.11 0.13 0.15 0.17 65
63 0.12 0.14 0.16 0.18 66
64 0.13 0.15 0.17 0.19 67
Finally, lets solve the sample problem.
Repeat of the problem
For a select-and-ultimate mortality table with a 3-year select period:
(i)
x
[ ] x
q
[ ] 1 + x
q
[ ] 2 + x
q
3 + x
q
3 + x
60 0.09 0.11 0.13 0.15 63
61 0.10 0.12 0.14 0.16 64
62 0.11 0.13 0.15 0.17 65
63 0.12 0.14 0.16 0.18 66
64 0.13 0.15 0.17 0.19 67
Page 14 of 285 Deeper Understanding: Exam M November 7, 2006 2006 Yufeng Guo
http:/ / www.guo.coursehost.com

(ii) White was a newly selected life on 01/01/2000.
(iii) Whites age on 01/01/2001 is 61.
(iv) P is the probability on 01/01/2001 that White will be alive on 01/01/2006.

Calculate P .
Solution

Issue age = [ ] 60 = x .
x
[ ] x
q
[ ] [ ] 1 60 1 + +
= q q
x [ ] [ ] 2 60 2 + +
= q q
x 3 + x
q
3 + x
60 0.09 0.11 0.13 0.15 63
61 0.10 0.12 0.14 0.16 64
62 0.11 0.13 0.15 0.17 65
63 0.12 0.14 0.16 0.18 66
64 0.13 0.15 0.17 0.19 67
Policy Year From age To age Death rate Survival rate
2 61 62 0.11 0.89
3 62 63 0.13 0.87
4 63 64 0.15 0.85
5 64 65 0.16 0.84
6 65 66 0.17 0.83
So P =0.89(0.87)(0.85)(0.84)(0.83)=0.4589

Lets see why this script is powerful

(1) Designing a 3-minute solution script forces us to thoroughly analyze the key concepts
involved:
Select, Ultimate, Select & Ultimate Tables
Select period
Variables x , [ ] x ,
x
q ,
[ ] x
q
(2) Designing a 3-minute solution script forces us to understand the key concepts not only
in mathematical terms but also in a business sense. For example, we know that selection
means going through a medical exam; we know that after a period of time, the effect of
medical underwriting wears off. We not only know the math, but we also know whats
going on in the real world.

To solve problems fast, we need to understand whats going on in the real world.
Knowing how concepts are actually used in the real world transforms abstract math
Page 15 of 285 Deeper Understanding: Exam M November 7, 2006 2006 Yufeng Guo
http:/ / www.guo.coursehost.com

symbols and complex equations into concrete objects, enabling us to quickly solve an
exam problem.

(3) We developed the following step-by-step sequence on how to look up death rates:
If the table is select, we look up rates by issue age and policy year
If the table is ultimate, we look up rates by attained age
If the table is select & ultimate, we use select rates within the select period; we
use ultimate rates beyond the select period

This step-by-step sequence has remarkable power. It avoids the need for us to rethink the
key concepts involved in looking up rates problems. We have already spent a lot of
time identifying and understanding the key concepts necessary to solve the problem. We
definitely dont want to waste our time in the exam going through this time-consuming
process again. When we walk into the exam room and see a rate-looking-up problem, we
activate this script and quickly find the required death rate in the correct row and the
correct column in several seconds without thinking.

Putting this script to work

After designing a script for looking up death rates, we walk into the exam room and see
the following problem:

Sample M problem #73

For a select-and-ultimate table with a 2-year select period:

x
[ ] x
p
[ ] 1 + x
p
2 + x
p
2 + x
48 0.9865 0.9841 0.9713 50
49 0.9858 0.9831 0.9698 51
50 0.9849 0.9819 0.9682 52
51 0.9838 0.9803 0.9664 53
Keith and Clive are independent lives, both age 50. Keith was selected at age 45 and
Clive was selected at age 50.

Calculate the probability that exactly one will be alive at the end of three years.

[A] Less than 0.115
[B] At least 0.115, but less than 0.125
[C] At least 0.125, but less than 0.135
[D] At least 0.135, but less than 0.145
[E] At least 0.145

Solution

Page 16 of 285 Deeper Understanding: Exam M November 7, 2006 2006 Yufeng Guo
http:/ / www.guo.coursehost.com

We are very glad that we have a script ready for looking up rates problems. Without a
script, we have to invent solutions on the spot.

This problem has a new symbol
x
p and
[ ] x
p . You just need to know that p and q are
complements; 1 = + q p . So
x
q and
[ ] x
q represent the likelihood that x and [ ] x die,
respectively, next year;
x
p and
[ ] x
p represent the likelihood that x and [ ] x , respectively,
will not die (i.e. will be alive) next year.

Next, we get the big picture:
P (3 years later only K or C alive but not both)
= P (next 3 years K alive but C dead)
+ P (next 3 years C alive but K dead)

Because K and C are independent, we have

P (next 3 years K alive but C dead)
= P (next 3 years K alive) P (next 3 years C dead)

P (next 3 years C alive but K dead)
= P (next 3 years C alive) P (next 3 years K dead)

Next, we need to use the script to look up the right rates. The problem tells us that the
select period is 2 years.

Even if the problem doesnt tell us this information, we can tell from the select &
ultimate table that the select period is 2 years. We see that
2 + x
p is not written as
[ ] 2 + x
p .
So we know that after 2 years the selection effect wears off and [ ] 2 + x is the same as
2 + x .
Keith had a medical exam at age 45. Five years have passed since then and he is now 50.
The effect of his medical exam has worn off and we should look up his survival rates
using the ultimate table:

Ks survival rates for the next three years:

x
[ ] x
p
[ ] 1 + x
p
2 + x
p
2 + x
48 0.9865 0.9841 0.9713 50
49 0.9858 0.9831 0.9698 51
50 0.9849 0.9819 0.9682 52
51 0.9838 0.9803 0.9664 53
Please note that we dont use 0.9664 at age 53. The value of 0.9664 represents Ks
survival rate from age 53 to 54.

Page 17 of 285 Deeper Understanding: Exam M November 7, 2006 2006 Yufeng Guo
http:/ / www.guo.coursehost.com

Lets turn to Clive. Clive was selected just today. So for the first two years, well look up
his rates from the select table. For Year 3, we have to use the ultimate table:

Cs survival rates for the next 3 years:

x
[ ] x
p
[ ] 1 + x
p
2 + x
p
2 + x
48 0.9865 0.9841 0.9713 50
49 0.9858 0.9831 0.9698 51
50 0.9849 0.9819 0.9682 52
51 0.9838 0.9803 0.9664 53
We see that both Keith and Clive have the same survival rate of 0.9682 when they move
from age 52 to age 53. Why? At age 52, they both have passed the select period and their
survival rates depend only on their attained age. Since they have the same attained age,
their survival rates are always the same in the ultimate table.

Now we are ready to calculate the final result:

P (next 3 years K alive but C dead)
= P (next 3 years K alive) P (next 3 years C dead)
= P (next 3 years K alive) * [1- P (next 3 years C alive) ]
=0.9713(0.9698)(0.9682) * (1-0.9849 * 0.9819 * 0.9682)= 5.807%

P (next 3 years C alive but K dead)
= P (next 3 years C alive) P (next 3 years K dead)
= P (next 3 years C alive) * [ 1- P (next 3 years K alive) ]
=0.9849 * 0.9819 * 0.96821 * (1 - 0.9713 * 0.9698 * 0.9682)= 8.238%

P (next 3 years later only K or C alive but not both)
= 5.807%+ 8.238% = 14.045%

So the answer is [D] At least 0.135, but less than 0.145
.
FAQ

I just bought your manual and finished reading Chapter 1. What should I do next?
Should I start reverse engineering official M problems? Or should I read your
manual first and then start reverse engineering?

Either way is fine. Do whatever makes sense to you.

I started reverse engineering old M problems but got stuck in Problem #1. The
reverse study method is a lot harder than the traditional study method. What
should I do next?
Page 18 of 285 Deeper Understanding: Exam M November 7, 2006 2006 Yufeng Guo
http:/ / www.guo.coursehost.com

If reverse engineering is too time consuming, throw it away. Use the traditional study
sequence. Or you can use 60% (or some other combinations) traditional method (learn
first, solve problem second) and 40% reverse engineering (first solve SOA problems
and next learn whats necessary). Do whatever makes sense to you.
I like reverse engineering for two reasons: (1) Im in the drivers seat. Under the
traditional study method, Im spoon-fed by authors of textbooks and study manuals. They
do too much thinking for me. In contrast, under reverse engineering, I have to figure
things out myself. This forces me to really understand the core concepts. (2) Reverse
engineering cuts a lot of the fluff. Under the traditional study method, youll spend weeks
mastering a difficult concept only to find out that it was not tested in the exam. Under
reverse engineering, however, you learn what really matters for the exam from day one.

Whether you use reverse engineering or the traditional method, make sure you master the
official M exam problems.

Whats the risk of using the COM method?

Major risk: passing the exam quickly with only a moderate amount of study time. You
may have to change your vacation plans and go to the beach rather than studying with
your friends for a second exam-sitting.

Your approach makes sense. However, I still want to solve lots of old SOA
problems. If I dont solve lots of practice problems, I feel insecure. Is there anything
wrong with me solving lots of practice problems?

The central message of the COM method is this: Look. You really dont need to study
that hard. Master the official exam problems and youll be fine. However, not every
candidate is comfortable with this minimalist approach. If you still want to solve lots of
practice problems, do so.

Does the COM method work for the type of essay problems seen for Courses 5, 6, 7
and 8?

No, the COM method wont work for essay exams because in essay problems, there are
an unlimited number of ways to test a candidates knowledge. To pass Courses 5, 6, 7,
and 8, you must read everything in the syllabus.

On the other hand, Exams P, FM, M, and C are multiple-choice, formula-driven exams.
In these exams, there are a finite number of core concepts and formulas to master. SOA
tests the same core concepts and formulas over and over. As a result, if you can master
the officially-released M problems, you have gained sufficient knowledge of the core
concepts in Exam M and should be able to pass.

If SOA finds out that many people are using the COM method, cant they make the
exam different to purposely render COM useless?

Page 19 of 285 Deeper Understanding: Exam M November 7, 2006 2006 Yufeng Guo
http:/ / www.guo.coursehost.com

The COM method works even if SOA knows that many candidates are using it. SOA
purposely releases M exams so candidates will know whats really tested on the exam.
SOA wants people to master the official exam M problems. SOA will be flattered to
know that you have mastered the officially released Exam M problems prior to taking the
exam.
Page 20 of 285 Deeper Understanding: Exam M November 7, 2006 2006 Yufeng Guo
http:/ / www.guo.coursehost.com

Chapter 2 Eliminate calculation errors

Fundamental principle: To eliminate all calculation errors, you need to do your
calculations differently.
Many people ask how I could pass exams while many failed the same exam repeatedly.
Like an athlete I trained meticulously till the calculations became almost automatic. I
strove for perfection in the accuracy of my calculations so that even when SOA raised its
bar and threw in surprise problems, I could still succeed on the mundane and repeatedly-
tested problems. By practicing solving these easier problems without a single calculation
error, I could walk away from this exam with the assurance that I had succeeded in
passing.

If you look at top professional athletes, youll find that they are meticulous in every
aspect of their sport. They are perfectionists. Similarly, you need to be meticulous in all
your calculations.

Sloppy calculations will kill your exam score. The sad truth is, however, that most
candidates dont pay attention to calculation accuracy because eliminating errors is not
glamorous; solving 1,000 problems is. If you say that you have solved 1,000 practice
problems, youll impress a lot of people. If you say that you spent a lot of time reducing
calculation errors, that doesnt sound impressive at all. But for the sake of your exam
score, please do yourself a favor and learn how to eliminate calculation errors.
Eliminating calculation errors requires new calculation habits and diligence, but the
payoff is extraordinary: calculations 100% right 100% of the time.

If you are a borderline student, the

first thing you can do to boost your score is to improve
your calculation skills. Write yourself a note stating the following:

I have zero tolerance for my calculation errors.
I dare to bet $1,000 to anyone that I will make zero errors on the exam.
On the exam, if I know the concept of a problem, Im 100% certain that Ill solve
this problem right.
I will never walk into the exam room without perfect calculation skills.

Read the note from time to time and get rid of your sloppy calculation habits.

Next, lets talk about details. Before moving on in this chapter, you need to have at least
two calculators:
(1) Either BA II Plus or BA II Plus Professional, and
(2) TI-30X IIS.
Page 21 of 285 Deeper Understanding: Exam M November 7, 2006 2006 Yufeng Guo
http:/ / www.guo.coursehost.com

I recommend that you immediately download the calculator manuals (called
guidebooks) from ti.com (Texas Instruments website). Next, follow the instructions in
the guidebook and set your calculator to the following settings:

1. BA II Plus / BA II Plus Professional

Use the AOS mode. Never use the Chn method.
Display 8 decimal places

2. TI-30 IIS
Display 8 decimal places

Next, work through all the relevant examples in the guidebooks.

Fast and safe techniques for common calculations

Solve
2
0 ax bx c + + = .
The formula
2
4
2
b b ac
x
a

= is okay when , , a b and c are small. However, when
, , a b and c have many decimals or are large numbers and we are in the heat of the exam,
the standard solution is cumbersome.
Example 1. Solve
2
0.3247 89.508 0.752398 0 x x + = in 15 seconds.

The standard approach
2
4
2
b b ac
x
a

= is labor intensive and prone to errors.

To solve this equation 100% right under pressure and in a hurry, well do a little trick.
First, we set
1
1
x v
r
= =
+
. So we treat x as a dummy discounting factor. The original
equation becomes:

2
0.3247 89.508 0.752398 0 v v + =
If we can find r , the dummy interest rate, well be able to find x .
Finding r is a concept you learned in Exam FM. We first convert the equation to the
following cash flow diagram:

Page 22 of 285 Deeper Understanding: Exam M November 7, 2006 2006 Yufeng Guo
http:/ / www.guo.coursehost.com

Time t 0 1 2

Cash flow $0.752398 - $89.508 $0.3247

So at time zero, you receive $0.752398. At time one, you pay $89.508. Finally, at time
two, you receive $0.3247. Whats your IRR?

To find r (the IRR), we simply use the Cash Flow Worksheet in BA II Plus or BA II
Plus Professional.

Enter the following cash flows into the Cash Flow Worksheet:

Cash Flow 0 CF 01 C 02 C
0.752398 - 89.508 0.3247
Frequency 01 F 02 F
1 1
Because the cash flow frequency is one for both 01 C and 02 C , we dont need to enter
01 1 F = or 02 1 F = . If we dont enter the cash flow frequency, BA II Plus and BA II Plus
Professional will use one as the default cash flow frequency.

Using the IRR function, we find that 99.63722807 IRR = . Remember this is a
percentage. So 99.63722807% r =
1
1 1
275.6552834
1 1 99.63722807%
x
r
= = =
+
How do we find the second root? Well use the following formula:
If
1
x and
2
x are the two roots of
2
0 ax bx c + + = , then

1 2 2
1
1 c c
x x x
a x a
= =
2
1
1 1 0.752398
0.00840619
275.6552834 0.3247
c
x
x a
= = =
Page 23 of 285 Deeper Understanding: Exam M November 7, 2006 2006 Yufeng Guo
http:/ / www.guo.coursehost.com

Keystrokes for BA II Plus / BA II Plus Professional

Procedure Keystroke Display
Assume we set the calculator to
display 8 decimal places.

Use Cash Flow Worksheet
CF CF0=(old content)
Clear Worksheet
2
nd
[CLR WORK] CF0=0.00000000
Enter the cash flow at t = 0.
+
CF0=0.752398
+
C01 = 0.00000000
Enter the cash flow at t =1.

89.508 +/-
Enter
C01= - 89.50800000
Enter the # of cash flows for
C01

+
F01= 1.00000000

The default # is 1. We dont
need to enter anything.
+
C02 0.00000000
Enter the cash flow at t =2.

0.3247 Enter C02= 0.32470000
IRR IRR=0.00000000
Calculate IRR

CPT IRR= - 99.63722807
% IRR 0.9963722807 (This
is the dummy interest)
+ 1= IRR 0.00362772

Find the dummy discount factor
1
1
1 %
x
IRR
=
+
1 x
IRR 275.65528324

This is
1
x
Page 24 of 285 Deeper Understanding: Exam M November 7, 2006 2006 Yufeng Guo
http:/ / www.guo.coursehost.com

Store in Memory 0. This leaves
an auditing trail.
STO 0 IRR 275.65528324

Find the 2
nd
root.
2
1
1 c
x
x a
=
1 x 0.752398
0.3247

=
0.00840619 This is
2
x
Store in Memory 0. This leaves
an auditing trail.

STO 1 0.00840619
You can always double check your calculations. Retrieve
1
x and
2
x from the calculator
memory and plug in
2
0.3247 89.508 0.752398 x x + . You should get a value close to
zero. For example, plugging in
1
275.6552834 x = :
2
0.3247 89.508 0.752398 0.00000020 x x + = (OK)

Plugging in
2
0.00840619 x =
2 12
0.3247 89.508 0.752398 6.2 10 x x

+ = (OK)

We didnt get a zero due to rounding.

Does this look like a lot of work? Yes, the first time, but once you get familiar with this
process, it takes 15 seconds to calculate
1
x and
2
x and double check they are right.

Quick and error-free solution process for
2
0 ax bx c + + =
Page 25 of 285 Deeper Understanding: Exam M November 7, 2006 2006 Yufeng Guo
http:/ / www.guo.coursehost.com

Step 1 Rearrange
2
0 ax bx c + + = to
2
0 c bx ax + + = .
Step 2 Use BA II Plus/BA II Plus Professional Cash Flow Worksheet to find IRR
0 CF c = (cash flow at time zero)
01 C b = (cash flow at time one)
02 C a = (cash flow at time two)
Time t 0 1 2
Cash flow c b a
Step 3 Find
1
x and
2
x ; store them in your calculators memory.
1
1
1
100
x
IRR
=
+
,
2
1
1 c
x
x a
=
Step 4 Retrieve
1
x and
2
x from your calculators memory. Check that
2
0 ax bx c + + =
In the exam, if an equation is overly simple, just try out the answer. However, if an
equation is not overly simple, always use the above process to solve
2
0 ax bx c + + = .
For example, if you see
2
2 3 0 x x = , you can guess that
1
1 x = and
2
3 x = . However,
if you see
2
2 7.3 0 x x = , use the Cash Flow Worksheet to solve it.

Exercise

#1 Solve
2
10, 987 65, 864 98, 321 0 x x + + =
Answer:
1
7.2321003 x = and
2
1.23737899 x =
#2 Solve
2
2 7.3 0 x x = .
Answer:
1
3.88097206 x = and
2
1.88097206 x =
Page 26 of 285 Deeper Understanding: Exam M November 7, 2006 2006 Yufeng Guo
http:/ / www.guo.coursehost.com

#3 Solve
2
0.9080609 0.00843021 0.99554743 0 x x =
Answer:
1
1.0517168 x = and
2
1.04243305 x =
#4 Solve
2
2 3 0 x x + = .
Answer: youll get an error message if want to calculate IRR. Theres no solution.
( )
2
2
2 3 1 2 2 x x x + = + > . So theres no solution.
Linear Interpolation

Example 1 (one step in Sample M # 15)

You are given the following values of the cdf of a standard normal distribution:

( ) 0.4 0.6554 u = , ( ) 0.5 0.6915 u =
Use linear interpolation, calculate ( ) 0.443 u
Solution

The standard solution is

( ) ( ) ( )
0.5 0.443 0.443 0.4
0.443 0.4 0.5
0.5 0.4 0.5 0.4

u = u + u


( ) ( ) 0.57 0.4 0.43 0.5 = u + u
( ) ( ) 0.57 0.6554 0.43 0.6915 0.6709 = + =
This is the SOA solution.

The downside of this approach is that its very prone to errors. The math logic is simple,
but there are simply too many numbers to calculate. And its very easy to make a
mistake, especially in the heat of the exam.

Fortunately, the LIN (LIN stands for standard linear regression) Statistics Worksheet in
BA II Plus and BA II Plus Professional can do linear interpolation for us.

The keystrokes for linear interpolation BA II Plus and BA II Plus Professional:

2
nd
Data (activate statistics worksheet)
2
nd
CLR Work (clear the old contents)
X01=0.4, Y01=0.6554
X02=0.5, Y02=0.6915
Page 27 of 285 Deeper Understanding: Exam M November 7, 2006 2006 Yufeng Guo
http:/ / www.guo.coursehost.com

2
nd
STAT (keep pressing 2
nd
Enter until you see LIN)

Press the down arrow key + , youll see 2 n =
Press the down arrow key + , youll see 0.45 X =
Press the down arrow key + , youll see 0.07071068
X
S =
Press the down arrow key + , youll see 0.05
X
o =
Press the down arrow key + , youll see 0.67345 Y =
Press the down arrow key + , youll see 0.02552655
y
S =
Press the down arrow key + , youll see 0.01805
y
o =
Press the down arrow key + , youll see 0.511 a =
Press the down arrow key + , youll see 0.361 b =
Press the down arrow key + , youll see 1 r = (this is the correlation coefficient)
Press the down arrow key + , youll see
'
0.00 X =
Enter
'
0.443 X =
Press the down arrow key + .
Press CPT. Youll get
'
0.670923 Y =
So ( ) 0.443 0.670923 u =
If you have trouble understanding any of the above keystrokes, read the TI calculator
guidebook on Two Variable Statistics Example. Youll see detailed examples on how
to use the LIN Worksheet.

The above keystrokes may seem to be an awful lot of work, but after you get the hang of
the calculator keystrokes, the calculation procedure is really fast. And more importantly,
the calculation result is 100% right.

Beside its perfect precision, another nice thing about using BA II Plus and BA II Plus
Professional for linear interpolation is that once you have entered two data pairs ( )
1 1
, x y
and ( )
2 2
, x y , you can generate any 3
rd
data pair ( )
3 3
, x y .
In the above example, after generating ( ) 0.443 0.670923 u = , if you want to generate
( ) 0.412345 u , this is what you do:

Goal -- Generate ( ) 0.412345 u
Page 28 of 285 Deeper Understanding: Exam M November 7, 2006 2006 Yufeng Guo
http:/ / www.guo.coursehost.com

Enter
'
0.412345 X =
Press the down arrow key + .
Press CPT. Youll get
'
0.65985655 Y =
If you want to generate ( ) 0.46789 u , this is what you do:

Goal - ( ) 0.46789 u
Enter
'
0.46789 X =
Press the down arrow key + .
Press CPT. Youll get
'
0.67990829 Y =
General procedure
Given two data pairs ( )
1 1
, c d and ( )
2 2
, c d and a single data
3
c , to generate
3
d using BA
II Plus and BA II Plus Professional LIN Worksheet, enter
X01=
1
c , Y01=
1
d
X02=
2
c , Y02=
2
d
'
3
X c =
In other words, the independent variable
1 2 3
, , c c c must be entered as ' X s ;
1 2
, d d must
be entered as ' Y s .
Example 2

You are given the following values of the cdf of a standard normal distribution:

( ) 0.4 0.6554 u = , ( ) 0.5 0.6915 u =
Using linear interpolation, find , , a b c , and d (all these are positive numbers) such that

( ) 0.6666 a u =
( ) 0.6777 b u =
( ) 0.6888 c u =
( ) 0.6999 d u =
Page 29 of 285 Deeper Understanding: Exam M November 7, 2006 2006 Yufeng Guo
http:/ / www.guo.coursehost.com

Solution

In BA II Plus and BA II Plus Professional LIN Statistics Worksheet, enter

X01=0.6554, Y01=0.4
X02=0.6915, Y02=0.5

Enter
'
0.6666 X = . Then the calculator will generate
'
0.43102493 Y = .
So 0.43102493 a = .
Enter
'
0.6777 X = . Then the calculator will generate
'
0.46177285 Y =
So 0.46177285 b = .
Enter
'
0.6888 X = . Then the calculator will generate
'
0.49252078 Y =
0.49252078 c =
Enter
'
0.6999 X = . Then the calculator will generate
'
0.52326870 Y =
So 0.52326870 d =
Example 3

The population of a survivor group is assumed to be linear between two consecutive
integer ages. You are given the following:

Age # of people alive at this age
50 598
51 534

Calculate the # of people alive at the following fractional ages:
50.2, 50.5, 50.7, 50.9
Solution

In BA II Plus and BA II Plus Professional LIN Statistics Worksheet, enter

X01=50, Y01=598
X02=51, Y02=534

Enter
'
50.2 X = . Then the calculator will generate
'
585.2 Y =
Enter
'
50.5 X = . Then the calculator will generate
'
566 Y =
Enter
'
50.7 X = . Then the calculator will generate
'
553.2 Y =
Page 30 of 285 Deeper Understanding: Exam M November 7, 2006 2006 Yufeng Guo
http:/ / www.guo.coursehost.com

Enter
'
50.9 X = . Then the calculator will generate
'
540.4 Y =
Calculate mean and variance of a discrete random variable

There are two approaches:
Use TI-30 IIS (using the redo capability of TI-30IIS)
Use BA II Plus/BA II Plus Professional 1-V Statistics Worksheet

Exam #1 (#8 Course 1 May 2000) A probability distribution of the claim sizes for
an auto insurance policy is given in the table below:

Claim Size Probability
20 0.15
30 0.10
40 0.05
50 0.20
60 0.10
70 0.10
80 0.30

What percentage of the claims are within one standard deviation of the mean claim size?

(A) 45%, (B) 55%, (C) 68%, (D) 85%, (E)100%

Solution

This problem is conceptually easy but calculation-intensive. It is easy to make calculation
errors. Always let the calculator do all the calculations for you.

One critical thing to remember about the BA II Plus and BA II Plus Professional
Statistics Worksheet is that you cannot directly enter the probability mass function ( )
i
f x
into the calculator to find ( ) E X and ( ) Var X . BA II Plus and BA II Plus Professional 1-
V Statistics Worksheet accept only scaled-up probabilities that are positive integers. If
you enter a non-integer value to the statistics worksheet, you will get an error when
attempting to retrieve ( ) E X and ( ) Var X .
To overcome this constraint, first scale up ( )
i
f x to an integer by multiplying ( )
i
f x by a
common integer.

Page 31 of 285 Deeper Understanding: Exam M November 7, 2006 2006 Yufeng Guo
http:/ / www.guo.coursehost.com

Claim Size x Probability Pr( ) x Scaled-up probability
=100Pr( ) x
20 0.15 15
30 0.10 10
40 0.05 5
50 0.20 20
60 0.10 10
70 0.10 10
80 0.30 30
Total 1.00 100
Next, enter the 7 data pairs of (claim size and scaled-up probability) into the BA II Plus
Statistics Worksheet to get ( ) E X and
X
o .
BA II Plus and BA II Plus Professional calculator key sequences:

Procedure Keystrokes Display
Set the calculator to display
4 decimal places

2nd [FORMAT] 4 ENTER DEC=4.0000
Set AOS (Algebraic
operating system)

2nd [FORMAT],
keep pressing + multiple
times until you see Chn.
Press 2nd [ENTER]
(if you see AOS, your
calculator is already in
AOS, in which case press
[CLR Work] )

AOS
Select data entry portion of
Statistics worksheet

2nd [Data] X01 (old contents)
Clear worksheet

2nd [CLR Work] X01 0.0000
Enter data set
20 ENTER
+ 15 ENTER
X01=20.0000
Y01=15.0000
+ 30 ENTER
X02=30.0000
Y02=10.0000
Page 32 of 285 Deeper Understanding: Exam M November 7, 2006 2006 Yufeng Guo
http:/ / www.guo.coursehost.com

+ 10 ENTER
+ 40 ENTER
+ 5 ENTER
X03=40.0000
Y03=5.0000
+ 50 ENTER
+ 20 ENTER
X04=50.0000
Y04=20.0000
+ 60 ENTER
+ 10 ENTER
X05=60.0000
Y05=10.0000
+ 70 ENTER
+ 10 ENTER
X06=70.0000
Y06=10.0000
+ 80 ENTER
+ 30 ENTER
X07=80.0000
Y07=30.0000
Select statistical calculation
portion of Statistics
worksheet

2nd [Stat] Old content
Select one-variable
calculation method

Keep pressing 2
nd
SET
until you see 1-V

1-V
View the sum of the scaled-
up probabilities
+
n=100.0000 (Make sure the
sum of the scaled-up
probabilities is equal to the
scaled-up common factor,
which in this problem is
100. If n is not equal to the
common factor, youve
made a data entry error.)
View mean
+
x =55.0000
View sample standard
deviation
+
x
S =21.9043 (this is a
sample standard deviation--
- dont use this value). Note
that
2
x
1
1
( )
1
n
i
i
S X X
n
=
=

Page 33 of 285 Deeper Understanding: Exam M November 7, 2006 2006 Yufeng Guo
http:/ / www.guo.coursehost.com

View standard deviation
+ o
X
=21.7945
View

X
+

X =5,500.0000 (not
needed for this problem)
View

2
X
+

2
X =350,000.0000 (not
needed for this problem,
though this function might
be useful for other
calculations)
You should always scroll up and down using |+ to double check that your data entry is
correct before accepting ( ) E X and
X
o generated by BA II Plus.

If you have made an error in data entry, you can 2
nd
DEL to delete a data pair (X, Y) or
2
nd
INS to insert a data pair (X,Y). If you typed a wrong number, you can use to delete
the wrong number and then re-enter the correct number. Refer to the BA II Plus
guidebook for details on how to correct data entry errors.

If this procedure of calculating ( ) E X and
X
o seems more time-consuming than the
formula-driven approach, it could be because you are not yet familiar with the BA II Plus
Statistics Worksheet. With practice, you will find that using the calculator is quicker and
more accurate than manually calculating with formulas.

Then, we have

( , ) (55 21.7945, 55 21.7945)
=(33.21, 76.79)
X X X X
o o + = +
Finally, you find

Pr(33.21 76.79) Pr( 40) Pr( 50) Pr( 60) Pr( 70)
=0.05+0.20+0.10+0.10 =0.45
X X X X X s s = = + = + = + =
Using TI-30X IIS
First, calculate ( ) E X using =

( ) ( ) E X xf x . Then modify the formula



2
( ) to ( ) xf x x f x to calculate Var(X) without re-entering ( ) f x .
To find ( ) E X , we type:
Page 34 of 285 Deeper Understanding: Exam M November 7, 2006 2006 Yufeng Guo
http:/ / www.guo.coursehost.com

20*.15+30*.1+40*.05+50*.2+60*.1+70*.1+80*.3

Then press Enter. ( ) E X =55.
Next we modify the formula

20 .15+30 .1+40 .05+50 .2+60 .1+70 .1+80 .3 - - - - - - -
to

2 2 2 2 2 2 2
20 .15+30 .1+40 .05+50 .2+60 .1+70 .1+80 .3 - - - - - - -
To change 20 to
2
20 , move the cursor immediately to the right of the number 20 so
your cursor is blinking on top of the multiplication sign -. Press 2nd INS
2
x .

You find that

2 2 2 2 2 2 2
20 .15+30 .1+40 .05+50 .2+60 .1+70 .1+80 .3 - - - - - - -
=3500

So
2
( ) E X =3,500

=
2 2
( ) ( ) ( ) Var X E X E X =3,500-
2
55 =475.

Finally, you can calculate
X
o and the range of o o + ( , )
X X X X
.
Keep in mind that you can enter up to 88 digits for a formula in TI-30X IIS. If your
formula exceeds 88 digits, TI 30X IIS will ignore the digits entered after the 88
th
digit.
Example 2 (#19, Course 1 November 2001)

A baseball team has scheduled its opening game for April 1. If it rains on April 1, the
game is postponed and will be played on the next day that it does not rain. The team
purchases insurance against rain. The policy will pay 1,000 for each day, up to 2 days,
that the opening game is postponed. The insurance company determines that the number
of consecutive days of rain beginning on April 1 is a Poisson random variable with a 0.6
mean. What is the standard deviation of the amount the insurance company will have to
pay?

(A) 668, (B) 699, (C) 775, (D) 817, (E) 904

Page 35 of 285 Deeper Understanding: Exam M November 7, 2006 2006 Yufeng Guo
http:/ / www.guo.coursehost.com

Solution

Let N = number of days it rains consecutively. N can be 0,1,2, or any non-negative
integer.

0.6
0.6
( =0,1,2,..+ )
!
Pr( )
!
n n
e n
n
N n e
n


= = =
Let X = payment by the insurance company. According to the insurance contract, if there
is no rain (n=0), then X=0. If it rains for only one day, X=$1,000. If it rains for two or
more days in a row, X is always $2,000. We are asked to calculate
X
o .
If a problem asks you to calculate the mean, standard deviation, or other statistics of a
discrete random variable, it is always a good idea to list the variables values and their
corresponding probabilities in a table to organize your data before doing the calculation.
So lets list the data pair ( X , probability) in a table:

Payment X Probability of receiving X
0
( )

= = =
0.6 0.6
0
0.6
0
0!
P N e e
1,000
( )

= = =
1
0.6 0.6
0.6
1 0.6
1!
P N e e
2,000
( ) ( ) ( ) > = = + = + 2 2 3 ... P N P N P N
( ) ( ) = = + = (

1- 0 1 P N P N

=
0.6
1- 1.6e
Once you set up the table above, you can use BA II Pluss Statistics Worksheet or TI-30
IIS to find the mean and variance.

Calculation Method 1 --- Using TI-30X IIS
First we calculate the mean by typing:

1000*.6e^(-.6)+2000(1-1.6e^(-.6

When typing e^(-.6) for
0.6
e

, you need to use the negative sign, not the minus sign, to
get -6. If you type the minus sign in e^( .6), you will get an error message.

Additionally, for 0.6
0.6
e

, you do not need to type 0.6*e^(-.6), just type .6e^(-.6). Also,


to calculate
.6
2000(1 1.6 ) e

, you do not need to type 2000*(1-1.6*(e^(-.6))). Simply


type
Page 36 of 285 Deeper Understanding: Exam M November 7, 2006 2006 Yufeng Guo
http:/ / www.guo.coursehost.com

2000(1-1.6e^(-.6

Your calculator understands you are trying to calculate
.6
2000(1 1.6 ) e

. However, the
omission of the parenthesis sign works only for the last item in your formula. In other
words, if your equation is

.6 .6
2000(1 1.6 ) 1000 .6 e e

+
you have to type the first item in its full parenthesis, but can skip typing the closing
parenthesis for the second item:

2000(1-1.6e^(-.6)) + 1000*.6e^(-.6

If you type

2000(1-1.6e^(-.6 + 1000*.6e^(-.6

your calculator will interpret this as

2000(1-1.6e^(-.6 + 1000*.6e^(-.6)))

Of course, this is not your intention.

Lets come back to the calculation. After you type

1000*.6e^(-.6)+2000(1-1.6e^(-.6

press ENTER. You should get ( ) E X = 573.0897. This is an intermediate value. You
can store it on your scrap paper or in your calculators memory.

Next, modify your formula to get
2
( ) E x by typing:

2 2
1000 .6 ^( .6) 2000 (1 1.6^( .6 e - + -
You will get 816892.5107. This is
2
( ) E x . Next, calculate ( ) Var X
2 2
( ) ( ) ( ) Var X E x E x = =488460.6535
( ) 698.9960
X
Var x o = = .
Page 37 of 285 Deeper Understanding: Exam M November 7, 2006 2006 Yufeng Guo
http:/ / www.guo.coursehost.com

Calculation Method 2 --Using BA II Plus/ BA II Plus Professional
First, please note that you can always calculate
X
o without using the BA II Plus built-in
Statistics Worksheet. You can calculate
2
( ), ( ), ( ) E X E X Var X in BA II Plus as you do
any other calculations without using the built-in worksheet.

In this problem, the equations used to calculate
X
o are:

.6 .6 .6
( ) 0* 1,000(.6 ) 2,000(1 1.6 ) E x e e e

= + +
2 2 .6 2 .6 2 .6
( ) 0 1,000 .6 2,000 (1 1.6 ) E x e e e

= + +
2 2
( ) ( ) ( ), ( )
X
Var x E x E x Var x o = =
You simply calculate each item in the above equations with the BA II Plus. This will give
you the required standard deviation.

However, we do not want to do this hard-core calculation in an exam. BA II Plus already
has a built-in statistics worksheet and we should utilize it.

The key to using the BA II Plus Statistics Worksheet is to scale up the probabilities to
integers. To scale the three probabilities:

.6 .6 .6
( , 0.6 , 1 1.6 ) e e e

is a bit challenging, but there is a way:



Payment X Probability (assuming you set
your BA II Plus to display 4
decimal places)
Scale up probability to integer
(multiply the original probability
by 10,000)
0
0.6
e

= 0.5488
5,488
1,000
0.6
0.6e

= 0.3293
3,293
2,000
0.6
1-1.6e

=0.1219
1,219
Total 1.0 10,000
Then we just enter the following data pairs into BA II Pluss statistics worksheet:

X01=0 Y01=5,488;
X02=1,000 Y02=3,293;
X03=2,000 Y03=1,219.

Then the calculator will give you 698.8966
X
o =
Page 38 of 285 Deeper Understanding: Exam M November 7, 2006 2006 Yufeng Guo
http:/ / www.guo.coursehost.com

Make sure your calculator gives you n that matches the sum of the scaled-up
probabilities. In this problem, the sum of your scaled-up probabilities is 10,000, so you
should get n=10,000. If your calculator gives you n that is not 10,000, you know that at
least one of the scaled-up probabilities is wrong.

Of course, you can scale up the probabilities with better precision (more closely
resembling the original probabilities). For example, you can scale them up this way
(assuming you set your calculator to display 8 decimal places):

Payment X Probability Scale up probability to integer
more precisely (multiply the
original probability by
100,000,000)
0
0.6
e

= 0.54881164
54,881,164
1,000
0.6
0.6e

= 0.32928698
32,928,698
2,000
0.6
1-1.6e

=0.12190138
12,190,138
Total 100,000,000
Then we just enter the following data pairs into BA II Pluss statistics worksheet:

X01=0 Y01=54,881,164;
X02=1,000 Y02=32,928,698;
X03=2,000 Y03=12,190,138.

Then the calculator will give you
X
o =698.8995993 (remember to check that
n=100,000,000)

For exam problems, scaling up the original probabilities by multiplying them by 10,000
is good enough to give you the correct answer. Under exam conditions it is unnecessary
to scale the probability up by multiplying by 100,000,000.

Find the conditional mean and variance

Example

For an insurance policy:

A policyholders annual losses can be 100, 200, 300, and 400 with respective
probabilities 0.1, 0.2, 0.3, and 0.4.

The insurance has an annual deductible of $250 per loss.

Calculate the mean and the variance of the annual payment made by the insurer to the
policyholder, given theres a payment.
Page 39 of 285 Deeper Understanding: Exam M November 7, 2006 2006 Yufeng Guo
http:/ / www.guo.coursehost.com

Solution

Let X represent the annual loss. Let Y represent the claim payment by the insurer to the
policyholder.

Then
0 if 250
250 if 250
X
Y
X X
s
=

>

We are asked to find


( )
250 E Y X > and
( )
250 Var Y X >
Standard solution

X 100 200 300 400
Y 0 0 50 150
( ) P X
0.1 0.2 0.3 0.4
( ) ( ) ( ) 250 300 400 0.3 0.4 0.7 P X P X P X > = = + = = + =
( )
( ) 250
P X
P X >
0.1
0.7
0.2
0.7
0.3
0.7
0.4
0.7
( )
1 2 3 4
250 250 0 0 50 150 107.1428571
7 7 7 7
E X X
| | | | | | | |
> = + + + =
| | | |
\ . \ . \ . \ .

( )
2
2 2 2 2
1 2 3 4
150 150 0 0 50 150 13, 928.57143
7 7 7 7
E X X
+
| | | | | | | |
(
> = + + + =
| | | |

\ . \ . \ . \ .

( )
2
150 150 13, 928.57143 107.1428571 2, 448.99 Var X X
+
(
> = =


Fast solution using BA II Plus/BA II Plus Professional 1-V Statistics Worksheet

X 100 200 300 400
Y >250?
If Yes, Keep; if No,
discard.
No. Discard No. Discard. Yes. Keep. Yes. Keep.
Page 40 of 285 Deeper Understanding: Exam M November 7, 2006 2006 Yufeng Guo
http:/ / www.guo.coursehost.com

New table after discarding 250 X s :
X 300 400
Y 50 150
( ) P X
0.3 0.4
( ) 10P X -- scaled up probability
3 4
Enter the following into 1-V Statistics Worksheet:

X01=50, Y01=3; X02=150, Y02=4

BA II Plus or BA II Plus Professional should give you:

7, 107.14, 49.48716593
X
n X o = = =
2
2, 4489.98 Var o = =
General procedure to calculate ( ) E Y x x a ( >

using BA II Plus and BA II Plus
Professional 1-V Statistics Worksheet:
Throw away all the data pairs ( ) ,
i i
Y X where the condition X a > is NOT met.
Using the remaining data pairs to calculate ( ) E Y and ( ) Var Y .
General procedure to calculate ( ) E Y x x a ( <

using BA II Plus and BA II Plus
Professional 1-V Statistics Worksheet:

Throw away all the data pairs ( ) ,
i i
Y X where the condition X a < is NOT met.
Using the remaining data pairs to calculate ( ) E Y and ( ) Var Y .
Find the present value of life insurance or annuity

This occurs frequently. Make sure you know how to calculate.

Example 1

Time t 0 1 2 3
# of people who died at time t 0 0.2 0.3 0.4
Death benefit per death 0 $1,000 $1,000 $1,000
Page 41 of 285 Deeper Understanding: Exam M November 7, 2006 2006 Yufeng Guo
http:/ / www.guo.coursehost.com

The interest rate is 10%

Calculate the present value of the total death benefit.

Solution

Time t 0 1 2 3
# of people who died at
time t
0 0.2 0.3 0.4
Death benefit per death 0 $1,000 $1,000 $1,000
Total death benefits paid
at time t
0 0.2(1,000)
=$200
0.3(1,000)
=$300
0.4(1,000)
=$400
Here we have 3 cash flows:

$200 at t=1
$300 at t=2
$400 at t=3

Present value of the death benefit:

2 3
2 3
200 1 1
200 300 400 300 400 730.28
1.1 1.1 1.1
v v v + + = + + =
This direct calculation is fine for this problem where the # of cash flows is few. However,
if we have many cash flows, then we want to use BA II Plus or BA II Plus Professional
Cash Flow Worksheet to do the calculation for us.

Enter the following cash flows into the Cash Flow Worksheet

$200 at t=1
$300 at t=2
$400 at t=3

Enter 10% as the interest rate. You should get:

NPV=730.2779865

Example 2 (#8 May 2000 Course 3)

For a two-year term insurance on a randomly chosen member of a population:

1/3 of the population are smokers and 2/3 are nonsmokers.

The future lifetimes follow a Weibull distribution with:
Page 42 of 285 Deeper Understanding: Exam M November 7, 2006 2006 Yufeng Guo
http:/ / www.guo.coursehost.com

2 t = and 1.5 u = for smokers.
2 t = and 2 u = for non-smokers.

The death benefit is 100,000 payable at the end of the year of death.

0.05 i = .
Calculate the actuarial present value of this insurance.

Solution

1 2
3 3
S NS
NSP NSP NSP = +
Smoker: ( )
2
1.5
t t
s t e e
t
u
| | | |

| |
\ . \ .
= =
Time t 0 1 2
# of people alive
2
1.5
t
e
| |

|
\ .
1
2
1
1.5
e
| |

|
\ .
2
2
1.5
e
| |

|
\ .
# of deaths

2
1
1.5
1 e
| |

|
\ .

2 2
1 2
1.5 1.5
e e
| | | |

| |
\ . \ .

Use the Cash Flow Worksheet to find PV.



2
1
1.5
01 1 C e
| |

|
\ .
= ,
2 2
1 2
1.5 1.5
02 C e e
| | | |

| |
\ . \ .
= , 5 I = , 0.77000242 NPV =
Non-smoker: ( )
2
2
t t
s t e e
t
u
| | | |

| |
\ . \ .
= =
Time t 0 1 2
# of people alive
2
2
t
e
| |

|
\ .
1
2
1
2
e
| |

|
\ .
2
2
2
e
| |

|
\ .
# of deaths

2
1
2
1 e
| |

|
\ .

2 2
1 2
2 2
e e
| | | |

| |
\ . \ .

Use the Cash Flow Worksheet to find PV.



2
1
2
01 1 C e
| |

|
\ .
= ,
2 2
1 2
2 2
02 C e e
| | | |

| |
\ . \ .
= , 5 I = , 0.58338369 NPV =
Page 43 of 285 Deeper Understanding: Exam M November 7, 2006 2006 Yufeng Guo
http:/ / www.guo.coursehost.com

If the death benefit is $1, then

( ) ( )
1 2
0.77000242 0.58338369 0.64559
3 3
NSP = + =
Because the death benefit is $100,000, NSP should be $64,559.

How to eliminate errors in complex calculations

Problem 1 (Reserve example)

Given:

20
l
9,617,802
30
l
9,501,381
50
l
8,950,901
50
A
0.24905
20
a 16.5133
30
a 15.8561
50
a 13.2668
Interest rate 6%
Calculate
10 30
20 30
20 50 20
50
30 50
30
20 50
20
l
a v a
l l
V A v
l
l
a v a
l



Solution

This calculation is complex. Unless you use a systematic method, youll make mistakes.

Calculation steps using BA II Plus/BA II Plus Professional:

Step 1 Simplify calculations

Page 44 of 285 Deeper Understanding: Exam M November 7, 2006 2006 Yufeng Guo
http:/ / www.guo.coursehost.com

10 10 10 30 30
20 30 20 30 20 30
30 20 20 20 20 50 20 30 20
50 50 50
30 30 50
30 30 50
20 50 20 50
20 50
50 20 20
50 20
1 1 1
1 1
1
l l
a v a a v a a v a
l l l l l l
A v A v A v
l
l l
a v a a v a
a v a
l l l
l l
| |

|
\ .
= =
| |

|
\ .




1
1.06 v

=
20 30 10
20 30 20
50
20 50 30
50 20
1.06
1.06
1.06
a a
l l
V A
a a
l l




Make sure you dont make mistakes in simplification. If you are afraid of making
mistakes, dont simplify and just do your calculations using the original equation:

10 30
20 30
20 50 20
50
30 50
30
20 50
20
l
a v a
l l
V A v
l
l
a v a
l



Step 2 Assign a memory to each input in the formula above

Input Memory Value
20
l
M0 9,617,802
30
l
M1 9,501,381
50
l
M2 8,950,901
50
A
M3 0.24905
20
a M4 16.5133
30
a M5 15.8561
50
a M6 13.2668
After you assign a memory to each input, the formula becomes:

( )
20 30 10
10
20 20 30 20
50
30
20 50 30
50 20
4 5
1.06
1.06
1 0
1.06 3 1.06
4 6
1.06
1.06
2 0
a a
M M
l l
M M
V A M
a a M M
M M
l l

= =




Calculator key sequence to assign memories to the inputs:

Page 45 of 285 Deeper Understanding: Exam M November 7, 2006 2006 Yufeng Guo
http:/ / www.guo.coursehost.com

Procedure Keystroke Display
Set to display 8 decimal
places

2
nd
Format 8 Enter
DEC=8.00000000
Set AOS (Algebraic
operating system)

2nd [FORMAT],
keep pressing + multiple
times until you see Chn.
Press 2nd [ENTER]
(if you see AOS, your
calculator is already in AOS,
in which case press
[CLR Work] )

AOS
Clear existing numbers
from the memories

2
nd
MEM 2
nd
CLR Work
M0=0.00000000
Enter 9,617,802 in M0
9,617,802 Enter
M0=9,617,802.000
Move to the next memory
+
M1=0.00000000
Enter 9,501,381 in M1
9,501,381 Enter
M1=9,501,381.000
Move to the next memory
+
M2=0.00000000
Enter 8,950,901 in M2
8,950,901 Enter
M2=8,950,901.000
Move to the next memory
+
M3=0.00000000
Enter 0.24905 in M3
0.24905 Enter
M3=0.24905000
Move to the next memory
+
M4=0.00000000
Enter 16.5133 in M4
16.5133 Enter
M4=16.51330000
Move to the next memory
+
M5=0.00000000
Enter 15.8561 in M5
15.8561 Enter
M5=15.85610000
Move to the next memory M6=0.00000000
Page 46 of 285 Deeper Understanding: Exam M November 7, 2006 2006 Yufeng Guo
http:/ / www.guo.coursehost.com

+
Enter 13.2668 in M6
13.2668 Enter
M6=13.26680000
Leave the memory
workbook and get back to
the normal calculation
mode

CE/C
This is the button on the
bottom left corner. This is the
same button for
CLR Work
Step 3 Double check data entry.

Dont bypass this step; its easy to enter wrong data.

Keystrokes: press 2
nd
MEM. Then keep pressing the down-arrow key + to view all the
data you entered in the memories. Make sure all the correct numbers are entered.

Step 4 Do the final calculation.

( )
10
20
30
4 5
1.06
1 0
3 1.06
4 6
1.06
2 0
M M
M M
V M
M M
M M

Well break down the calculation into two pieces:



10
4 5
1.06 7
1 0
M M
M
M M

= (store the result in M7)



30
4 6
1.06 8
2 0
M M
M
M M

= (store the result in M8)


( )
20
7
3 1.06
8
M
V M
M

=
Page 47 of 285 Deeper Understanding: Exam M November 7, 2006 2006 Yufeng Guo
http:/ / www.guo.coursehost.com

Procedure Keystroke Display
Calculate
4
1
M
M

10
5
1.06
0
M
M

Recall 4 Recall 1 - Recall 5 Recall 0


1.06
x
y 10 +/- =
0.00000082
Store the result in M7.
Go back to the normal
calculation mode.

STO 7 CE/C
Calculate
30
4 6
1.06
2 0
M M
M M

Recall 4 Recall 1 - Recall 5 Recall 0


1.06
x
y 10 +/- =
0.00000160
Store the result in M8.
Go back to the normal
calculation mode.

STO 8 CE/C
Calculate
( )
20
7
3 1.06
8
M
V M
M

=
Recall 3 1.06
x
y 20 +/-
Recall 7 Recall 8
0.0399556010
So 0.0399556 0.04 V = ~
Though this calculation process looks long, once you get used to it, you can do it in less
than one minute.

Advantages of this calculation process:

Inputs are entered only once. In this problem,
20
l and
20
a are used twice in the
formula
20 30 10
20 30 20
50
20 50 30
50 20
1.06
1.06
1.06
a a
l l
V A
a a
l l



. However, we enter
20
l and
20
a into
the memories only once. This reduces data entry error.

This process gives us a good auditing trail, enabling us to check the data entry and
calculations.

Page 48 of 285 Deeper Understanding: Exam M November 7, 2006 2006 Yufeng Guo
http:/ / www.guo.coursehost.com

We can isolate errors. For example, if a wrong value of
30
l is entered into the
memory, we can reenter
30
l , recalculate
20 30 10
30 20
1.06
a a
l l


, and store the
calculated value into M7. Next, we recalculate ( )
20
7
3 1.06
8
M
V M
M

= .
Bottom line: I recommend that you master this calculation method. The extra work
enables you to do messy calculations 100% right on the exam.

When exams get tough and calculations get messy, many candidates who know as much
as you do will make calculation errors here and there and fail the exam. In contrast, youll
stand above the crowd and make no errors, passing the exam.

Problem 2 (Reserve example revised)

In Problem 1, you calculated that 0.04 V = . However, none of the answer choices given
is 0.04. Suspecting that you made an error in calculations, you decided to redo the
calculation. First, you scrolled over the memories and gladly you found no error in data
entry. Next, you recalculated
10
4 5
1.06 7
1 0
M M
M
M M

= and
30
4 6
1.06 8
2 0
M M
M
M M

= .
Once again, you found your previous calculations were right. Finally, you recalculated
( )
20
7
3 1.06
8
M
V M
M

= . Once again, you got 0.04 V = .


You already spent four minutes on this problem. You decided to spend two more minutes
on this problem. If you couldnt figure out the right answer, you just had to give up and
move on to the next problem.
So you quickly read the problem again. Oops! You found that your formula was wrong.
Your original formula was:

10 30
20 30
20 50 20
30 50
30
20 50
20
50
l
a v a
l l
V
l
a
l
A v
l
v a



The correct formula should be:

Page 49 of 285 Deeper Understanding: Exam M November 7, 2006 2006 Yufeng Guo
http:/ / www.guo.coursehost.com

10 30
20 30
20 50 20
30 50
30
20 50
20
50
l
a v a
l l
V v
l
l
a v
a
a
l




How could you find the answer quickly, using the correct formula?

Solution

The situation described here sometimes happens in the actual exam. If you dont use a
systematic method to do calculations, you wont leave a good auditing trail. In that case,
all your previous calculations are gone and you have to redo calculations from scratch.
This is awful.

Fortunately, you left a good auditing trail. Correcting errors was easy.

Your previous formula after assigning memories to inputs:

( )
10 30
20 30
20 20 50 20
30 50
30
20 50
20
50
7
1.06
8
3
l
a v a
l l M
V v
l
l M
M
a v a
l
A

= =




The correct formula is:

( )
10 30
20 30
20 20 50 20
30 50
30
20 50
2
50
0
7
1. 6 06
8
l
a v a
l l M
V v
l
l M
a v
a M
a
l

= =




Remember
50
6 a M =
You simply reuse M7 and M8 and calculate

( )
20
7
1.06 2.10713362 2.11
8
6
M
V
M
M

= = ~
Now you look at the answer choices again. Good. 2.11 is there!

Page 50 of 285 Deeper Understanding: Exam M November 7, 2006 2006 Yufeng Guo
http:/ / www.guo.coursehost.com

Chapter 3 Survival distribution
Age-at-death random variable of a newborn

X =age-at-death (lifetime) of a newborn (a random variable)

We assume that
( )
X
F x exists and is continuous.
( )
X
F x is differentiable so ( )
X
f x exists

( )
X
F x = cdf (cumulative distribution function) of X
= ( ) P X x s
= ( ) newborn dies by age P x
( )
X
S x = Survival function of X
( ) 1
X
F x =
( ) newborn will surive to age P x =
( ) ( ) ( ) ( )
X X X
d d
f x P X x F x S x
dx dx
= = = =
What makes a good survival function

( ) 0 1
X
S =
( ) 0 1
X
S x s s , where 0 x >
( ) ( )
X X
S x S x a > + , where 0 a > (decreasing function)
( ) lim 0
X
x
S x
e
= , where e is the limiting (maximum) age

Age-at-death random variable of an individual aged x
( ) x =an individual aged x
( ) T x = age-at-death (lifetime) of an individual aged ( ) x
( ) T x is a random variable
Page 51 of 285 Deeper Understanding: Exam M November 7, 2006 2006 Yufeng Guo
http:/ / www.guo.coursehost.com

Distribution function of ( ) T x
( ) ( ) f t P T t = =
( ) ( ) ( ) dies by age
t x
F t P T x t q P x x t = s = = + ( (

( ) ( ) 1 survives to age
t x t x
p q P x x t P T x t = = + = > ( (

Relationship between ( ) T x and X
X = age-at-death (lifetime) of a newborn
( ) T x = age-at-death (lifetime) of individual aged ( ) x
( ) 0 X T =
( )
( )
( )
( ) ( )
( )
1
X
T x
f x t
d
f t P X x t X x s x t
s x s x dx
+
= = + > ( = = +


( )
( )
t x T x
q F x =
( ) P T x t = s (

( ) dies in years P x t = (

( ) dies before reaching age P x x t = + (

newborn dies between and newborn survived to age P x x t x = + (

P x X x t X x = < < + > (

( )
( )
P x X x t
P X x
< < +
=
>
( ) ( )
( ) 1
X X
X
F x t F x
F x
+
=

( ) ( )
( )
X X
X
S x S x t
S x
+
=
( )
( )
newborn dies between x and
newborn survived to age
P x t
P x
+
=
1
t x t x
p q =
( ) P T x t = > (

Page 52 of 285 Deeper Understanding: Exam M November 7, 2006 2006 Yufeng Guo
http:/ / www.guo.coursehost.com

( ) survives next years P x t = (

newborn survives to age newborn survived to age P x t x = + (

P X x t X x = > + > (

( )
( )
P X x t
P x x
> +
=
>
( )
( )
S x t
S x
+
=
Special case when 1 t = . Well drop t
( )
( ) ( )
( )
1
1
dies in one year
x x
S x S x
q q P x
S x
+
= = = (


( )
( )
( )
1
1
survives one year 1
x x x
S x
p p P x q
S x
+
= = = = (


Difficult symbol

( ) dies between and
x t u
q P x x t x t u = + + + (

( ) P t T x t u = < s + (

[ ] P x t X x t u = + < s + +
( ) ( )
( )
S x t S x t u
S x
+ + +
=
( )
( )
( ) ( )
( )
S x t S x t S x t u
S x S x t
+ + + +
=
+
t x u x t
p p
+
=
Curtate future lifetime of ( ) x
( ) T x
Continuous random variable
Represents the EXACT time-to-death for ( ) x
( ) K x
Discrete random variable
Page 53 of 285 Deeper Understanding: Exam M November 7, 2006 2006 Yufeng Guo
http:/ / www.guo.coursehost.com

Represents the FULL years lived by ( ) x
Equal to ( ) T x rounded DOWN to an integer
Example. If ( ) 5.3 T x = , then ( ) 5 K x = ; If ( ) 0.3 T x = , then ( ) 0 K x = .
( ) ( ) dies between and 1 P K x k P x x k x k = = + + + ( (

( ) 1 P k T x k = s < + (

( ) 1 P k T x k = < s + (

The above equation stands because we assume that
( )
( )
T x
F x is continuous. It
doesnt matter whether we include or exclude a single point. As a result,
( ) ( ) P T x k P T x k > = > ( (

and ( ) ( ) 1 1 P T x k P T x k < + = s + ( (

.
Continue

( ) ( ) 1 P K x k P k T x k = = < s + ( (

1 x x k k
q q = =
1 k x k x
p p
+
=
So
1 x k x k x k
q p p
+
= is the probability distribution of ( ) K x
( )
0
1
k
P K x k

=
= = (

(Sum of all the probabilities should be one)



Double check:
( ) ( )
1 0
0 0
1
k x k x x
k k
P K x k p p p
+
= =

= = = = (


(OK)

for limiting age e =
( ) ( )
0 0
1
k k
P K x k P K x k
e
= =

= = = = ( (



Page 54 of 285 Deeper Understanding: Exam M November 7, 2006 2006 Yufeng Guo
http:/ / www.guo.coursehost.com

Force of mortality (instantaneous death rate/mortality) - ( ) x and
( )
x
t
( )
( )
0
0
lim
x
P T x x
x
x

A
< < A (

=
A
0
lim
x
P x X x x X x
x
A
< < + A > (

=
A
( ) ( )
( )
0
lim
x
S x S x x
S x
x
A
+ A
=
A
( ) ( )
( )
0
1
lim
x
S x S x x
x S x
A
( + A
=
(
A

( )
( ) ( )
0
1
lim
x
S x x S x
S x x
A
+ A
A

(
=
(

( )
( )
1 d
S x
S x dx
=
( ) ln
d
S x
dx
=
( ) ( )
( ) ( )
0 0
0
x x
t dt t dt
S x S e e

= =
} }
For a small x A , ( )
( ) ( )
( )
S x S x x
x x
S x

+ A
A ~ P x X x x X x = < < + A > (

( ) x x A is the conditional probability that a newborn will die in the age interval
] (
, x x x + A given it has lived to age x
( ) ( )
0
( ) lim
x
t
P t T x t t T x t
t
t

A
( < < + A >

=
A
Page 55 of 285 Deeper Understanding: Exam M November 7, 2006 2006 Yufeng Guo
http:/ / www.guo.coursehost.com

0
lim
t x t t x
t x
t
p p
p
t
+A
A

=
A
0
1
lim
t t x t x
t x
t
p p
p t
+A
A

A

(
=
(

1
t x
t x
d
p
p dt
= ( ) ln
t x
d
p
dt
=
Alternatively,

( ) ( )
0
( ) lim
x
t
P t T x t t T x t
t
t

A
( < < + A >

=
A
0
lim
x t
t
q
t
A
A
=
A
t x
d
q
dt
=
( ) ( )
0 0
0
x x
t t
t x x
s ds s ds
p p e e

= =
} }

For a small t A , ( ) ( ) ( )
x x t
t t P t T x t t T x t q
A
( A ~ < < + A > =


( ) x x A is the conditional probability that ( ) x will die in the age interval
] (
, x t x t t + + + A given it has lived to age x t + .
FAQs

Q1 ( ) S x and
t x
p are both survival functions. Whats the difference?
A ( ) S x is the survival function of a newborn;
t x
p is the survival function of
someone aged ( ) x . Mathematically, ( ) ( ) newborn will survive to age S x P x = .
( ) survives next years
t x
p P x t = (

Q2 Whats the difference between ( ) x and ( )
x
t
A ( ) x is the instant death rate of a newborn at age x . ( )
x
t is the instant death
rate of ( ) x at time t (i.e. at age x t + ).

Q3 ( )
x
t is the instant date rate. How can it be over 100%? The population cant die
over 100%.
Page 56 of 285 Deeper Understanding: Exam M November 7, 2006 2006 Yufeng Guo
http:/ / www.guo.coursehost.com

A ( )
x
t is the annualized instant death rate at time t when the insured is aged
x t + . We calculate ( ) t in three steps. First, we calculate the probability that age x ,
given that hes alive at time t , dies in t A years:

( ) ( )
t x t t x
x
t x
t
p p
q P t T x t t T x t
p
+A
A

( = < < + A > =


Next, we annualize this death rate:

1 year 1
annualized
year
x x x t t t
q q q
t t
A A A
| |
| |
= =
| |
A A
\ .
\ .

Finally, we find the instant annualized death rate:

0
( ) lim
x t
t x
t
q
d
x q
t dt

A
A
= =
A
( ) t can exceed 100% due to annualization. If you annualize something, you assume a
pattern that lasts only for a short period of time lasts for a full year. As a result, you may
get a very big number.

For example, you invested $100 for a full year. Your return is 5% overall for the whole
year, but you got an exceptional return of 15% in December (i.e. if you invested $100 on
December 1, your investment increases to $115 on December 31). If you annualize your
December return, you are assuming that every month you get a 15% return. The
annualized return is 15%(12)=180% (using simple interest) or
12
1.15 1 435% = (using
compounding interest).

By the same reasoning, ( ) x can exceed 100%.

Q4 I have trouble memorizing the difference between
t x
p and
t x
q . Can you help?
A Remember: p = Persist (live) q =Quit (die)

Q5 Please help me memorize this crazy formula:
( ) ( )
( )
x t u
S x t S x t u
q
S x
+ + +
=
A Find the intuition behind this formula.

( ) [ ] % of die during the age interval ,
x t u
q x x t x t u = + + +
Page 57 of 285 Deeper Understanding: Exam M November 7, 2006 2006 Yufeng Guo
http:/ / www.guo.coursehost.com

[ ] # of deaths during the age interval ,
# of people alive @ age
x t x t u
x
+ + +
=
# of people alive @ age # of people alive @ age
# of people alive @ age
x t x t u
x
+ + +
=
We can treat ( ) S x as representing the number of people alive at age x . For example, we
can interpret ( ) 0 1 S x = = as meaning that the number of newborns is 1 unit (you can
think of one unit as representing 1 billion people). So we start off with 1 unit of people
alive.

If ( ) 20 0.85 S x = = , we then interpret this as meaning that the number of people alive at
age 20 is 0.85 unit. So if we start off with 1 unit of people alive at age zero, well have
0.85 unit of people still alive at age 20.

Then it follows:

# of people alive @ age # of people alive @ age
# of people alive @ age
x t u
x t x t u
q
x
+ + +
=
( ) ( )
( )
S x t S x t u
S x
+ + +
=
Following the same line of thinking, you can easily memorize:

[ ] survival rate over the age interval ,
t x
p x x t = +
# of people alive @ age
# of people alive @ age
x t
x
+
=
( )
( )
S x t
S x
+
=
Page 58 of 285 Deeper Understanding: Exam M November 7, 2006 2006 Yufeng Guo
http:/ / www.guo.coursehost.com

Common problems and model solutions

Type 1 From ( ) S x to force of mortality
x

The survival function is ( ) S x . Calculate


x

( ) [ ] instant death rate during age , x x x dx = +


[ ] # of deaths during age ,
# of people alive @ age
x x dx
x
+
=
( )
( )
d
s x
dx
s x
=

Problem 1 Sample M #32

Given: The survival function ( ) s x , where

( ) 1 s x = , 0 1 x s <
( ) 1
100
x
e
s x = , 1 4.5 x s <
( ) 0 s x = , 4.5 x >
Calculate ( ) 4
Solution

( ) 1
100
x
e
s x = , 1 4.5 x s <
( ) 1
100 100
x x
d d e e
s x
dx dx
| |
= =
|
\ .

( )
( )
( )
1
100
100 1
1
100
x
x x
d e
s x
dx
x
e s x e


= = =

( )
4
1
4 1.2025
100 1 e


= =

Page 59 of 285 Deeper Understanding: Exam M November 7, 2006 2006 Yufeng Guo
http:/ / www.guo.coursehost.com

Problem 2

( ) 1 , 0 120
120
x
s x x = s s
Calculate
20

Solution

( )
( ) 1
x
ds x
s x dx
=
( )
1
120
d
s x
dx
=
1 1 1
120 120
1
120
x
x
x

| |
= =
|

\ .

20
1
1%
120 20
= =

Type 2 From ( ) S x find ( ) f t , ( ) E T , ( ) Var T


The survival function is ( ) S x . Calculate ( ) f t , ( ) E T , and ( ) Var T
( )
( ) ds t
f t
dt
= , ( ) ( )
0
E T S t dt

=
}
,
( ) ( )
2 2
0
E T S t dt

=
}
, ( ) ( ) ( )
2 2
Var T E T E T =
Problem 3

The survival function of a newborn is ( ) 121 s t k t = where k is a constant and
0 121 t s s .
Calculate ( ) 40 f , ( ) E T , and ( ) Var T
Page 60 of 285 Deeper Understanding: Exam M November 7, 2006 2006 Yufeng Guo
http:/ / www.guo.coursehost.com

Solution

First, we need to find k .
( )
1
0 0 121 0 1
11
s k k = = =
( )
( )
( )
( ) ( )
1
2
1 1
2 2
1
121
1 1 1
11
121 121
11 2 22
d t
ds t
f t t t
dt dt

| |
= = = =
|
\ .

( ) ( )
1
2
1 1 1
21 121 21 0.4545
11 2 220
f
| |
= = =
|
\ .

( ) ( )
121 121
0 0
1
121
11
E T s t dt t dt = =
} }

( )
121
1
1
1
1
2
2
0
1 1 1 2
121 121 80.6667
1
11 11 3
1
2
t
+
+
( | |
( |
| |
= = =
( | |
\ .
( |
+
(
\ .

( ) ( )
121 121
2 2
0 0
1
121 2
11
E T s t dt t tdt = =
} }

To simplify the integration, we set 121 s t =
( ) ( ) ( )
0
2
121
2
121 7, 808.5333
11
E T s s d s = =
}
( ) ( ) ( )
2 2 2
7, 808.5333 80.6667 1, 301.4168 Var T E T E T = = =
General formula:

If a non-negative random variable X has a survival function of ( ) S x , then
( ) ( ) ( )
0 0
n n n
E x x f x dx s x dx

= =
} }
. To see why, notice that
Page 61 of 285 Deeper Understanding: Exam M November 7, 2006 2006 Yufeng Guo
http:/ / www.guo.coursehost.com

( ) ( ) ( ) ( ) 1
d d d
f x F x S x S x
dx dx dx
= = = (


( ) ( ) ( ) ( ) 1 f x dx dF x d S x d S x = = = ( (

( ) ( ) ( ) ( ) ( )
0
0 0 0
n n n n n
E x x f x dx x d S x x S x S x d x

( ( = = = + (
} } }

( ) ( ) ( ) ( )
0
0 0
n n n n
x S x S S S

( = =


Because ( ) S approaches 0 faster than
n
approaches , ( ) 0
n
S =
( ) ( )
0
n n
E x S x dx

=
}
Type 3 From ( ) S x to
t x
p ,
t x
q
The survival function is ( ) S x . Calculate
t x
p and
t x
q
( )
( )
t x
S x t
p
S x
+
= , 1
t x t
q p =
Example 1

( )
2
1
120
x
S x
| |
=
|
\ .
, for 0 120 x s s
Calculate
10 35
p
Solution

( )
( )
( )
( )
2
10 35 2
45
1
35 10 45
120
93.93%
35 35
35
1
120
S S
p
S S
| |

|
+
\ .
= = = =
| |

|
\ .

Page 62 of 285 Deeper Understanding: Exam M November 7, 2006 2006 Yufeng Guo
http:/ / www.guo.coursehost.com

Example 2 (#8 CAS Fall, 2004)

Given ( ) 1
100
x
S x = for 0 100 x s s , calculate the probability that a life age 36 will
die between age 51 and 64.

Solution

( ) ( )
( )
51 64
1 1
51 64
0.49 0.36 0.7 0.6
100 100
0.125
36 0.8 36 0.64
1
100
S S
S


= = = = =

Example 3 (#16 May 2000 SOA EA-1)



For a certain population the survival function is

( )
100
, for 0 100
10
x
S x x

= s s
Calculate the probability that a member of that population age 19 will not die between the
ages of 51 and 64.

Solution

( ) 19 not die between age 51 and 64 P (

( ) 1 19 die between age 51 and 64 P = (

( ) 19 die between age 51 and 64 P (

( ) ( )
( )
51 64
19
S S
S

=
100 51 100 64
49 36 7 6 1
10 10
9 9 100 19 81
10


= = = =

( )
8
19 not die between age 51 and 64
9
P = (


Page 63 of 285 Deeper Understanding: Exam M November 7, 2006 2006 Yufeng Guo
http:/ / www.guo.coursehost.com

Type 4 From a series of
t x
q find
x x t
q
Given
x
q ,
x i
q
+
,
x j
q
+
,. Find
n x k m
q
+
( ) ( )
( )
# of people alive at age # of people alive at age
# of people alive at age
n x k m
x k m x k m n
q
x k
+
+ + + +
=
+
+
Example 1

60
1% q = ,
61
2% q = ,
62
3% q = ,
63
4% q =
Calculate the probability that ( ) 60 dies between age 62 and 64.

Solution

Probability that ( ) 60 dies between age 62 and 64.

# of people alive @ age 62 # of people alive @ age 64
# of people alive @ age 60
=

Set # of people alive @ age 60 1 =
( )
60
# of people alive @ age 61 # of people alive @ age 60 p =
60 60
1 99% p q = = =
( )
61
# of people alive @ age 62 # of people alive @ age 61 p =
( ) 99% 1 2% 97.02% = =
( )
62
# of people alive @ age 63 # of people alive @ age 62 p =
( ) 97.02% 1 3% 94.109% = =
( )
63
# of people alive @ age 64 # of people alive @ age 63 p =
( ) 94.109% 1 4% 90.345024% = =
Page 64 of 285 Deeper Understanding: Exam M November 7, 2006 2006 Yufeng Guo
http:/ / www.guo.coursehost.com

# of people alive @ age 62 # of people alive @ age 64
# of people alive @ age 60
=

97.02% 90.345024%
6.67%
1

= ~
Type 5 What makes a good
x

( ) x must satisfy the following requirements:



( ) 0 x > for 0 x >
( )
0
x dx

=
}
Example 1 (#7 CAS Fall, 2004)

Which of the following formulas could serve as a force of mortality?

1 , 0, 1
x
x
BC B C = > >
2 ( )
1
, 0, 0
x
a b x a b

= + > >
3 ( )
3
1 , 0
x
x x

= + >
[A] 1 only
[B] 2 only
[C] 3 only
[D] 1 and 2 only
[E] 1 and 3 only

Solution

Actuarial Mathematics Table 3.2.1 (page 57) states that ( ) x must satisfy the following
requirements:

( ) 0 x > for 0 x >
( )
0
x dx

=
}
Page 65 of 285 Deeper Understanding: Exam M November 7, 2006 2006 Yufeng Guo
http:/ / www.guo.coursehost.com

We see that 1,2, and 3 all have a non-negative force of mortality. So the first condition is
satisfied. We just need to check ( )
0
x dx

=
}
.
0 0
ln
x x
B
BC dx C
C

(
= =
(

}
So 1 is okay.

( ) ( )
1
0
0
ln a b x dx a b x


+ = + = (
}
So 2 is okay.
( ) ( )
3 2
0 0
1 1
1 1
2 2
x dx x

(
+ = + =
(

}
So 3 is not okay.

So [D]

Type 6 From ( )
x
t to
t x
p
( ) ( )
( )
# of people alive at age # of people alive at age
# of people alive at age
n x m
x m x m n
q
x
+ + +
=
Example 1 (#4 CAS Fall, 2003)

Given:

( )
2
100
x
x
=

, for 0 100 x s <


Calculate
65 10
q
Solution

( ) ( )
( )
# of people alive at age # of people alive at age
# of people alive at age
n x m
x m x m n
q
x
+ + +
=
In the above formula, we adopted a deterministic view. In reality though, the number of
people alive at a particular point in time is a random variable. We assume that the number
Page 66 of 285 Deeper Understanding: Exam M November 7, 2006 2006 Yufeng Guo
http:/ / www.guo.coursehost.com

of people alive at any time is known in advance. Such a view often allows us to quickly
solve a problem using 6
th
-grade math, as opposed to using complex probability theories.

So
( ) ( )
( )
65 10
# of people alive at age 65 10 # of people alive at age 65 10 1
# of people alive at age 65
q
+ + +
=
To begin with, we set

# of people alive at age 0 = 1.

We start off at age 0 because
x
given to us to starts off at age 0.
( ) ( )
0
# of people alive @ age # of people alive @ age 0 exp -
t
t x dx
(
=
(

}
( )
0
exp -
t
x dx
(
=
(

}
Now we have the theoretical framework to calculate
65 10
q . The rest of the work is simply
integration.

( )
( )
( )
0
0 0 0
10
2
- - 2 2 ln 100 2ln 1
100 100 100
t t t
t d x
t
x dx dx x
x x


| |
= = = = (
|

\ .
} } }

2
2
# of people alive @ age exp 2ln 1 exp ln 1 1
100 100 100
t t t
t
( ( | | | | | |
= = =
`
| | | ( (
\ . \ . \ . )

( )
2
2
65
# of people alive at age 65 1 0.35
100
| |
= =
|
\ .

( )
2
2
75
# of people alive at age 65 10 1 0.25
100
| |
+ = =
|
\ .

( )
2
2
76
# of people alive at age 65 10 1 1 0.24
100
| |
+ + = =
|
\ .

2 2
65 10 2
0.25 0.24 1
0.04
0.35 25
q

= = =
To speed up your calculation in the exam, you might want to memorize:
Page 67 of 285 Deeper Understanding: Exam M November 7, 2006 2006 Yufeng Guo
http:/ / www.guo.coursehost.com

x
k
x

e
=

where

k is a positive constant
e is the limiting age
0 x e s <
# of people alive @ age exp ln 1 1
k
t t
t k
e e
( | | | |
= =
| | (
\ . \ .

Type 7 Two independent lives, first death after m years but before n
years

( ) x and ( ) y are two independent lives. Whats the probability that the first death occurs
m years from now but before n years (where n m > ) from now?
Answer:
m x m y n x n y
p p p p
Let
( ) ( )
min
min ,
x y
T T T
(
=

Probability that the first death occurs after m years and before n years from now:

( ) ( )
min min
P m T n P m T n s s = < s ( ) ( ) min min
T T
F n F m =
In the above equation, we assume that ( ) min
T
F x is continuous. As a result,
( ) ( )
min min
P x T P x T s = <
Continue

( ) ( ) ( ) ( ) min min min min
1 1
T T T T
F n F m S n S m ( ( =

( ) ( ) min min
T T
S m S n =
( ) ( )
( ) ( )
{ }
( ) ( )
{ } min
min
min ,
x y x y
T
S t P T t P T T t P T t T t
(
= > = > = > >


( ) ( ) x y
P T t P T t
( (
= > >

[because ( ) x and ( ) y are independent ]
Page 68 of 285 Deeper Understanding: Exam M November 7, 2006 2006 Yufeng Guo
http:/ / www.guo.coursehost.com

m x m y
p p =
( ) ( ) ( ) ( ) min min min min
m x m y n x n y
T T T T
F n F m S m S n p p p p = =
Then the probability that the first death occurs after m years and before n years from
now is:
m x m y n x n y
p p p p .
Example 1 (#1 CAS Exam 3, Fall 2003)

Given:

3 40
0.990 p =
6 40
0.980 p =
9 40
0.965 p =
12 40
0.945 p =
15 40
0.920 p =
18 40
0.890 p =
For two independent lives aged 40, calculate the probability that the

first death occurs
after 6 years, but before 12 years.

Solution

40, 40 x y = =
6, 12 m n = =
m x m y n x n y
p p p p
( ) ( )
2 2
2 2
6 40 6 40 12 40 12 40 6 40 12 40
0.98 0.945 6.7375% p p p p p p = = = =
Example 2

Two independent lives ( ) x and ( ) y . 4%
x
= . 6%
x
=
Calculate the probability that the first death occurs after 10 years and before 20 years
from now.

Page 69 of 285 Deeper Understanding: Exam M November 7, 2006 2006 Yufeng Guo
http:/ / www.guo.coursehost.com

Solution

m x m y n x n y
p p p p
( )
( )
0
0.04
x
x
t
t t
t x x
s ds
P T t p e e e



(
> = = = =

}
( )
( )
0
0.06
y
y
t
t
t
t x y
s ds
P T t p e e e


(
> = = = =

}
( ) ( ) min min
0.04 10 0.06 10 0.04 20 0.06 20 1 2
10 20 23.25%
T T
S S e e e e e e

= = =
Type 8 Two independent lives, second death after m years but before
n years

( ) x and ( ) y are two independent lives. Whats the probability that the second death
occurs m years from now but before n years (where n m > ) from now?
Answer: ( ) ( ) ( ) ( )
1 1 1 1
n x n y m x m y n x n y m x m y
q q q q p p p p =
Let
( ) ( )
max
max ,
x y
T T T
(
=

Probability that the second death occurs after m years and before n years from now

( ) ( ) ( ) max max
max
T T
P m T n F n F m s s =
( ) ( )
( ) ( )
{ }
( ) ( )
{ } max
max
max ,
x y x y
T
F t P T t P T T t P T t T t
(
= s = s = s s


( ) ( ) x y
P T t P T t
( (
= s s

[because ( ) x and ( ) y are independent ]

( ) ( )
,
m x t y x y
P T t q P T t q
( (
s = s =


( )
( ) ( )
max
m x m y x y
P m T n P T t P T t q q
( (
s s = s s =


Example 1

Two independent lives ( ) x and ( ) y . 4%
x
= . 6%
x
= .
Calculate the probability that the second death occurs after 10 years and before 20 years
from now.
Page 70 of 285 Deeper Understanding: Exam M November 7, 2006 2006 Yufeng Guo
http:/ / www.guo.coursehost.com

Solution

( ) ( ) ( )( )
20 20 10 10 20 20 10 10
1 1 1 1
x y x y x y x y
q q q q p p p p =
( )
0
0.04
x
x
t
t t
t x
s ds
p e e e



= = =
}
( )
0
0.06
y
y
t
t
t
t x
s ds
p e e e


= = =
}
( )( ) ( )( )
0.04 20 0.06 20 0.04 10 0.06 10
20 20 10 10
1 1 1 1 23.61%
x y x y
q q q q e e e e

= =
The probability that the second death occurs after 10 years and before 20 years from now
is 23.61%.

Type 9 Two independent lives, at least one will die in n years

( ) x and ( ) y are two independent lives. Whats the probability that at least one of them
will die in n years?
Answer: 1
n x n y n x n y n x n y
q q q q p p + =
Proof

Method 1

( ) ( ) at least one of or will die within years P x y n (

( ) ( ) dead in years dead in years P x n y n = (

( ) ( ) dead in years dead in years P x n P y n = + ( (

( ) ( ) dead in years dead in years P x n y n (

( ) x and ( ) y are independent, so
( ) ( ) dead in years dead in years P x n y n (

( ) ( ) dead in years dead in years P x n P y n = ( (

Page 71 of 285 Deeper Understanding: Exam M November 7, 2006 2006 Yufeng Guo
http:/ / www.guo.coursehost.com

( ) dead in years
n x
P x n q = (

, ( ) dead in years
n y
P y n q = (


( ) ( ) at least one of or will die within years
n x n y n x n y
P x y n q q q q = + (


Method 2

( ) ( ) at least one of or will die within years P x y n (

( ) ( ) 1 both and alive after years P x y n = (

( ) ( ) 1 alive after years alive after years P x n y n = (

( ) ( ) 1 alive after years alive after years P x n P y n = ( (

1
n x n y
p p =
Method 3

At least one of ( ) x and ( ) y will die in n years
= 1
st
death of ( ) x and ( ) y occurs in n years

Apply the formula in Type 7:

( ) x and ( ) y are two independent lives. Then the probability that the

first death occurs m
years from now but before n years (where n m > ) from now is
m x m y n x n y
p p p p .
At least one of ( ) x and ( ) y will die in n years
= 1
st
death of ( ) x and ( ) y occurs in n years
= 1
st
death of ( ) x and ( ) y occurs zero years from now but before n years
0 0 x y n x n y
p p p p =
1
n x n y
p p =
Example 1 (#10 CAS Fall, 2004)

For John, currently 30 years old, the force of mortality is
1
100
x
x
=

For Bob, an independent life also 30 years old, it is known that



Page 72 of 285 Deeper Understanding: Exam M November 7, 2006 2006 Yufeng Guo
http:/ / www.guo.coursehost.com

10 30
0.94 p =
5 35
0.96 p =
Calculate the probability that at least one of these two men, will die within five years.

Solution

5 30 5 30 5 30 5 30
J B J B
q q q q = +
( )
5 30
John dead within 5 years 1
J
P p =
( )
( )
5 30
35
30
J
J
J
S
p
S
= (general formula
( )
( )
t x
S x t
p
S x
+
= )
Applying the formula ( )
( )
0
x
x
s ds
S x e

=
}
, we have

( )
0
1
100
x
ds
s
J
S x e

}
=
0
0
1 100
ln 100 ln ln 1
100 100 100
x
x x x
ds s
s
| | | |
= ( = =
| |

\ . \ .
}
( )
0
1
ln 1
100
100
1
100
x
x
ds
s
J
x
S x e e
| |


|
\ .
}
= = = ,
( )
( )
5 30
35
1
35
100 35
100
30
30 100 30
1
100
J
J
J
S
p
S


= = =

5 30 5 30
100 35 35 30 5
1 1
100 30 100 30 70
J J
q p

= = = =


( )
5 30
Bob dead within 5 years 1
B
P p =
10 30 5 30 5 35
B B B
p p p = (general formula is:
m n x m x n x m
p p p
+ +
= )
Page 73 of 285 Deeper Understanding: Exam M November 7, 2006 2006 Yufeng Guo
http:/ / www.guo.coursehost.com

10 30
5 30 5 30
5 35
0.94 0.94 1
, 1
0.96 0.96 48
B
B B
B
p
p q
p
= = = =
( ) at least one of these two men will die within 5 years P
5 30 5 30 5 30 5 30
J B J B
q q q q = +
5 1 5 1
9.077%
70 48 70 48
= + ~
Since
1
x
x

e
=

shows up a lot on the exam, you might want to memorize:



0
0
1
ln ln 1
x
t t
ds s
s
e
e e
| |
= ( =
|

\ .
}
( )
ln 1
1
x
x
S x e
e
e
| |

|
\ .
= = ,
( )
( )
( )
1
1
1
t x
x t
S x t x t
t
p
x
S x x x
e
e
e e
e
+

+ +
= = = =

1
t x t x
t
q p
x e
= =

This is De Moivres Law



Type 10 Multiple independent lives

Use the general probability theory to solve the problem.

Example 1 (#23 May 2001 EA-1)

Consider the following:

Smith, age 20, with
20
0.95
n
n
p = , 0 n >
Brown, age 25, with
25
0.90
n
n
p = , 0 n >
Green, age 30, with
30
0.85
n
n
p = , 0 n >
Page 74 of 285 Deeper Understanding: Exam M November 7, 2006 2006 Yufeng Guo
http:/ / www.guo.coursehost.com

In what range is the probability that all three are alive five years from now and at least
two are alive 15 years from now?

Solution

One intuitive approach is to use basic probability theories. First, we define the following
random variables:

S
T = Smiths time to die
B
T = Browns time to die
G
T = Greens time to die

In addition, well use S, B, and G to represent Smith, Brown, and Green respectively.

Scenarios all three are alive 5
years from now, but at least two are
alive 15 years from now
Probability
#1

S,B,G are all alive 15 years from
now (if they are all alive 15 years
from now, of course they are alive 5
years from now)
( ) ( ) ( )
15 15 15
S B G
P T T T
(
> > >

( ) ( ) ( )
15 15 15
S B G
P T P T P T = > > >
, ,
S B G
T T T are independent

( )
15
15 20
15 0.95
S
P T p > = =
( )
15
15 25
15 0.9
B
P T p > = =
( )
15
15 30
15 0.85
G
P T p > = =
( ) ( ) ( )
15 15 15
S B G
P T P T P T > > >
=
( )( )
15 15 15
0.95 0.9 0.85 0.83% =
#2

S and B are alive 15 years from
now, but G dies between years 5
and 15.
( ) ( ) ( )
15 15 5 15
S B G
P T T T
(
> > < <

( ) ( ) ( )
15 15 5 15
S B G
P T P T P T = > > < <
( ) ( ) ( )
15 15
0.95 0.9 5 15
G
P T = < <
( )
5 15
5 30 15 30
5 15 0.85 0.85
G
P T p p < < = =
( ) ( ) ( )
15 15 5 15
S B G
P T T T
(
> > < <

( ) ( ) ( )
15 15 5 15
0.95 0.9 0.85 0.85 3.4% = =
Page 75 of 285 Deeper Understanding: Exam M November 7, 2006 2006 Yufeng Guo
http:/ / www.guo.coursehost.com

#3

S and G are alive 15 years from
now, but B dies between 5 and 15
years.
( ) ( ) ( )
15 15 5 15
S G B
P T T T
(
> > < <

( )( )( )
15 15 5 15
0.95 0.85 0.9 0.9 1.56% = =
#4

B and G are alive 15 years from
now, but S dies between 5 and 15
years.
( ) ( ) ( )
15 15 5 15
B G S
P T T T
(
> > < <

( )( ) ( )
15 15 5 15
0.9 0.85 0.95 0.95 0.56% = =
Total 0.83% 3.4% 1.56% 0.56% 6.35% = + + + =
Page 76 of 285 Deeper Understanding: Exam M November 7, 2006 2006 Yufeng Guo
http:/ / www.guo.coursehost.com

Chapter 4 Life Table

Basic concepts

Start off with
0
l individuals aged 0.
0
l is called radius.
We often set
0
l equal to an arbitrary number such as 10,000.

Let
x
B = number of survivors at age x .
x
B is a random variable.
(You wont be able to find
x
B in the textbook. I made it up.)

x
B has a binomial distribution with parameter ( )
0,
l S x (

( ) ( ) newborn survives to age S x P x =
( ) ( )
0 x x
l E B l S x = =
Where
x
l = number of people out of
0
l who are expected to survive to age x .
Let
x
C = number of deaths between age x and 1 x + .
x
C is a random variable.
(You wont be able to find
x
C in the textbook. I made it up.)

x
C has a binomial distribution with parameter ( ) ( )
0,
1 l S x S x ( +

( ) ( ) ( ) 1 newborn dies between age and 1 S x S x P x x + = +
( ) ( ) ( )
0 1
1
x x x x
d E C l S x S x l l
+
= = + = (


Where
x
d = number of people out of
0
l expected to die between age x and 1 x +
Let
n x
C = number of deaths between age x and x n + .
n x
C is a random variable.
(You wont be able to find
n x
C in the textbook. I made it up.)

n x
C has a binomial distribution with parameter ( ) ( )
0
, l S x S x n + (

( ) ( ) ( ) newborn dies between age and S x S x n P x x n + = +
( ) ( ) ( )
0 n x n x x x n
d E C l S x S x n l l
+
= = + = (


Page 77 of 285 Deeper Understanding: Exam M November 7, 2006 2006 Yufeng Guo
http:/ / www.guo.coursehost.com

Other formulas

x t
t x
x
l
p
l
+
=
1
x t x x t
t x
x x
l l l
q
l l
+ +

= =
x n x n m
x n m
x
l l
q
l
+ + +

=
x x x
d l q =
Complete expectation of future lifetime

( )
( )
( ) ( )
0
0 0 0
x
t x x t t x T x
e E T x t f t dt t p dt p dt

+
= = = = (
} } }

To see why the last equality stands, notice

( )
( ) ( )
( )
( )
( )
( )
( )
( )
1
t x t x
d
S x t
S x t S x t
d d d
dt
p S x t p x t
dt dt S x S x dt S x S x t

+
+ +
= = + = = +
+
( ) ( ) ( )
0
0 0 0
t x x t t x t x t x
t p dt t d p t p p dt

+
= = + (
} } }

( ) ( )( ) ( ) ( ) ( ) ( )
1
0
0 0
t x x x x
t p p p p


= = = (


[because
t x
p approaches zero much faster than t approaches ]
( )
( )
( ) ( )
2 2 2
0 0 0
2
t x x t t x T x
E T x t f t dt t p dt t p dt

+
( = = =
} } }

( ) ( ) ( )
2
0
2 2
0
2 x
t x
Var T x E T x E T x t p dt e

| |
( = = ( (
|

\ .
}
Curtate expectation of life

( )
1 k x k x
P K x k p p
+
= = (


Page 78 of 285 Deeper Understanding: Exam M November 7, 2006 2006 Yufeng Guo
http:/ / www.guo.coursehost.com

( ) ( )
0
x
k
e E K x kP K x k

=
= = = ( (

( )
1 1
0 0 0
k x k x k x k x
k k k
k p p k p k p

+ +
= = =
= =


( )
0 1 1
1
k x k x k x
k k k
k p k p p

= = =
= =

( ) ( )
2 2
0 k
E K x k P K x k

=
( = = (


( )
2 2 2
1 1
0 0 0
k x k x k x k x
k k k
k p p k p k p

+ +
= = =
= =


( ) ( )
2
2
0 1 1
1 2 1
k x k x k x
k k k
k p k p k p

= = =
= =


( ) ( ) ( ) ( )
2 2 2
1
2 1
k x x
k
Var K x E K x E K x k p e

=
( = = ( (


Relationship between
0
x e and
x
e
( ) ( ) ( ) 1 K x T x K x s s +
( ) ( ) ( ) 1 E K x E T x E K x s s + ( ( (

0
1 x
x x
e e e s s +
Relationship between
0
x e and
x
e under UDD

If deaths occur uniformly between integral ages, then

0
1
2
x
x
e e = +
( ) ( )
1
12
Var T x Var K x = + ( (


Less common life table functions

Total expected number of years lived between x and 1 x + by survivors of the initial
group of
0
l lives:

1
0
x x t
L l dt
+
=
}
Page 79 of 285 Deeper Understanding: Exam M November 7, 2006 2006 Yufeng Guo
http:/ / www.guo.coursehost.com

Total expected number of years lived between x and x n + by survivors of the initial
group of
0
l lives:
0
n
n x x t
L l dt
+
=
}
So
1 x x
L L =
Total expected number of years lived beyond x by survivors of the initial group of
0
l
lives:

0
x x x t
T L l dt

+
= =
}
Central death rate at age x
1
0
1
0
x t x t
x
x
x
x t
l dt
d
m
L
l dt

+ +
+
= =
}
}
Average number of years lived between x and 1 x + among
x
d deaths

( )
( )
( )
1
0
1
0
1
t x
x
t p x t dt
x E T T
p x t dt

+
= < ( =

+
}
}
Page 80 of 285 Deeper Understanding: Exam M November 7, 2006 2006 Yufeng Guo
http:/ / www.guo.coursehost.com

Common problems and model solutions

Type 1 Build a life table

Example 1

Lab rats have the following mortality:

x
x
p
0 0.6
1 0.5
2 0.4
3 0.3
4 0.2
5 0.1
6 0.0
Set
0
1, 000 l = . Construct a life for lab rats.
Solution

x
x
p 1
x x
q p =
x
l
1 x x x
d l l
+
=
x
e
0 0.6 0.4 1,000 1,000-600=400 1.06392
1 0.5 0.5 1,000(0.6)=600 600-300=300 0.7732
2 0.4 0.6 600(0.5)=300 300-120=180 0.5464
3 0.3 0.7 300(0.4)=120 120-36=84 0.3660
4 0.2 0.8 120(0.3)=36 36-7.2=28.8 0.22
5 0.1 0.9 36(0.2)=7.2 7.2-0.72=6.48 0.1
6 0.0 1.0 7.2(0.1)=0.72 0.72 0
7 0
6
0 0 1 0 2 0 3 0 4 0 5 0 6 0
1
k
k
e p p p p p p p
=
= = + + + + +

1 0
0.6 p =
2 0 0 1
0.6 0.5 0.3 p p p = = =
3 0 0 1 2
0.6 0.5 0.4 0.12 p p p p = = =
4 0 0 1 2 3
0.6 0.5 0.4 0.3 0.036 p p p p p = = =
5 0 0 1 2 3 4
0.6 0.5 0.4 0.3 0.2 0.0072 p p p p p p = = =
6 0 0 1 2 3 4 5
0.6 0.5 0.4 0.3 0.2 0.1 0.00072 p p p p p p p = = =
0
0.6 0.3 0.12 0.036 0.0072 0.00072 1.06392 e = + + + + + =
Page 81 of 285 Deeper Understanding: Exam M November 7, 2006 2006 Yufeng Guo
http:/ / www.guo.coursehost.com

As a quicker calculation,

1 2 3
1 2 3
...
...
x x x
x x x x x
x
l l l l
e p p p p
l
+ + +

+ + +
= + + + + =
1 2 3 4 5 6
0
0
600 300 120 36 7.2 0.72
1.06392
1, 000
l l l l l l
e
l
+ + + + + + + + + +
= = =
2 3 4 5 6
1
1
300 120 36 7.2 0.72
0.7732
600
l l l l l
e
l
+ + + + + + + +
= = =
3 4 5 6
2
2
120 36 7.2 0.72
0.5464
300
l l l l
e
l
+ + + + + +
= = =
4 5 6
3
3
36 7.2 0.72
0.3660
120
l l l
e
l
+ + + +
= = =
5 6
4
4
7.2 0.72
0.22
36
l l
e
l
+ +
= = =
6
5
5
0.72
0.1
7.2
l
e
l
= = =
Type 2 From the Life Table, find
t x
p and
t x
q
Example 1

Based on the Illustrative Life Table, calculate

(1) The probability that a newborn survives to age 20.
(2) The probability that age 35 survives to age 80.
(3) The probability that age 35 dies in the next 25 years.
(4) The probability that age 35 dies between ages 60 and 75.

Solution

Let X = age-at-death (lifetime) of a newborn.
Let ( ) T x = age-at-death (lifetime) of individual aged ( ) x
(1)
20
20 0
0
9, 617,802
0.9618
10, 000, 000
l
p
l
= = =
(2)
80
80 35 35 45 35
35
3, 914, 365
0.4120
9, 420, 657
l
p p
l

= = = =
(3)
25 35 60
25 35 25 35
35 35
8,188, 074
1 1 1 1 0.1308
9, 420, 657
l l
q p
l l
+
= = = = =
Page 82 of 285 Deeper Understanding: Exam M November 7, 2006 2006 Yufeng Guo
http:/ / www.guo.coursehost.com

(4)
60 75
35
8,188, 074 5, 396, 081
0.2964
9, 420, 657
l l
l

= = =
Example 2

100
x
l x = where 0 100 x s s .
Calculate the probability that ( ) 30 will die after attaining age 60 but before attaining age
80.

Solution

60 80
30
100 60 100 80 40 20 4 2
0.2214
100 30 70 7
l l
l

= = = = =

Type 3 Find mean and variance of the number of survivors



Use binomial and normal approximation.

Example 1

A company offers retirement benefits to its 200 employees. Each employee is exactly 35
years old now. The retirement benefits start when an employee attains age 65.

Using the Illustrative Life Table and normal approximation, find the probability that at
least 20 employees wont survive to age 65 to receive the retirement benefits.

Solution

Let Y represent the number of employees who will receive the retirement benefits.

Y is a binomial random variable with parameter

200 n =
30 35
7, 533, 964
0.8
9, 420, 657
p p = = = .
( )
30 35
200 200 0.8 160 E Y np p = = = =
( )
30 35 30 35
200 200 0.8 0.2 32 Var Y npq p q = = = =
The probability that at least 20 employees wont survive to age 65 is
Page 83 of 285 Deeper Understanding: Exam M November 7, 2006 2006 Yufeng Guo
http:/ / www.guo.coursehost.com

( ) ( )
180 160
200 20 180 3.536 1
32
P Y
| |
s = = u = u ~
|
\ .

Type 4 Recursive formula
x
e
Key formulas ( )
1
1
1 ,
1
x
x x x x
x
e
e p e p
e
+
+
= + =
+
Example 1 (#4, SOA EA-1, 2000)

Smith, Brown, and Green are all age 81.

80
9.8694 e =
81
9.3315 e =
3 80
0.8642 p =
Calculate the probability that at least two of the three will die prior to attaining age 83.

Solution

( ) at least two die before age 83 P
( ) ( )
binomial distribution binomial distribution
two dead by age 83 one alive at 83 all three dead by age 83 P P = +

( ) ( ) ( )
2
2
3 2 81 2 81
two dead by age 83 one alive at 83 P C q p =
( ) ( )
3
2 81
all three dead by age 83 P q =
The key is to find
2 81
q or
2 81
p .
3 80
3 80 80 2 81 2 81
80
,
p
p p p p
p
= =
( )
80 80 81
1 e p e = +
The general formula is: ( )
1
1
x x x
e p e
+
= + . To see why, note
Page 84 of 285 Deeper Understanding: Exam M November 7, 2006 2006 Yufeng Guo
http:/ / www.guo.coursehost.com

1 2 3 1 1 2 3
1
... ...
x x x x x x x
x
x x x
l l l l l l l l l
e
l l l
+ + + + + + +
+
| | + + + + + +
= =
|
\ .
( ) ( )
1 2 3 1
1 1
1
...
1 1 1
x x x x
x x x
x x x
l l l l l
e p e
l l l
+ + + +
+ +
+
| | + +
= + = + = +
|
\ .

( )
1
1
1 ,
1
x
x x x x
x
e
e p e p
e
+
+
= + =
+
( )
80
80 80 81 80
81
9.8694
1 0.95527
1 1 9.3315
e
e p e p
e
= + = = =
+ +

3 80
2 81 2 81
80
0.8642
0.9047, 1 0.9047 0.0953
0.95527
p
p q
p
= = = = =
( ) ( ) ( ) ( ) ( ) ( )
2 3 2 3
2
3 2 81 2 81 2 81
3 0.0953 0.9047 0.0953 2.55% C q p q + = + =
The probability that at least two of the three will die prior to attaining age 83 is 2.55%.

Type 5 From
x
l find
x

Key formula ( ) ln
x
x
x
d
l
d
dx
x l
l dx
= =
Example 1 (#13, SOA EA-1 2002)

From Mortality Table A:
2
20, 000 100
x
l x x =
Mortality Table B has a constant force of mortality equal to
41
from Mortality Table A.
In addition, from Mortality Table B,
45
100, 000 l = .
Calculate
41
l from Mortality Table B.

Solution

Mortality Table A:

Page 85 of 285 Deeper Understanding: Exam M November 7, 2006 2006 Yufeng Guo
http:/ / www.guo.coursehost.com

( )
( )
2
2 2
20, 000 100
2 100
20, 000 100 20, 000 100
x
x
d d
l x x
x
dx dx
x
l x x x x


+
= = =


( )
( )
( )
2
2 41 100
41 1.28%
20, 000 100 41 41

+
= =


Mortality Table B:

( ) ( )
0
0
exp exp
x y x
x y
y
l l s ds l s ds

(
(
= =
(
(
(

} }

( ) ( )
45 41 4
45 41 41 41
0 0
exp exp exp 4 l l s ds l ds l

( (
= = =
( (

} }

( ) 1.28% 4 4
41 45
100, 000 105, 253 l l e e

= = =
Type 6 Force of mortality doubles

Key formula
( ) ( ) ( )
2
2
0 0
exp 2 exp
t t
t x t x
new old old old
p x s ds x s ds p
( (

= + = + =
` ( (
)
} }

Example 1 (#22, SOA EA-1 2004)

Given the following values:

x
x
e
107 0.6
108 0.2
109 0.0
A new table is constructed such that the force of the mortality is doubled.

107
Y e = based on the new table.

Calculate Y .
Page 86 of 285 Deeper Understanding: Exam M November 7, 2006 2006 Yufeng Guo
http:/ / www.guo.coursehost.com

Solution

Under the old table:

110 111
109 110 111
109
...
0 ... 0
l l
e l l
l
+ +
= = = = =
So no one lives beyond age 110.

Since the new table doubles the force of the mortality of the old table and no one lives
beyond age 110 either, we have:

108 109
107 107 2 107
107
new new
new new new new
new
l l
Y e p p
l
+
= = = +
Next, we need to figure out the effect of doubling the force of mortality.

( )
0
exp
t
t x
p x s ds
(
= +
(

}
( a generic formula for any table)

( ) ( ) ( )
2
2
0 0
exp 2 exp
t t
t x t x
new old old old
p x s ds x s ds p
( (

= + = + =
` ( (
)
} }

( ) ( )
2 2
107 107 2 107 2 107
,
new old new old
p p p p = =
2 107
107 107
108
0.6
0.5, 0.5 0.25
1 1 0.2
old
old
old new
e
p p
e
= = = = =
+ +

2 108
108 108
109
0.2
0.2, 0.2 0.04
1 1 0
old
old
old new
e
p p
e
= = = = =
+ +

2 107 107 108
0.25 0.04 0.01
new new new
p p p = = =
107 2 107
0.25 0.01 0.26
new new new
Y p p = + = + =
Page 87 of 285 Deeper Understanding: Exam M November 7, 2006 2006 Yufeng Guo
http:/ / www.guo.coursehost.com

Chapter 5 UDD between integral ages

UDD (uniform distribution of death) between two integral ages is repeatedly tested in
Course 3 and Exam M. To tackle UDD under exam conditions, you need to understand
and memorize the following shortcut:

Shortcut for UDD between [ ] , 1 x x +
Age x x t + 1 x +
Time to die 0 t 1 T
# of people alive 1 1
x
t q 1
x
q
(setting 1
x
l = )
for 0 1 t s s
( )
x
f t q = ,
t x x
q t q = , 1
t x x
p t q = ,
( )
1 1
x x
t
t q
=
1 1 1
2
x x
m q
= ( ) 0.5
x x
m =
1
1
2 0
0
1
2
x
x x t
q
L l dt l l
+
| |
= = =
|
\ .
}
Explanation

First, you need to understand UDD intuitively without using any formulas. Under UDD
between ages x and 1 x + , the speed of deaths is constant between age x and 1 x + .
For example, if 24 people died over one year period, then the constant speed of deaths is
2 per month. It then follows that the number of deaths over 2 months is 4.

Generally, for any two points a and b where 0 1 a b < < < , then the number of deaths
between [ ] , a b is calculated as follows:

Page 88 of 285 Deeper Understanding: Exam M November 7, 2006 2006 Yufeng Guo
http:/ / www.guo.coursehost.com

Age x 1 x +
Time to die 0 a b 1 T
( ) Deaths during [ , ] = - Total deaths during [0,1] a b b a t e
Next, lets prove the shortcut. To see why ( )
x
f t q = , notice that under UDD:

( )
1 1 f t t < = for ( ) 0,1 t e -- This is the definition of UDD between integral ages

( )
( )
( )
( )
( ) 1 1
1
x
x
f t f t
f t t f t q
P t q
< = = = =
<
( )
0 0 0
t t t
t x x x x
q f t dt q dt q dt t q = = = =
} } }

1 1
t x x x
p q t q = =
To see why
( )
1 1
x x
t
t q
= , please note that

( )
( )
1
x
x
t x x
f t q
t
p t q
= =
( )
1 1 1
x
x x x
t q
t
t q q

= =
To see why
1 1 1
2
x x
m q
= , first we need to understand what
x
m means.

From Actuarial Mathematics 2
nd
Edition Section 3.5, we see that:

( )
1
0
1
0
1
x t
x x
x
x
x t
l x t dt
l l
L
l dt
m

+
+
+
+

= =
}
}
x
m is called the central death rate over the interval from x to 1 x +
Page 89 of 285 Deeper Understanding: Exam M November 7, 2006 2006 Yufeng Guo
http:/ / www.guo.coursehost.com

1
0
x x t
L l dt
+
=
}
x
L is the total expected number of years lived between age x and 1 x + by survivors of
the initial group of
0
l lives.

( )
1 1 1
0 0 0
0 0
t x t x x x t
L l dt l p dt l p dt
+
= = =
} } }

( )
1
1 1
2
0 0 0
1 1
2 2
x x
t x x
q q
p dt t q dt t t
(
= = =
(

} }

( )
1
1
2 0
0 0
1 1
2
x
x
q
l t q dt l l
| |
= =
|
\ .
}
Please also note that

( ) ( ) ( )
1 1 1
0 0 0
1 1 1 1
0 0 0 0
t x
x
t x t x t x
x t
x
x t
l x t dt p x t dt f t dt
q
l dt p dt p dt p dt
m

+
+
+ +
= = = =
} } }
} } } }

x
m is the weighted average force of mortality over ( ) 0,1 t e , with weights being
x t
l
+
(or
t x
p ).

Finally, we are ready to prove
1 1 1
2
x x
m q
= .
1
0
x
t x
x
q
p dt
m =
}
1
0
1
2
x
t x
q
p dt =
}
( )
1
0
1
0
1
1 1 1
2
,
2
1
2
x
x
x
x x
t x
x
x
q
f t dt
q
q
q q
p dt
m
m

= = = =

}
}
Page 90 of 285 Deeper Understanding: Exam M November 7, 2006 2006 Yufeng Guo
http:/ / www.guo.coursehost.com

We can also intuitively see why
1
2
x
x
x
q
q
m =

.
The number of deaths over ( ) 0,1 t e is
0
x
l q . The average population over ( ) 0,1 t e is:

( ) ( )
0 1 0 0 0 0
1 1 1
1
2 2 2
x x
l l l l l q l q
| |
+ = + =
|
\ .
.
As the result, the average force of mortality is:

( )
( )
0
0
# of total deaths over 0,1
1 average # of people alive over 0,1
1 1
2 2
x
x
x
x
l q
t q
q
t
l q
e
= =
e | |

|
\ .

Please also note that if we set
1
2
t = , then
( ) 1 1
2 2
1 1 1 1
2
x x x
x
t t
t
t q q
m

= =
(
= = =
(


UDD shortcut examples

Problem 1 (SOA May 2004 EA-1, #11)

1
960
x
l
+
=
975
x
L =
Assume a uniform distribution of deaths between x and 1 x + .
In what range is 1000*
x
m ?
(A) Less than 30.0
(B) 30.0 but less than 30.5
(C) 30.5 but less than 31.0
(D) 31.0 but less than 31.5
(E) 31.5 or more

Solution

We have a UDD between x and 1 x + . As a result,
1 1 1
2
x x
m q
= .
Page 91 of 285 Deeper Understanding: Exam M November 7, 2006 2006 Yufeng Guo
http:/ / www.guo.coursehost.com

So we need to find
x
q .
Under UDD,

0
1 975
2
x
x
q
L l
| |
= =
|
\ .

( )
1 0
1 960
x x
l l q
+
= =
1
975
2
, 3.03%
1 960
x
x
x
q
q
q

= =

1 1 1 1 1
, 1000 30.77
2 3.03% 2
x
x x
m
m q
= = =
The correct answer is C.

Problem 1 (SOA May 2004 EA-1, #29)

Selected items related to a mortality table over the age interval [98, 99]:

98.55
0.5980 = under the uniform death distribution
98.35
0.2486
y
q = under the uniform death distribution
98.15 y
Z q = under the constant force of mortality

In what range is Z ?
(A) Less than 0.246
(B) 0.246 but less than 0.254
(C) 0.254 but less than 0.262
(D) 0.262 but less than 0.270
(E) 0.270 or more

Solution

This problem is difficult. Unless you know the shortcuts for UDD, chances are that you
wont be able to solve this problem under exam conditions.

Page 92 of 285 Deeper Understanding: Exam M November 7, 2006 2006 Yufeng Guo
http:/ / www.guo.coursehost.com

To find
98.15 y
Z q = under the constant force of mortality, we need to know two things:
(1) y , and (2) the constant force of mortality over the age interval [98, 99].

98.55
0.5980 = ( )
98.55 98
0.55 0.5980 = =
Applying the formula
( )
1 1
x x
t
t q
= , we have

98
98
1 1
0.55, 0.45
0.5980
q
q
= =
Next, we need to find y by solving the equation
98.35
0.2486
y
q = . This equation is a bit
tricky to solve. Here we have someone aged 98.35 whose death rate over the time horizon
[ ] 0, y is 0.2486. To solve this equation, we use the definition of
98.35 y
q :
( ) ( )
98.35
death rate between age 98.35 and age 98.35
y
q y = +
( )
( )
# of people alive at age 98.35
1 0.2486
# of people alive at age 98.35
y +
= =
To quickly and accurately track down the population over the age interval [98, 99], we
first assume that there is only one person alive at age 98. So we start off with
0
1 l = at
0 t = (i.e. at age 98).

If you feel unsure about arbitrarily setting the number of people alive at 0 t = to one, you
can think that one actually represents one million people or one thousand people.

So at 0 t = (i.e. at age 98), we have only 1 person alive. Then at 0.35 t = (i.e. at age
98.35), the number of people alive is (under UDD):

( ) 1 1 1 0.35 1 0.35 0.45
t x x x
q t q q = = =
Similarly, at 0.35 t y = + (i.e. at age 98.35 y + ), the number of people alive is (under
UDD):

( ) ( ) ( ) 1 1 1 0.35 1 0.35 0.45
t x x x
q t q y q y = = + = +
This can be summarized in the following table:

Page 93 of 285 Deeper Understanding: Exam M November 7, 2006 2006 Yufeng Guo
http:/ / www.guo.coursehost.com

Number of people alive during ( ) 0,1 t e under UDD with
98
0.45 q =
Time 0 0.35 0.35 y + 1
Age 98 98.35 98.35 y + 99
# of people
alive
1
( ) 1 0.35 0.45 ( ) ( ) 1 0.35 0.45 y +
1 0.45
( )
( )
98.35
# of people alive at age 98.35
1
# of people alive at age 98.35
y
y
q
+
=
( ) ( )
( ) ( )
1 0.35 0.45
0.45
0.2486 1
1 0.35 0.45 1 0.35 0.45
y
y
+
= =


( ) 1 0.35 0.45 1
0.2486 0.2486 0.35 0.4654
0.45 0.45
y
(
| |
= = =
( |
\ .


To find
98.15 y
Z q = under the constant force of mortality, we need to know the constant
force of mortality over the age interval [98, 99]. We have:

98 98
1 q p e

= =
( )
98
1 0.45 , ln 1 0.45 ln 0.55 0.5978 p e

= = = = =
( ) ( ) 0.4654 0.5978 0.4654
0.4654 98.15
0.7573 p e e

= = =
98.15 98.15
1 1 0.7573 0.2427
y y
Z q p = = = =
The correct answer is A.

Problem 2 May 2000 #12

For a certain mortality table, you are given:

( ) 80.5 2.02% = , ( ) 81.5 4.08% = , ( ) 82.5 6.19% = .
Deaths are uniformly distributed between integral ages.

Calculate the probability that a person aged 80.5 will die within two years.
Page 94 of 285 Deeper Understanding: Exam M November 7, 2006 2006 Yufeng Guo
http:/ / www.guo.coursehost.com

Solution

UDD between [ ] , 1 x x +
( )
1 1
x x
t
t q
=
80
80
1 1
0.5 2%
2.02%
q
q
= =
81
1 1
0.5
4.08% q
=
81
4% q =
82
1 1
0.5
6.19% q
=
82
6% q =
Age 80 80.5 81 82 82.5
# of
people
alive
1
80
1 0.5q
99% =
80
1 98% q =
( )
81
98% 1 q
94.08% =
( )
8 2
94.08% 1 0. 5q
91.2576% =
2 80.5
# of people @ 82.5 91.2576%
1 1 7.82%
# of people @ 80.5 99%
t
q
t
=
= = =
=
Page 95 of 285 Deeper Understanding: Exam M November 7, 2006 2006 Yufeng Guo
http:/ / www.guo.coursehost.com

Chapter 6 The heart and soul of Actuarial
Mathematics

Before you bother to learn life insurance and life annuities and related symbols such as
x
A and
x
a , you need to understand the fundamental law of life insurance. Once you
understand this fundamental law, everything else falls into place. If you dont understand
this fundamental law, memorizing all the symbols and formulas will make your head
ache.

Fundamental law of life insurance

Life insurance is self-supporting; healthy people subsidize the unhealthy people.

Example 1

You are a pricing actuary of a life insurance company. Your company wants to sell a 3-
year term life insurance policy. If a policyholder dies within the first 3 years after the
insurance policy is sold, your company pays his family $1,000. If he is still alive and
kicking after 3 years, he gets nothing and the insurance contract simply terminates.
Additional information given to you:
(1) Exactly 10 people buy the 3 year term insurance policy. (If you have trouble
accepting that only 10 people buy the insurance policy, you can think that 1 person
actually means 1 million people so you have 10 million buyers. This doesnt change
the answer.)
(2) Of the 10 policyholders, 1 person dies in the first year after the policy is sold, 3
people die in the second year, and 4 people die in the third year.
(3) Your company operates on a non-profit basis (i.e. your company makes zero profit).
(4) Premium is paid at the beginning of each year during the first three years. Death
benefits are paid at the end of the year in which a death occurs.
(5) The interest rate is zero.
You are asked to:
(1) Calculate the premium each policyholder needs to pay.
(2) Analyze the net gain by each policyholder.
Page 96 of 285 Deeper Understanding: Exam M November 7, 2006 2006 Yufeng Guo
http:/ / www.guo.coursehost.com

Solution

Because your company plans to make zero profit, your pricing equation should be:

Total future premiums = Total death benefits

Let P represent the premium each policyholder needs to pay.

Time T 0 1 2 3 Total
# of policyholders alive (thus paying premiums) 10 9 6 2
Premiums collected 10P 9P 6P 25P
# of deaths during [T-1, T ] (policy year T) 1 3 4
Death benefits paid 1,000 3,000 4,000 8,000
Total dollar amount of future premiums = Total dollar amount of future death benefits

25 $8, 000 $320 P P = =
Net gain by Amount Subtotal
the policyholder who
died in the 1
st
policy
year
1, 000 P
1, 000 320 680 = =
680
each of the 3
policyholders who
died in the 2
nd
policy
year
1, 000 2P
1, 000 2(320)
360
=
=
3(360)=1,080
each of the 4
policyholders who
died in the 3
rd
policy
year
1, 000 3 1000 3(320)
40
P =
=
4(40)=160
each of the 2
policyholders alive
after t=3
3 3(320) 960 P = = 2(-960)= -1,920
Grand Total 680 + 1,080 + 160 + (-1,920) =0
The total net gain earned by the 10 policyholders is zero. In other words, the total net
gain earned by those who died during the first 3 policy years was funded exactly by the 2
policyholders who didnt die.

This should make sense. Money doesnt come out of thin air. If a life insurance company
earns zero-profit and some policyholders profited from the life insurance, then other
Page 97 of 285 Deeper Understanding: Exam M November 7, 2006 2006 Yufeng Guo
http:/ / www.guo.coursehost.com

policyholders must have suffered some financial loss. Life insurance (and other insurance
too) is a zero sum game; some gain and some lose.

You might wonder why the life insurance mechanism continues to flourish. Arent those
surviving policyholders mad at their losses? They have paid thousands of dollars of
premium but have collected nothing in return (like the two surviving policyholders in our
simple example). Several reasons:

(1) When people buy life insurance (or other insurance) policies, nobody knows that
he will die early (hence get a good deal) or die late (thus lose money). Many buy
life insurance for peace of mind.

(2) The families of those who died early are glad that they got a good deal; those who
survived and wasted their premiums are glad that they didnt die.

(3) You have to buy a life insurance policy in some circumstances. For example,
some types of loans require you to purchase a life insurance policy. This policy
will pay the remaining balance of your loan if you die before paying off your
loan.

(4) Some buy life insurance policies mainly for tax benefits. Some grandparents buy
life insurance policies and name their grandchildren as beneficiaries. Should
grandparents die, the death benefit goes to the grandchildren tax-free.

Example 2

You are a pricing actuary of a life insurance company. Your company wants to sell a 3-
year term life insurance policy. If a policyholder dies within the first 3 years after the
insurance policy is sold, your company pays him $1,000. If he is still alive and kicking
after 3 years, he gets nothing and the insurance contract simply terminates.
Additional information given to you:
(1) Exactly 10 people buy the 3-year term insurance policy.
(2) Of the 10 policyholders, 1 person dies in the first year after the policy is sold, 3
people die in the second year, and 4 people die in the third year.
(3) Your company operates on a non-profit basis (i.e. your company makes zero profit).
(4) Premium is paid at the beginning of each year during the first 3 years. Death
benefits are paid at the end of the year when a death occurs.
(5) The interest rate is 5%.
Page 98 of 285 Deeper Understanding: Exam M November 7, 2006 2006 Yufeng Guo
http:/ / www.guo.coursehost.com

You are asked to:

(1) Calculate the premium each policyholder needs to pay.
(2) Analyze the net gain by each policyholder.

Solution

Because we now need to consider the time value of money, we have to change our
pricing equation to:

Total present value of future premiums = Total present value of future death benefits

Time T 0 1 2 3
# of deaths during [T-1,T] (policy year T) 1 3 4
# of policyholders alive (thus paying premiums) 10 9 6
Premiums collected 10P 9P 6P
Present Value of Premiums collected 10P 9Pv 6Pv
2
Death benefits paid 1,000 3,000 4,000
Present value of death benefits paid 1,000v 3,000 v
2
4,000 v
3
2 2 3
(10 9 6 ) 1000( 3 4 ) P v v v v v + + = + + , where
1 1
1 1.05
v
i
= =
+
2 3
2
3 4 7.1288
1000 1000 $296.87
10 9 6 24.0136
v v v
P
v v
+ +
= = =
+ +

Present value of
the net gain by
Amount Sub Total
the policyholder
who died in the
1
st
policy year
1, 000v P
1
1, 000(1.05 ) 296.87 655.51

= =
655.51
each of the 3
policyholders
who died in the
2
nd
policy year
2
1, 000 (1 ) v P v +
2 1
1, 000(1.05 ) 296.87(1 1.05 )
327.43

= +
=
3(327.43)=982.29
each of the 4
policyholders
who died in the
3
rd
policy year
3 2
1, 000 (1 ) v P v v + +
3 1 2
1, 000(1.05 ) 296.87(1 1.05 1.05 )
14.96

= + +
=
4(14.96)=59.84
each of the 2
policyholders
who survived the
policy
2
(1 ) P v v + +
1 2
296.87(1 1.05 1.05 )
848.87

= + +
=
2(-848.87)= -1,697.74
Grand Total 655.51+ 982.29+
59.84 + (-1,697.74)
= - 0.1
Page 99 of 285 Deeper Understanding: Exam M November 7, 2006 2006 Yufeng Guo
http:/ / www.guo.coursehost.com

We didnt get zero due to rounding. Once again, the total net gain by the 8 policyholders
was funded by the 2 policyholders who survived the 3 year-term policy.

Mathematically, this should make sense. The total PV of net gain by the 10 policyholders
is:

[ ]
2 3 2
net gain by the one
net gain by the 3 policyholders net gain by the 4 polic
policyholder who
who died in the 2nd year
died in the 1st year
1, 000 3 1, 000 (1 ) 4 1, 000 (1 ) v P v P v v P v v ( ( + + + + +


2
yholders net gain by the 2
who died in the 3rd year policyholders who
survived 3 years
2 (1 ) P v v ( + + +


2 3 2
1000( 3 4 ) (10 9 6 ) 0 v v v P v v = + + + + =
2 3 2
1000( 3 4 ) (10 9 6 ) v v v P v v + + + + is zero because this is how we calculated premium
P in the first place.

Example 3

You are a pricing actuary of a life insurance company. Your company wants to sell a 3-
year term life insurance policy. If a policyholder dies within the first 3 years after the
insurance policy is sold, your company pays him $1,000. If a buyer doesnt die after the
end of the first 3 years, he gets nothing and the insurance contract simply terminates.
You are asked to calculate the premium each policyholder needs to pay.
Additional information given to you:
(1) Exactly 10 people buy the 3-year term insurance policy.
(2) Of the 10 policyholders, 1 person dies in the first year after the policy is sold, 2
people die in the second year, and 4 people die in the third year.
(3) Your company operates on a non-profit basis (i.e. your company makes zero profit)
.
(4) Premium is paid at the beginning of each year. Death benefits are paid at the end of
the year in which a death occurs.
(5) The interest rate is 5%.
Solution

The difference between Example 2 and Example 3 is that the number of deaths for
Example 2 in Policy Year 2 is three, while the number of deaths for Example 3 in Policy
Year 2 is two.

Page 100 of 285 Deeper Understanding: Exam M November 7, 2006 2006 Yufeng Guo
http:/ / www.guo.coursehost.com

The question asks us to calculate the premium only.

Total present value of premiums collected = Total present value of death benefits paid out
Time T 0 1 2 3
# of deaths during [T-1,T] (policy year T) 1 2 4
# of policyholders alive (thus paying premiums) 10 9 7
Premiums collected 10P 9P 7P
Present Value of Premiums collected 10P 9Pv 7Pv
2
Death benefits paid 1,000 2,000 4,000
Present value of death benefits paid 1,000v 2,000 v
2
4,000 v
3
2 2 3
(10 9 7 ) 1000( 2 4 ) P v v v v v + + = + + , where
1 1
1 1.05
v
i
= =
+
2 3
2
2 4 6.2218
1000 1000 $249.66 $296.87
10 9 7 24.9206
v v v
P
v v
+ +
= = = <
+ +

The premium in Example 3 is less than the premium in Example 2. This should make
sense. In Example 3, the number of deaths is fewer and more people pay premiums to
cover less death benefits, producing a lower premium.

By now we see that premium rate (the cost of an insurance policy) varies by the death
rate of the policyholders. More deaths, higher premiums.

When actuaries price life insurance policies, theres no way to know precisely how many
policyholders will die from year to year. As a result, no one knows precisely the true cost
of a life insurance policy in advance.

This is quite different from pricing manufactured goods, where manufacturers know, with
good precision, the true cost of a product (raw materials, labor, overhead, etc.) in
advance.

Moral of these examples:

The true cost of a life insurance policy CANNOT be known in advance.
Page 101 of 285 Deeper Understanding: Exam M November 7, 2006 2006 Yufeng Guo
http:/ / www.guo.coursehost.com

Equivalence principle

Because as actuaries, we really dont know precisely how much premium we will collect
and how much death benefits we will pay out, we have to tweak our intuitive pricing
equation from

Total present value of future premiums = Total present value of future death benefits

To

Total expected present value of future premiums
= Total expected present value of future death benefit
When actuaries price life insurance products, they often spend a lot of time (weeks and
months) estimating the year-by-year death rate of the potential policyholders, using their
companys own experience and the mortality rates of the general population. Then they
calculate the premium rates based on the estimated mortality using the equivalence
principle:

Total expected present value of future premiums
= Total expected present value of future death benefits

In the real world, the equivalence principle is applied to each risk group (such as male
age 35 non-smokers) to calculate the premium rate for that risk group. Clearly, we cant
charge a 60-year old policyholder (more likely to die) and a 30-year old policyholder
(less likely to die) the same rates. Often we divide the policyholders into many risk
groups based on factors such as age, gender, smoking or non-smoking, and some other
important underwriting criteria. The key point here is that we cant blindly apply the
equivalence principle to different risk groups. We need to apply the equivalence principle
only to the same risk group.

Example 4

You are a pricing actuary of a life insurance company. Your company wants to sell a 3-
year term life insurance policy. If a policyholder dies within the first three years after the
insurance policy is sold, your company pays him $1,000. If a buyer doesnt die after three
years, he gets nothing and the insurance contract simply terminates.
You are asked to calculate the premium rate for a 35-year old non-smoking male.
Additional information given to you:
(1) The mortality for 35-year old non-smoking males is assumed to follow the Illustrated
Life Table.
Page 102 of 285 Deeper Understanding: Exam M November 7, 2006 2006 Yufeng Guo
http:/ / www.guo.coursehost.com

(2) Premium is paid at the beginning of each year during the three years. Death benefits
are paid at the end of the year in which a death occurs.
(3) The interest rate is 5%.
Solution

If you look at the Illustrated Life Table, youll find that:

Age # of people alive
35 9,420,657
36 9,401,688
37 9,381,566
38 9,360,184
So we assume that the population of 35-year old non-smoking males follows the above
table.

Time T 0 1 2 3
# of policyholders
EXPECTED alive (thus
paying premiums)
9,420,657 9,401,688 9,381,566 9,360,184
# of deaths EXPECTED
during [T-1,T] (policy
year T)
9,420,657-9,401,688
=18,969
9,401,688-9,381,566
=20,122
9,381,566-9,360,184
=21,382
EXPECTED Premium 9,420,657P 9,401,688P 9,381,566P
EXPECTED Present
Value of Premium
9,420,657P 9,401,688P v 9,381,566P v
2
EXPECTED Death
benefits
1,000(18,969) 1,000(20,122) 1,000(21,382)
EXPECTED Present
value of death benefits
1,000(18,969)v 1,000(20,122) v
2
1,000(21,382) v
3
Total expected present value of future premiums:

2
9, 420, 657 9, 401, 688 9, 381, 566 P Pv Pv + +
2
(9, 420, 657 9, 401, 688 9, 381, 566 ) P v v = + +
Total expected present value of future death benefit:

2 3
1, 000(18, 969) 1, 000(20,122) 1, 000(21, 382) v v v + +
2 3
1, 000(18, 969 20,122 21, 382 ) v v v = + +
Page 103 of 285 Deeper Understanding: Exam M November 7, 2006 2006 Yufeng Guo
http:/ / www.guo.coursehost.com

2 2 3
(9, 420, 657 9, 401, 688 9, 381, 566 ) 1, 000(18, 969 20,122 21, 382 ) P v v v v v + + = + +
2 3
2
18, 969 20,122 21, 382
1, 000
9, 420, 657 9, 401, 688 9, 381, 566
v v v
P
v v
+ +
=
+ +

54,787.5370
1000 $2.04
26,884,002.4875
= =
This premium rate of $2.04 is more realistic (because we used a more realistic mortality
table). If you try to get a term insurance quote from the internet for a male age 35 (either
smoking or non-smoking), you can easily get a quote ranging from $200 to $300 annual
premium per $100,000 death benefit for 10-year term insurance (i.e. $2 to $3 per $1,000
death benefit).

The premium rates calculated in Example 1 ($320), Example 2 ($296.87), and Example 3
($249.66) are far too high for term insurance (term insurance is the cheapest insurance).
If you really sell this product, no one is going to buy it because its too expensive. The
premium rates are too high because we used a fake mortality table (whose mortality rate
is far higher than a more realistic mortality table).

n-year Term Insurance Model
n-year term fully discrete

Next, lets extend the 3-year term insurance model to an n-year term insurance.

n-year term insurance where the death benefit is paid at the end of the year of death
and premiums are paid at the beginning of the year

Your company asks you to set the price for an n-year term insurance. In an n-year term
insurance issued to a policyholder age ( ) x , if ( ) x dies in the n years after the policy is
sold, his family gets paid $1 at the end of the year of death. If ( ) x is alive and kicking
after n years, he gets none.
Premiums are paid at the beginning of each year, until the policyholder is dead or n
years have elapsed, whichever occurs first.
The interest rate is i .
Calculate the benefit premiumP . Benefit premium is the premium calculated using the
equivalence principle.
Page 104 of 285 Deeper Understanding: Exam M November 7, 2006 2006 Yufeng Guo
http:/ / www.guo.coursehost.com

Solution

Age x 1 x + 2 x + 1 x n + x n +
Time t 0 1 2 1 n n
# of people
alive
x
l
1 x
l
+ 2 x
l
+

1 x n
l
+ x n
l
+
Premium
collected
x
Pl
1 x
Pl
+ 2 x
Pl
+

1 x n
Pl
+
# of deaths
1 x x
l l
+

1 2 x x
l l
+ +

2 1 x n x n
l l
+ +

1 x n x n
l l
+ +

Expected PV of future premiums:



( )
2 1 2 1
1 2 1 1 2 1
... ...
n n
x x x x n x x x x n
Pl Pl v Pl v Pl v P l l v l v l v

+ + + + + +
+ + + + = + + + +
Expected PV of future death benefits:

( ) ( ) ( )
2
1 1 2 1
...
n
x x x x x n x n
l l v l l v l l v
+ + + + +
+ + +
We apply the equivalence principle:

Expected PV of future premiums = Expected PV of future benefits

( ) ( ) ( ) ( )
2 1 2
1 2 1 1 1 2 1
... ...
n n
x x x x n x x x x x n x n
P l l v l v l v l l v l l v l l v

+ + + + + + + +
+ + + + = + + +
2 1 2 1 2 1 1 1 2 1
1 ... ...
n n x x x n x x x x x n x n
x x x x x x
l l l l l l l l l
P v v v v v v
l l l l l l
+ + + + + + + +
| |
+ + + + = + + +
|
\ .

However,

1 2 1
2 1
, , ...,
x x x n
x x n x
x x x
l l l
p p p
l l l
+ + +

= = =
1 1 2 1
1 1 1 1 1 1
, , ...,
x x x x x n x n
x x x x x n x x n n
x x x
l l l l l l
q q p q q p q
l l l
+ + + + +
+

= = = = =
( )
2 1 2
2 1 1 1 1 1
1 ... ...
n n
x x n x x x x n
P p v p v p v q v q v q v


+ + + + = + + +
Next, we define two new concepts:
Page 105 of 285 Deeper Understanding: Exam M November 7, 2006 2006 Yufeng Guo
http:/ / www.guo.coursehost.com

1
2 1
2 1
:
0
1 ...
n
n k
x x n x k x
x n
k
a p v p v p v p v

=
= + + + + =


1
1 2 1
1 1 1 1 :
0
...
n
n k
x x x k x x k n x n
k
A q v q v q v v p q

+
+
=
= + + + =

: x n
a is the present value of n cash flows of $1 occurring at 0 t = , 1, 2, , and 1 n ,
with each cash flow contingent on the survival of ( ) x . If ( ) x is still alive at 0 t = , 1, 2,
, and 1 n , hell pay $1; if hes dead, we wont collect any money from him. This
pattern of ( ) x paying premiums for n years as long as hes alive is called an n year
contemporary annuity due.
: x n
a is the present value of this n year contemporary annuity
due where the premiums are paid at the beginning of the year.

Age x 1 x + 2 x + 1 x n + x n +
Time t 0 1 2 1 n n
Cash flow $1 1 1 1
Probability of getting 1
x
p
2 x
p
1 n x
p

this cash flow


1
2 1
2 1
:
0
1 ...
n
n k
x x n x k x
x n
k
a p v p v p v p v

=
= + + + + =


In Exam FM you learned that
: x n
a is similar to
n
a . Both represent the present value of
n evenly spaced cash flow of $1 occurring at 0 t = , 1, 2, , and 1 n . The only
difference is that in
n
a we are 100% certain to collect each cash flow of $1, whereas in
: x n
a the cash flows depend on the survival of ( ) x .
Next, lets look at the symbol
1
: x n
A . This is the present value of n cash flows of $1 each
occurring at 1, 2, 3 t = , and n , with each cash flow contingent on the death of ( ) x . The
Page 106 of 285 Deeper Understanding: Exam M November 7, 2006 2006 Yufeng Guo
http:/ / www.guo.coursehost.com

insurer will pay $1 at 1 t = if ( ) x dies during 0 1 t < s . The insurer pays $1 at 2 t = if
( ) x dies during 1 2 t < s , and so on. The insurer pays the final cash flow of $1 at t n = to
those who die during 1 n t n < s . To those who are still alive and kicking at t n > , the
insurer pays zero.

Age x 1 x + 2 x + 1 x n + x n +
Time t 0 1 2 1 n n
Death benefits per death $1 1 1 1
Probability of death
x
q
1 1 x
q
2 1 x n
q
1 1 x n
q

1
1 2 1
1 1 1 1 :
0
...
n
n k
x x x k x x k n x n
k
A q v q v q v v p q

+
+
=
= + + + =

The equivalence principle requires that

( )
1
: : x n x n
P a A =
1
:
:
x n
x n
A
P
a
=

To clearly indicate that P is the benefit premium for an n-yr term insurance (as opposed
to the premium for another form of life insurance policy such as a whole life policy), a
special symbol was invented. This symbol is
1
: x n
P . So if we really want to be a sticklers
about symbols, the equivalence principle is:

( )
1 1
: : : x n x n x n
P a A =
1
: 1
:
:
x n
x n
x n
A
P
a
=

Im a minimalist on fancy actuarial symbols and terms. Yes, I need to understand


symbols and terms used in the exam. However, when I solve a problem, I simply throw
symbols away and focus on the essence of the calculation.

In the exam, I use P to represent the premium, no matter whether the insurance is whole
life or n-yr term, no matter whether the death benefit is paid at the time of death or at the
end of the year.
Page 107 of 285 Deeper Understanding: Exam M November 7, 2006 2006 Yufeng Guo
http:/ / www.guo.coursehost.com

In addition, I typically dont write
1
: 1
:
:
x n
x n
x n
A
P
a
=

. Ill calculate the PV of future benefits


and the PV of annuity (i.e. when the premiums are $1 each). Then,

future death benefits
future premiums if premiums are always 1$
PV
P
PV
=
I recommend that you do the same: Understand fancy symbols, but throw them away to
simplify your calculations during the exam. Focus on the essence of the calculation.

n-year Term Fully Continuous

n-year term insurance where the death benefit is paid at the moment of death and
premiums are paid continuously.

Your company asks you to set the price for an n-year term insurance. In this n-year term
insurance issued to a policyholder age ( ) x , if ( ) x dies at time t where 0 t n < s , his
family gets paid
t
b dollars at t . However, if hes alive and kicking after n years, he gets
nothing.
Premiums are paid continuously at the constant rate throughout each year, until the
policyholder is dead or n years have elapsed, whichever occurs first.
The force of interest is o .
Calculate the benefit premium P .
Solution

To understand how premiums can be paid continuously, imagine that the insurance
company collects premiums on a daily or weekly basis. Though premiums are rarely paid
daily or weekly, paying premiums continuously is a nice simplification.

Since the death benefit is paid immediately upon the death of the insured, this time we
have to use integration.

Consider a small time interval of [ ] , t t dt + . The probability for ( ) x to die during this
interval is ( ) ( ) P t T t dt f t dt < s + = .
Page 108 of 285 Deeper Understanding: Exam M November 7, 2006 2006 Yufeng Guo
http:/ / www.guo.coursehost.com

Age x t + x t dt + +
Time t t t dt +

Death benefit per death over [ ] , t t dt +


t
b
Probability of death over [ ] , t t dt + ( ) f t dt
Present value of the death benefits paid over [ ] , t t dt + ( )
t
t
e b f t dt
o
Present value of the death benefits paid over [ ] 0, t n e ( )
0
n
t
t
e b f t dt
o
=
}
Because ( ) ( )
t x x
f t p t = , the PV of the death benefits paid over [ ] 0, t n e can be
rewritten as:

( ) ( )
0 0
n n
t t
t t x t x
e b f t dt e b t p dt
o o


=
} }

Next, lets calculate the present value of the premiums collected. ( ) x pays premiums
continuously at the constant rate of $1 per year during the first n years as long as hes
alive. Once again, we consider a small time interval [ ] , t t dt + . Since $1 premium is paid
continuously and dt time period has elapsed during [ ] , t t dt + , then the total premium
dollars collected during [ ] , t t dt + is ( ) 1 dt dt = . Since dt is so small, we assume the
probability for ( ) x to be alive during [ ] , t t dt + is still
t x
p .
Page 109 of 285 Deeper Understanding: Exam M November 7, 2006 2006 Yufeng Guo
http:/ / www.guo.coursehost.com

Age x t + x t dt + +
Time t t t dt +

Premiums collected over [ ] , t t dt + if ( ) x is alive dt


Probability for ( ) x to be alive over [ ] , t t dt +
t x
p
Present value of the premiums collected over [ ] , t t dt +
t
t x
e p dt
o
Present value of the premiums collected paid over [ ] 0, t n e
0
n
t
t x
e p dt
o
=
}
The actuarial symbol for
0
n
t
t x
e p dt
o
}
is : x n a . : x n a represents the present value of a
continuous stream of cash flow during [ ] 0, t n e paid at a constant rate of $1 per year as
long as ( ) x is still alive.

The pattern of ( ) x paying premiums for n years as long as hes alive is still an n -year
contemporary annuity due. However, this time, the payments are made continuously at a
constant rate of $1 per year.

Assume ( ) x pays premium P . Applying the equivalence principle, we have:

PVFP = PVFB

( ) :
0 0
n n
t t
x n
t x t x t x
P e p dt P a e b t p dt
o o


= =
} }

( ) ( )
0 0
:
0
n n
t t
t x t x t x t x
n
x n
t
t x
e b t p dt e b t p dt
P
a
e p dt
o o
o

= =
} }
}
Page 110 of 285 Deeper Understanding: Exam M November 7, 2006 2006 Yufeng Guo
http:/ / www.guo.coursehost.com

If $1
t
b = , then ( ) ( )
0 0
n n
t t
t x t x x t x
e b t p dt e t p dt
o o


=
} }
. A symbol was invented to
represent this value:

( )
1
:
0
n
t
x n
x t x
A e t p dt
o

=
}
1
: x n A represents the present value of an n -year term insurance issued to ( ) x where $1
death benefit is paid at the time of death.

If $1
t
b = , then the benefit premium becomes:

( ) 1
:
0
: :
n
t
x t x
x n
x n x n
e t p dt
A
P
a a
o

= =
}
Of course, theres a symbol designed for the P calculated above:

( )
1
1
:
:
:
x n
x n
x n
A
P A
a
=
Make sure you understand the n -year term insurance (fully discrete or fully continuous)
model. If you understand these two models, you wont have trouble understanding how to
calculate premiums for other types of insurance (such as whole life, either fully discrete
or fully continuous).

Page 111 of 285 Deeper Understanding: Exam M November 7, 2006 2006 Yufeng Guo
http:/ / www.guo.coursehost.com

Chapter 7 Fundamentals of life insurance and
annuity

After you understand how to calculate premiums for an n-year term insurance and other
types of insurance, you really have understood the core theories of Actuarial
Mathematics. Next, you need to learn some definitions and symbols.

TIP : Since this chapter contains many symbols, use these two general strategies for
memorizing the symbols:
Be patient. Mastering these symbols takes some time.
Start by learning the common sense behind life insurance, not from learning symbols.
Once you have developed a basic knowledge of life insurance, learning the symbols
becomes much easier.
Memorizing definitions
Term vs. whole life

Insurance agents like to say, Term insurance is like renting a house; whole life insurance
is like owning a house. Term insurance is cheaper but the ownership is temporary. Whole
life is more expensive but you are building equity.

Remember the above analogy. An n-year term insurance provides death benefits if a
policyholder dies within n-years after the policy is purchased. After n-years, the contract
expires. If the policyholder is still alive after n-years, he gets nothing. Term insurance is
pure insurance. Your premium just covers the death benefit for a finite period of time.

A whole life policy, on the other hand, is part insurance plus part savings. A whole life
insurance policy provides death benefits whenever the policyholder dies, providing
protection during the entire life of the policyholder. Consequently, whole life insurance is
more expensive than term insurance.

Typically in a whole life policy, you pay the same amount of premium each year. If you
buy the policy as a young person, you will pay this same amount of premium even
though as you age, you are more likely to die and the cost of the insurance goes up. This
works because you overpay premiums in the early years and underpay premiums in the
latter years. In the early years, the excess premiums above and beyond the true cost of the
insurance are put into a savings account to earn interest. Then when you become older,
Page 112 of 285 Deeper Understanding: Exam M November 7, 2006 2006 Yufeng Guo
http:/ / www.guo.coursehost.com

the insurance company takes out money from those savings to subsidize your insurance,
because you underpay premiums in later years.

Should you buy term or a whole life policy? This is not on the syllabus, but its good to
know. There are two philosophies: (1) Buy whole life for long term protection, (2) Buy
term and invest the difference. The

first philosophy says that as a breadwinner, you
should buy a whole life policy now to protect your family. Dont wait till you are older to
buy a whole life policy; if bought when you are older, the premiums are very high.

The second philosophy says that you should just buy 100% pure insurance and do the
savings part yourself. Just buy cheap term insurance and invest the difference between a
term policy premium and a whole life policy premium (the saved premiums) somewhere
to earn interest. Why overpay premiums in early years and earn paltry interest at an
insurance company, when you can earn a higher return in the stock market, for example,
with your money?

Which school of thought is right? Well, both have merits. If you are disciplined and can
resist the temptation of spending the saved premiums, go buy term insurance and invest
the difference somewhere on your own. If on the other hand, you are going to spend the
saved premiums and buy a fancy car or take an expensive vacation, youre better off
locking your money in a whole life policy.

Endowment

An endowment is the opposite of a typical life insurance. In typical life insurance, a
breadwinner buys a life insurance policy in case he dies too soon. If he dies, his family
gets the death benefits.

An endowment, on the other hand, protects the policyholder if he lives too long and
outlives his resources. An endowment is more like an investment where you save for a
rainy day (rainy days meaning that you live too long).

An n-year pure endowment insurance provides payment to the policyholder if he is still
alive after n years. If he dies within n years, he gets nothing. People rarely purchase n-
year pure endowment insurance today because the risk of forfeiting all paid premiums if
the policyholder dies within n years is too great.

In contrast, n-year endowment insurance provides death benefits to a policyholder if he
dies within n years; if he is still alive after n years, hell receive an amount equal to the
death benefit. N-year endowment insurance is a combination of insurance and
investment.

Among many insurance policies, endowment insurance has the highest premiums
because it protects the policyholder from outliving his resources.

Page 113 of 285 Deeper Understanding: Exam M November 7, 2006 2006 Yufeng Guo
http:/ / www.guo.coursehost.com

Deferred Life Insurance

Imagine this dilemma. Today you are healthy (young or old) and can buy insurance at
lower rates, but you dont need life insurance. However, you may need insurance several
years later, at which time your health may be poor and you would have to pay hefty
premiums. So youre in a tight spot. Now you can buy insurance cheap, but you dont
need it. But if you wait, the price of the insurance may go up.

To solve your dilemma, insurance companies invented an m-year deferred insurance
policy. Under this policy, during the first m years, you prepay premiums for a future
insurance contract, which becomes effective m years later. If you die during the
prepayment period of m years, you get nothing. This shouldnt bother you because you
dont need the insurance during the first m-years. If you are still alive after m-years, you
have an insurance policy that has been pre-paid (meaning you dont need to pay any more
premiums after the pre-payment period is over).

For m-year deferred insurance, a policyholder generally pays premiums only during the
first m years. However, insurance companies can certainly design different m-year
deferred insurance policies and let a policyholder pay premiums before or after the
deferred period.

Memorizing symbols

A vs. A. Both mean the expected value of an insurance policy at issue where the
death benefit is $1. If the death benefit is paid immediately upon the death of the insured,
use A. If the death benefit is paid at the end of the year when the policyholder dies,
use A. This is similar to the annuity symbols in FM, where a bar means continuous cash
flow and no bar means discrete cash flows.

1
: x n
,
1
: x n
, and
: x n
. The best way to memorize these symbols is to keep using them
until familiarity breeds memory. In these 3 symbols, x means the age of the insured
when he first bought the insurance policy and n is the term of the insurance policy.

Page 114 of 285 Deeper Understanding: Exam M November 7, 2006 2006 Yufeng Guo
http:/ / www.guo.coursehost.com

Type of
insurance
Symbol Meaning How to memorize
n year term
1
: x n
If the insured ( ) x dies in n
years, he gets paid $1. If he
is still alive after n years, he
gets nothing.
1=expire.

1
x
means ( ) x expires
(i.e. die).

Because 1 is before n ,
so ( ) x must expire
(i.e. die) in n years in
order to get paid.
n-year pure
endowment

1
: x n
If the insured ( ) x is still
alive after n years he gets
$1. If he dies in n years, he
gets nothing. This is just the
reverse of
1
: x n
.
1
n
means that n years
must expire (i.e.
elapse) for ( ) x to get
any benefits.
n-year
endowment

: x n
1 1
: : : x n x n x n
= +
If the insured( ) x dies in the
next n years, his family gets
$1. If ( ) x is still alive and
breathing after n years, his
family still gets $1 at time n.

So an n-yr endowment is
really an n-yr term insurance
plus a pure n-yr endowment.

Carve this equation into your
mind so you wont forget it.
In the exam, its very
easy to forget that an
n-yr endowment has a
pure endowment at the
end of n years. To not
forget, use the phrase
No one buys pure
endowment insurance
(because such
insurance doesnt
make sense to most
people) If you dont
see the number 1 in the
symbol
: x n
, add a pure
endowment at the end
of n years.
Whole life x
If the insured ( ) x dies any
time, he gets paid $1. So

1
: : x x
x
+ +
= =
Whole life is an infinite term
insurance or an infinite
endowment. The value of the
pure endowment insurance
in a whole life insurance

Page 115 of 285 Deeper Understanding: Exam M November 7, 2006 2006 Yufeng Guo
http:/ / www.guo.coursehost.com

policy is zero because no
person lives beyond +.
m-yr deferred n-
yr term insurance
m n If the insured ( ) x dies in m
years his family gets
nothing.

If he dies after m years and
before m n + years, his
family gets $1.

If he is still alive and
breathing after m n + , he
gets nothing.
You shouldnt have
trouble memorizing
this symbol. Its also
used in
x m n
p and
x m n
q
Life insurance policies with level death benefits are the most common life insurance
policies sold in the real world. Level benefits insurance are easy for insurance agents to
explain and for potential customers to understand.

Besides these, there are insurance policies that pay increasing or decreasing death
benefits. For example, VUL (variable universal life) provides variable or changing death
benefits (hence the name variable). VUL is essentially insurance plus investment funds.
A VUL policyholder invests his premiums in several investment funds of his choice
(such as stocks and bonds). If the investment experience is good, these funds generate
interest and increase the death benefit. If the investment experience is poor, the fund
value drops and so does the death benefit.

Another example of insurance that provides variable death benefit is mortgage insurance
or student loan insurance. If you borrow money from a bank to buy a house, if you
borrow money from the U.S. government to go to college, your lending bank or the U.S.
government may require you to buy a decreasing life insurance policy. The death benefit
of mortgage insurance or student loan insurance is the remaining debt you owe.

Typically, your debt is paid off over many years (such as 20 years) by monthly payments.
As you pay your monthly installments month after month, the outstanding balance of
your debt decreases. A decreasing life insurance provides decreasing death benefits that
exactly offset your outstanding loan balance. So if you die, the lending bank or the U.S.
government will get the death benefit, which is equal to what you owe them at your time
of death.

Now you know some of the commonly-used variable insurance polices in the real world.
Next, you want to memorize the corresponding symbols. These symbols are self-
explanatory.

Page 116 of 285 Deeper Understanding: Exam M November 7, 2006 2006 Yufeng Guo
http:/ / www.guo.coursehost.com

n-yr increasing term insurance. If you die in Yr 1, your family gets $1; if you die in
Yr 2, your family gets $2, etc.. The expected value at issue of this increasing term
insurance is ( )
1
: x n
I A
n-yr decreasing term insurance. If you die in Yr 1, your family gets $n; if you die in
Yr 2, your family gets $(n-1). If you die in Yr n, your family gets $1. The expected
value at issue of this increasing term insurance is ( )
1
: x n
DA
If death benefits are paid at the moment of death, then the expected value at issue of an n-
yr increasing term insurance is
( )
1
: x n
I A ; if death benefits are paid at the end of the year
of death, the expected value at issue of an n-yr decreasing term insurance is
( )
1
: x n
DA
Exercises
#1 John is 35 years old. He bought a life policy. If John dies any time, the insurance
company will immediately pay Johns family $1. Whats the expected value of this
insurance policy?
Answer: This is whole life insurance where the death benefit is paid immediately upon
death. The expected value of this policy at issue is 35 A . Of course, you can also write
1
35: A + or 35: A +
#2 John is 35 years old. He bought a life policy. If John dies any time, the insurance
company will pay Johns family $1 at the end of the year when John dies. The insurance
company pays $1 at the end of the year because it needs time to handle the death claims
(e.g. to make sure John is really dead, to cut a check for Johns family, etc). Whats the
expected value of this insurance policy?
Answer: This is a whole life policy where the death benefit is paid at the end of the year.
The expected value of this policy at issue is
35
A . Of course, you can also write
1
35:
A
+
or
35:
A
+
#3 John is 35 years old. He bought a life policy. If John dies any time in the next 10
years, his family gets nothing. If, however, John is still alive and kicking after 10 years,
he gets $1. Whats the expected value of this insurance policy?
Answer: This is a pure endowment policy. The expected value at issue is
1
35:10
A . You can
also write
10 35
E . However, you cant write
1
35:10 A . Theres no such symbol as
1
: x n A . The
folks who invented actuarial symbols didnt think it necessary to invent
1
: x n A . So the pure
endowment is either
1
: x n
A or
n x
E , but not
1
: x n A .
Page 117 of 285 Deeper Understanding: Exam M November 7, 2006 2006 Yufeng Guo
http:/ / www.guo.coursehost.com

#4 John is 35 years old. He bought a life policy. If John dies any time in the next 10
years, his family gets $1 immediately upon Johns death. If, however, John is still alive
and kicking after the next 10 years, his family or John still gets $1. Whats the expected
value of this insurance policy?
Answer: This is a 10-year endowment policy where the death benefit is paid immediately
upon the death of the insured. The expected value at issue is:
1 1
1
35:10 35:10 35:10
10 35
35:10
A A E A A = + = + .
#5 John is 35 years old. He bought a life policy. If John dies any time in the next 10
years, his family gets $1 at the end of the year when John dies. If, however, John is still
alive after the next 10 years, his family or John still gets $1. Whats the expected value of
this insurance policy?
Answer: This is a 10-year endowment policy where the death benefit is paid at the end of
the year of death. The expected value at issue is:
1 1 1
10 35
35:10 35:10 35:10 35:10
A A E A A = + = + .
#6 John is 35 years old. If he buys insurance now, he can qualify for a low-rate class
and pay low premiums. He doesnt need insurance now, however, hes worried that his
health may deteriorate later, at which time he may need life insurance and have to pay
higher premiums. So he decides to buy a 5-year term insurance now while hes healthy,
but he wants this 5-year term insurance to kick in 10 years from now. The 5-year term
insurance pays $1 at the end of the year of death. Whats the expected value of this
insurance policy?
Answer: This is a 10-year deferred 5-year term insurance where the death benefit is paid
at the end of the year of death. The expected value at issue is
10 5
A.
#7 John is 35 years old. If he buys insurance now, he can qualify for a low-rate class
and pay low premiums. He doesnt need insurance now, however, hes worried that his
health may deteriorate later, at which time he may need life insurance and have to pay
higher premiums. So he decides to buy whole life insurance now while hes healthy, but
he wants it to kick in 10 years from now. This whole life insurance pays $1 at the end of
the year of death. Whats the expected value of this insurance policy?
Answer: This is a 10-year deferred whole life insurance where the death benefit is paid at
the end of the year of death. The expected value at issue is
10
A. You can also write
10
A
+
.
By now, you should be pretty comfortable with the business meanings and actuarial
symbols for the commonly-used life insurance policies. Next, you might want to look at
Actuarial Mathematics and familiarize yourself on how to rigorously define an insurance
policy using the following variables:

t
b , the death or survival benefits to be provided by the insurance policy
Page 118 of 285 Deeper Understanding: Exam M November 7, 2006 2006 Yufeng Guo
http:/ / www.guo.coursehost.com

Z , the present value random variable of the benefits provided by the insurance
policy.

I recommend that from time to time, as you are solving problems, refer to Actuarial
Mathematics Table 4.2.1 and 4.3.1. I dont memorize these tables; nor should you. The
key is, as stated earlier, to understand the business meaning of life insurance. Once you
understand how life insurance policies are used in the real world, you can quickly write
out
t
b and Z . That said, you do need to get comfortable with
t
b and Z because SOA
loves to test these symbols.

Formulas

Ill use term insurance as the example to illustrate the general procedure to calculate
( ) E Z and ( ) Var Z . For other types of insurance, you can easily modify these procedures
and calculate ( ) E Z and ( ) Var Z accordingly.

If the death benefit is paid at the end of year, then set up the following table to track
down the population at each time:

Age x x +1 x + 2 x + k x + n
Time t 0 1 2 k n
# of people alive
x
l
1 x
l
+ 2 x
l
+ x k
l
+ x n
l
+
# of deaths
1 x x
l l
+

1 2 x x
l l
+ +

1 x k x k
l l
+ +

1 x n x n
l l
+ +

death benefits per death


1
b
2
b
k
b
n
b
discount factor v
2
v
k
v
n
v
PV of the total death benefit if
x
l policies are issued @ 0 t = :
( ) ( ) ( )
2
1 1 2 1 2 1
...
n
x x x x n x n x n
b v l l b v l l b v l l
+ + + + +
+ + +
PV of the total death benefit if only one policy is issued @ 0 t = :
Page 119 of 285 Deeper Understanding: Exam M November 7, 2006 2006 Yufeng Guo
http:/ / www.guo.coursehost.com

( )
( ) ( ) ( )
2
1 1 2 1 2 1
...
n
x x x x n x n x n
x
b v l l b v l l b v l l
E Z
l
+ + + + +
+ + +
=
( )
( ) ( ) ( ) ( ) ( ) ( )
2 2
2
2
1 1 2 1 2 1
2
...
n
x x x x n x n x n
x
b v l l b v l l b v l l
E Z
l
+ + + + +
+ + +
=
( ) ( ) ( )
2 2
Var Z E Z E Z =
If you prefer, you can rewrite the following formula into:
( )
2 1 1 2 1
1 2
...
n x x x x x n x n
n
x x x
l l l l l l
E Z b v b v b v
l l l
+ + + + +

= + + +
( )
( ) ( ) ( )
2 3
1 1 2 3 1 1 2 1 1 1
...
n
x x x n x n
b v q b v q b v q b v q

= + + + +
( ) ( ) ( ) ( ) ( ) ( ) ( ) ( ) ( )
2 2 2
2
2 2 3
1 1 2 3 1 1 2 1 1 1
...
n
x x x n x n
E Z b v q b v q b v q b v q

= + + + +
( ) ( ) ( )
2 2
Var Z E Z E Z =
If the death benefit is paid at the moment of death:

( ) ( ) ( ) ( )
0 0
n n
t
t
E Z z t f t dt b v f t dt = =
} }

( ) ( ) ( ) ( ) ( )
2 2
2
0 0
n n
t
t
E Z z t f t dt b v f t dt = = (
} }

( ) ( ) ( )
2 2
Var Z E Z E Z =
Common problems and model solutions

Type 1 Discrete life insurance, find E(Z), Var(Z)

Example 1

Insurance 3 year term issued at age 50
Death benefit $1 payable at the end of the year
Mortality Illustrative Life Table
Interest rate 9%
Page 120 of 285 Deeper Understanding: Exam M November 7, 2006 2006 Yufeng Guo
http:/ / www.guo.coursehost.com

Calculate E(Z) and Var(Z)

Solution

This type of problem is conceptually easy. If it shows up on the exam, your first instinct
might be to blindly copy the following solution from the textbooks:

( )
1 2 3
50 50 51 2 50 53
50: 3
E Z A vq v p q v p q = = + +
( ) ( ) ( ) ( )
2 2
2
2 2 3
50 50 51 2 50 53
E Z v q v p q v p q = + +
( ) ( ) ( )
2 2
Var Z E Z E Z =
These formulas look great. However, if you actually use them on the exam, youll find
that they are clumsy and prone to errors. Errors can happen in each of the follow steps:

You need to calculate or look up
50 50 51 2 50
, , , q p q p and
53
q based on the
Illustrative Life Table.

You need to calculate E(Z ).

You need to calculate E(Z
2
). This step is most prone to errors. Many candidates
err in this step.

When studying for SOA exams, you need to understand that many solutions from the
textbooks are good for illustrating basic concepts, but often they are inefficient solutions
under exam conditions.

Authors of most textbooks use software to do statistics math, integration, matrix
multiplication, and other messy calculations. As a result, their calculation methods work
fine if computers do the math but may fall apart when humans actually have to perform
the calculations. You need to figure out fast manual solutions and which calculator keys
to use that will help you solve the problem 100% right every time. Wouldnt it be nice if
SOA allowed you to use a smart computer for the exam that will calculate the mean and
variance for you?

You might be shocked to know that (1) SOA indeed has allowed you to bring a smart
computer into the exam room, and (2) this smart computer works 24/7, flawlessly
calculating the mean and variance for you.

From Day One when SOA exams were invented, SOA let candidates bring a smart
computer into the exam room. The only trouble is that most candidates didnt know this.

The smart computer is called a BA II Plus or BA II Plus Professional calculator. This
calculator has a remarkable Statistics Worksheet. It calculates the mean and variance for
Page 121 of 285 Deeper Understanding: Exam M November 7, 2006 2006 Yufeng Guo
http:/ / www.guo.coursehost.com

a discrete random variable. All you need to do is to enter the data pair ( ) [ ]
i i
x P x , . Then
the Statistics Worksheet will work the magic for you.

Now lets solve the problem using BA II Plus or BA II Plus Professional. First, we need
to track down the distinct values of Z and its probabilities. We set up the following table:

Age 50 51 52 53 +
Time t 0 1 2 3 +
# of people
alive
8,950,901
(1)
8,879,913 8,840,770 8,779,.128 0
# of deaths 52,988
(2)

57,143 61,642 8,779,128
Z
09 . 1
1
= v
2
v
3
v
0
(1) You look up 8,950,901 from the Illustrative Life Table
(2) 52,988 = 8,950,901 - 8,879,913

Next, you enter the following data into BA II Plus or BA II Plus 1-V Statistics
Worksheet:

( )
1
09 . 1 01

= X , 988 , 52 01 = Y
( )
2
09 . 1 02

= X , 143 , 57 02 = Y
( )
3
09 . 1 03

= X , 642 , 61 03 = Y
0 04 = X , 128 , 779 , 8 04 = Y
BA II Plus or BA II Plus Professional will generate the following output:

8,950,901 = n . This should match the total population at age 50. If not, your data entry is
wrong.

01612216 . 0 = X . This is the ( ) Z E
11554921 . 0 =
X
o . This is the ( ) Z o .
( )
2
0.11554921 0.01335162 Var X = = .
Please note that to calculate ( )
2
x
Var X o = , you dont need to manually enter 0.11554924
and square it. When your calculator screen displays 11554921 . 0 =
X
o , simply press the
2
x button and your calculator will give you 0.01335162. Whats nice is that by pressing
Page 122 of 285 Deeper Understanding: Exam M November 7, 2006 2006 Yufeng Guo
http:/ / www.guo.coursehost.com

the
2
x button you wont change the internal value of 11554921 . 0 =
X
o . Internally, your
calculator still stores 11554921 . 0 =
X
o . After squaring 11554921 . 0 =
X
o and getting
0.01335162, if you want to retrieve 0.01335162, simply press:

+ (down arrow key)
| (up arrow key)

This will give you 11554921 . 0 =
X
o .
I recommend that you actually do the calculation using the following formula

( )
1 2 3
50 50 51 2 50 53
50: 3
E Z A vq v p q v p q = = + +
( ) ( ) ( ) ( )
2 2
2
2 2 3
50 50 51 2 50 53
E Z v q v p q v p q = + +
( ) ( ) ( )
2 2
Var Z E Z E Z =
See for yourself that BA II Plus or BA II Plus Professional gives you the right result.

You may wonder why BA II Plus or BA II Plus Professional gives you the right result.
Here is how the calculator works. Your data entry is:

( )
1
09 . 1 01

= X , 988 , 52 01 = Y
( )
2
09 . 1 02

= X , 143 , 57 02 = Y
( )
3
09 . 1 03

= X , 642 , 61 03 = Y
0 04 = X , 128 , 779 , 8 04 = Y
04 , 03 , 02 , 01 X X X X are four distinct values of your random variable. The Statistics
Worksheet now knows that your random variable has four values.

The 1
st
value is ( )
1
09 . 1

. This is the present value of the 1
st
year death benefit.
The 2
nd
value is ( )
2
09 . 1

. This is the present value of the 2
nd
year death benefit.
The 3
rd
value is ( )
3
09 . 1

. This is the present value of the 3
rd
year death benefit.
The 4
th
value is zero. This reflects the fact that all policyholders still alive at 3 = t
(or age 63) wont get any death benefit; the insurance is 3-year term.

Lets look at 01, 02, 03, 04 Y Y Y Y . These represent the frequency of occurrence. The
Statistics Worksheet now knows that:

( )
1
09 . 1 01

= X occurs 988 , 52 01 = Y times.
( )
2
09 . 1 02

= X occurs 143 , 57 02 = Y times.
Page 123 of 285 Deeper Understanding: Exam M November 7, 2006 2006 Yufeng Guo
http:/ / www.guo.coursehost.com

( )
3
09 . 1 03

= X occurs 642 , 61 03 = Y times.
0 04 = X occurs 128 , 779 , 8 04 = Y times.

Next the Statistics Worksheet figures out that the total frequency is:

52,988+57,143+61,642+8,779,128 = 8,950,901

The total frequency should be equal to the population at age 50. This makes sense. The
total deaths during the first 3 policy years plus the number of surviving policyholders at
3 = t should be equal to the starting population at 0 = t .
Then the Statistics Worksheet assigns the following probabilities to the random variable:

( )
901 , 950 , 8
988 , 52
01 = X P , ( )
901 , 950 , 8
143 , 57
02 = X P
( )
901 , 950 , 8
642 , 61
03 = X P , ( )
901 , 950 , 8
128 , 779 , 8
04 = X P
Finally, the Statistics Worksheet calculates the mean and standard deviation using the
following formulas:

( ) ( )

=
=
n
i
i i
x P x X E
1
( ) ( ) ( )
2 2
1
n
i i
i
X x P x E X o
=
=

Now you should be fully convinced that BA II Plus and BA II Plus Professional Statistics
Worksheet is a smart and capable machine.

You might say that its awful lot of work to use BA II Plus or BA II Plus Professional. It
looks like lot of work mainly because you are new to the Statistics Worksheet. Once you
understand how to use it, you can let the Statistics Worksheet find the mean and variance
for you 100% right in a hurry.
Example 2 (Continuation of Example 1)

After solving Example #1, you realize that you have misread the interest rate. The correct
interest rate is 6%, not 9%. So the correct inputs are:

Insurance 3-year term issued at age 50
Death benefit $1 payable at the end of the year
Mortality Illustrative Life Table
Interest rate 6%
Page 124 of 285 Deeper Understanding: Exam M November 7, 2006 2006 Yufeng Guo
http:/ / www.guo.coursehost.com

Calculate ( ) Z E and ( ) Z Var using the correct interest rate of 6%.

Solution

In the heat of the exam, you are under a lot of stress so its not unusual to misread data
and spend three minutes calculating the wrong thing. If you use the long-winded textbook
solution, you are out of luck. To correct your error, youll have to redo most of your
calculations and spend three more minutes on this problem plus a minute or two of brow
beating yourself for not being more careful.

This brings us to the second advantage of using the BA II Plus or BA II Plus Professional
Statistics Worksheet. Not only can you delegate the hard core calculation to your
calculator, you can also change your data input and redo calculations on the fly.

To redo the calculation using % 6 = i , you just need to re-enter 01, 02, 03 X X X and
reuse 01, 02, 03, 04 Y Y Y Y . So in the following data entry, only the numbers in red need
to be entered:

( )
1
01 1.06 X

= , 988 , 52 01 = Y
( )
2
02 1.06 X

= , 143 , 57 02 = Y
( )
3
03 1.06 X

= , 642 , 61 03 = Y
04 0 X = , 128 , 779 , 8 04 = Y
Now BA II Plus or BA II Plus Professional will generate the following output:

8,950,901 = n .
01704874 . 0 = X . This is ( ) Z E
12202291 . 0 =
X
o . This is ( ) Z o .
( ) 01488959 . 0 12202291 . 0
2
= = Z Var . This is
( ) Z Var
.
Page 125 of 285 Deeper Understanding: Exam M November 7, 2006 2006 Yufeng Guo
http:/ / www.guo.coursehost.com

Work for you

Problem #1 - You need to solve this problem 100% right in 2 minutes

You are given the following:

Insurance A special 5-year term issued at age 35
$1,000 if die at Yr 1
$3,000 if die at Yr 2
$5,000 if die at Yr 3
$7,000 if die at Yr 4
Death benefit payable at the end of
the year of death
$9,000 if die at Yr 5
Survival benefits $11,000 payable at the end of Yr 5 if still alive
Mortality Illustrative Life Table
Interest rate 11%
Calculate E(Z) and Var(Z)

Now cover up the page and dont look at my solution till youve attempted solving this
correctly in two minutes.

Solution
To simplify our calculation and data entry, we set $1,000 as one unit of money and
calculate E(Z) and Var(Z). Then multiply E(Z) by 1,000 and Var(Z) by 1,000
2
.
Age

35

36

37

38

39

40

+
Time t 0 1 2 3 4 5 +
# alive

9,420,657

9,401,688

9,381,566

9,360,184

9,337,427 9,313,166 0
# dead 18,969

20,122

21,382

22,757

24,261

9,313,166

Z
11 . 1
1
= v
2
3v
3
5v
4
7v
5
9v
5
11v
Enter the following data into BA II Plus or BA II Plus 1-V Statistics Worksheet:

( )
1
11 . 1 01

= X , 969 , 18 01 = Y
( )
2
11 . 1 3 02

= X , 122 , 20 02 = Y
Page 126 of 285 Deeper Understanding: Exam M November 7, 2006 2006 Yufeng Guo
http:/ / www.guo.coursehost.com

( )
3
11 . 1 5 03

= X , 382 , 21 03 = Y
( )
4
11 . 1 7 04

= X , 757 , 22 04 = Y
( )
5
11 . 1 9 05

= X , 261 , 24 05 = Y
( )
5
11 . 1 11 06

= X , 166 , 313 , 9 06 = Y
Now BA II Plus or BA II Plus Professional will generate the following output:

9,420,657 = n . This should match the total population at age 35. If not, your data entry is
wrong.

49368595 . 6 = X .
35998807 . 0 =
X
o .
( ) 12959141 . 0 35998807 . 0
2
= = X Var .
So ( ) ( ) 685949 . 493 , 6 000 , 1 = = X Z E , ( ) 4135 . 591 , 129 ) ( 000 , 1
2
= = X Var Z Var
Problem #2 You need to solve it in 100% right in one minute

You just discovered that you misread the death benefit patterns and the interest rate. The
correct inputs are:

Insurance A special 5-year term issued at age 35
$1,000 if die at Yr 1
$2,000 if die at Yr 2
$3,000 if die at Yr 3
$4,000 if die at Yr 4
Death benefit payable at the end of
the year of death
$5,000 if die at Yr 5
Survival benefits $6,000 payable at the end of Yr 5 if still alive
Mortality Illustrative Life Table
Interest rate 10%
Calculate E(Z) and Var(Z)

Now cover up the page and again dont look at my solution till youve tried it on your
own.

Solution
Enter the following data into the BA II Plus or BA II Plus 1-V Statistics Worksheet:

Page 127 of 285 Deeper Understanding: Exam M November 7, 2006 2006 Yufeng Guo
http:/ / www.guo.coursehost.com

( )
1
1 . 1 01

= X , 969 , 18 01 = Y
( )
2
1 . 1 2 02

= X , 122 , 20 02 = Y
( )
3
1 . 1 3 03

= X , 382 , 21 03 = Y
( )
4
1 . 1 4 04

= X , 757 , 22 04 = Y
( )
5
1 . 1 5 05

= X , 261 , 24 05 = Y
( )
5
11 . 1 6 06

= X , 166 , 313 , 9 06 = Y
Now BA II Plus or BA II Plus Professional will generate the following output:

9,420,657 = n . This should match the total population at age 35. If not, your data entry is
wrong.

70809087 . 3 = X .
18203293 . 0 =
X
o .
( ) 03313599 . 0 18203293 . 0
2
= = X Var .
So ( ) ( ) 090872 . 708 , 3 000 , 1 = = X Z E , ( ) 9887 . 135 , 33 ) ( 000 , 1
2
= = X Var Z Var
Problem #3 # 2 Fall 2004 Course 3 (Solve this in 2 minutes)

For a group of individuals all age x , you are given:

25% are smokers (s);
75% are nonsmokers (ns).
0.02 i =
k
s
x k
q
+
ns
x k
q
+
0 0.1 0.05
1 0.2 0.1
2 0.3 0.15
Calculate
1
: 2
10, 000
x
A for an individual chosen at random from this group.

Solution

( ) ( )
1 1 1
: 2 : 2 : 2
25% 75%
s ns
x x x
A A A = +
Page 128 of 285 Deeper Understanding: Exam M November 7, 2006 2006 Yufeng Guo
http:/ / www.guo.coursehost.com

For smokers:
Time t 0 1 2 3
# of people alive 1 0.9 0.9(0.8)=0.72 0.72 (0.7) =0.504
# of deaths 1-0.9=0.1 0.9-0.72=0.18 0.72-0.504=0.216
One method to calculate
( )
1
: 2
s
x
A is to use BA II Plus/BA II Plus Professional 1-V
Statistics Worksheet as we did in other problems. However, since this problem is simple,
we can directly calculate:

( )
1 2
: 2
0.1 0.18 0.271
s
x
A v v = + =
If you really want to save time and dont want to directly calculate
2
0.1 0.18 v v + , you can
use the BA II Plus/BA II Plus Professional Cash Flow Worksheet.
Enter the following into the Cash Flow Worksheet:
C01=0.1, C02=0.18, I=2
This tells the calculator you have two cash flows: $0.1 @ t=1 and $0.18 @ t=2;
and the interest rate is 2%.
The calculator should give you the following NPV:
NPV = 0.27104960
For non-smokers:
Time t 0 1 2 3
# of people alive 1 0.95 0.95(0.9)=0.855 0.855(0.85) = 0.72675
# of deaths 1-0.95=0.05 0.95-0.855=0.095 0.855-0.72675=0.12825
( )
1 2
: 2
0.05 0.95 0.140
ns
x
A v v = + =
( ) ( )
1
: 2
25% 0.271 75% 0.140 0.17275
x
A = + =
1
: 2
10, 000 $1, 727.5
x
A =
Page 129 of 285 Deeper Understanding: Exam M November 7, 2006 2006 Yufeng Guo
http:/ / www.guo.coursehost.com

Type 2 Continuous life insurance, find E(Z), Var(Z)

Problem #1 #1 Fall 2004 Course 3 (solve this in 2 minutes)
For a special whole life insurance on ( ) x , payable at the moment of death:
( ) 0.05, 0
x
t t = >
0.08 o =
The death benefit at time t is
0.06
, 0
t
t
b e t = >
Z is the present value random variable for this insurance at issue.

Calculate ( ) Var Z
Solution

Consider the time interval [ ] , t t dt + . The probability of death during this interval is
( ) ( )
( )
0
0.05
0.05
t
x
s ds
t
x
f t dt t e e

}
= = . We can also interpret ( ) f t dt as the number of
deaths during [ ] , t t dt + .
( ) ( ) ( )
( )

( )
0.06 0.08 0.05
0 0 0
0.05
t t t t
t
z t
E Z z t f t dt b e f t dt e e e dt
o
+ + +

= = =
} } }

0.07
0
0.05 5
0.05
0.07 7
t
e dt
+

= = =
}
( ) ( ) ( ) ( )
2
2 2 0.06 0.08 0.05
0 0
0.05 5
0.05
0.09 9
t t t
E Z z t f t dt e e e dt
+ +

= = = =
} }

( ) ( ) ( )
2
2 2
5 5
0.04535147
9 7
Var Z E Z E Z
| |
= = =
|
\ .

Integration shortcut (two formulas from Exam FM):
0
1
n n
t
n
e
e dt a
o
o
o
o


= =
}
( )
0
n n
n
t
n
a ne
t e dt I a
o
o
o
o
o


= =
}
Page 130 of 285 Deeper Understanding: Exam M November 7, 2006 2006 Yufeng Guo
http:/ / www.guo.coursehost.com

Free internet resources for learning life annuities

To learn more on the basics of annuity, I recommend that you look at:

http://www.annuityadvantage.com/immediate.htm and

http://www.totalreturnannuities.com/annuities-immediate/index.html.
Page 131 of 285 Deeper Understanding: Exam M November 7, 2006 2006 Yufeng Guo
http:/ / www.guo.coursehost.com

Chapter 8 Reserve

Whats reserve all about?

Assume that you want to buy a 20-year term life insurance policy and want to find the
cheapest premium on the market. Given this is the internet age, you decide to shop
premium rates online. So you use key words search and find http://www.lifeinsure.com.
This website gives you the instant quote.

Assume you have the following profile:
Sex Male
Do you smoke or use tobacco? No
Birth date June 1, 1961
Your health Preferred
Your state California
Initial insurance period 20 years
Amount of insurance $500,000
Premiums paid Annual
Enter your profile into http://www.lifeinsure.com and hit Submit to Calculate Your
Rates! Youll get something like this:

Quote Date Todays date (such as June 1,2005 if you request a quote
on June 1, 2005)
Face amount $500,000
Term life specifications 20-year level term
Applicants profiles 44-year old male non-smoker in preferred health
State Choose your state
Then, youll see a list of insurance companies that offer 20-year level term insurance and
their premium rates.

You see that you can buy, from many insurance companies, a 20-year term insurance
policy with level annual premiums. Lets think for a minute. Today, you are 44-years old.
As you get older and older each year, you are more likely to die and your premium
should go up each year. However, many insurance companies dont charge you annually-
increasing premiums to cover the annually-increasing mortality cost. They charge you a
level premium year after year. How can this be?

Insurance companies charge level premiums because level premium insurance policies
are easy for insurance sales agents to explain to potential customers. They are easy for
customers to understand and accept. Not many people like to see their bills increase year
after year.
Page 132 of 285 Deeper Understanding: Exam M November 7, 2006 2006 Yufeng Guo
http:/ / www.guo.coursehost.com

To offer level premium insurance, an insurance company must overcharge premium in
the early years when an insured is young and less likely to die. The insurer must
undercharge premium in later years when the insured is older and more likely to die.

Each year after collecting premiums, the insurance company puts the overpaid premiums
(overpaid premium is the collected premium minus the cost of insurance) into a fund. The
insurance company typically invests the overpaid premiums in government and corporate
bonds to accumulate interest.

The insurance company uses this fund to subsidize the cost of insurance in future years.
The cost of insurance in future years must be subsidized because in later years the
premiums collected are less than the cost of insurance.

This fund is called reserve. Simply put, reserve is money set aside today for rainy days
tomorrow.

The questions about reserve on Exam M assume a constant interest rate, even though in
reality, the interest rate earned by the reserve fluctuates from time to time. As a result, we
can think of the fund as a savings account with a fixed interest rate. In the early years of a
policy, extra premiums above and beyond the cost of the insurance are dumped into this
savings account. In later years, money from this savings account is taken out annually to
help pay the additional cost of the insurance.

Whats a benefit reserve?

If level premiums are calculated using the equivalence principle, then the reserve is called
the benefit reserve.

The point to remember here is that the amount of extra premium put aside in the early
years of a policy depends on how the level premiums are calculated. An insurance
company can use different methods to determine how much level premium dollars an
insured needs to pay each year. Different methods will produce different level premiums
and different reserve amounts.

For example, an insurer can set the level premiums such that the probability of incurring
a loss is no more than 0.1%. Alternatively, the insurer can set premiums using the
equivalence principle and require that on average the insurer breaks even (this is the
essence of the equivalence principle). These two methods will have different level
premiums and different reserves.

On Exam M, reserves are almost always based on the equivalence principle.

Page 133 of 285 Deeper Understanding: Exam M November 7, 2006 2006 Yufeng Guo
http:/ / www.guo.coursehost.com

How do I calculate reserve?

On Exam M, the benefit reserve at time t does not refer to the total value of the reserve
savings account at time t ; rather it refers to the value of the reserve savings account
per surviving policy.

The formula is:

( )
( )
( )
Accumulated value of the reserve savings account
Reserve
# of surviving policies
t
t
t
=
Steps to calculate the benefit reserve at time t
Step 1: For each year up to time t , find how much premium is really collected and how
much premium is really needed to pay the death benefit. The difference between these
two is the overpaid premium.

Step 2: Deposit the overpaid premiums collected from time zero to time t into a savings
account. Let these overpaid premiums accumulate interest. Calculate the total fund value
at time t .
Step 3: Divide the total fund value at time t by the number of policies still in force. The
result is the reserve at time t .
Example 1

Facts:
3-year term insurance
Premiums are collected at the beginning of the year
Death benefit of $100 is paid at the end of the year
10 =
x
l
9
1
=
+ x
l
6
2
=
+ x
l
1
3
=
+ x
l
Interest rate % 0 = i .
Calculate the benefit reserve at 3 , 2 , 1 , 0 = t .
Solution

Page 134 of 285 Deeper Understanding: Exam M November 7, 2006 2006 Yufeng Guo
http:/ / www.guo.coursehost.com

First, we need to solve for the level annual premium P .
Time t 0 1 2 3 Total
# of people alive 10 9 6 1
Premiums paid 10 P 9 P 6 P 25 P
# of deaths 10-9=1 9-6=3 6-1=5
Death benefit paid $100 $300 $500 $900
36 $ 900 25 = = P P
Policy Year 0 1 2 3
Time frame Immediately
before the
issue of the
policy
0 = t to 1 = t 1 = t to
2 = t
2 = t to 3 = t Total
Premiums
collected @
beginning of
the year
$0 10(36)=$360 9(36)=$324 6(36)=$216 $900
Death benefit
paid @ end of
the year
$0 $100 $300 $500 $900
Premium in
excess of death
benefit (we
deposit this
amount into the
reserve savings
account)
$0 $260 $24 - $284 $0
The value of
the reserve
savings account
@the end of
the year
$0 $260 $260+$24
=$284
$260+$24 -
$284
= $0

# of insureds
@end of the
year to share
the reserve
savings account
10 @ 0 = t 9 @ 1 = t 6 @ 2 = t 1 @ 3 = t
Reserve per
person @ end
of the year
$0 /10 = $0

@ 0 t =
$260/9=$28.89
@ 1 t =
$284/6
= $47.33
@ 2 t =
$0

@ 3 t =
Page 135 of 285 Deeper Understanding: Exam M November 7, 2006 2006 Yufeng Guo
http:/ / www.guo.coursehost.com

Lets walk through this table. At 0 = t (immediately before the policy is issued), no
premiums are collected and no death benefits are paid. As a result, the reserve savings
account has zero dollars in it.

During the first policy year from 0 = t to 1 = t , $360 in premiums is collected but only
$100 is truly needed to pay the death benefit (because theres only one death at 1 = t ). As
a result, $260 in overpaid premiums is deposited into the reserve savings account.

During the second policy year from 1 = t to 2 = t , $324 in premiums is collected but
only $300 is truly needed to pay the death benefit. As a result, $24 overpaid premiums
are deposited into the reserve savings account. At this time, the reserve savings account
has $260 + $24 = $284 in it.

During the third policy year from 2 = t to 3 = t , only $216 in premiums is collected but
$500 is truly needed to pay the death benefit. So theres a shortfall of $500 - $216 = $284.
Fortunately, the reserve savings account has exactly $284. So we dip into the reserve
savings account and take out all the money to help finance the death benefits.

The last row of the table calculates the reserve per person. Remember, when Exam M
talks about the reserve at time t , it refers to the reserve per surviving insureds (or per
surviving policy still in force). So we need to divide the value of the reserve savings
account by the number of insureds still alive at time t .
Key points from this example:
Origin of reserve The insurance company charges level premiums, even though
the mortality cost is annually increasing. As a result, the insurer must overcharge
premiums in the early years and undercharge premiums in later years.

The overcharged premiums are put into the reserve savings account to help pay
the death benefits in later years.

Reserve always starts off with zero before any policy is issued. Reserve always
ends with zero after the last claim is paid.

Example 2

Facts:
3-year term insurance
Premiums are collected at the beginning of the year
Death benefit of $100 is paid at the end of the year
10
x
l =

1
9
x
l
+
=

2
6
x
l
+
=

3
1
x
l
+
=
Interest rate 10% i = .
Page 136 of 285 Deeper Understanding: Exam M November 7, 2006 2006 Yufeng Guo
http:/ / www.guo.coursehost.com

Calculate the benefit reserve at 0, 1, 2, 3 t = .
Solution

First, we need to solve for the level annual premium P .
Time t 0 1 2 3
# of people alive 10 9 6 1
Premiums paid 10 P 9 P 6 P
# of deaths 10-9=1 9-6=3 6-1=5
Death benefit paid $100 $300 $500
We want to set the premium such that the insurer will break even and make zero profit
(the essence of the equivalence principle).

Present value @ 0 = t of the total premiums collected
= Present value @ 0 = t of the death benefits paid

( ) ( )
2 2 3
10 9 6 100 3 5 @10% P v v v v v + + = + +
( ) ( )
( ) ( )
1 2 3
2 3
2 1 2
1.1 3 1.1 5 1.1
3 5 7.145
100 100 100 $30.8766
10 9 6 23.1405 10 9 1.1 6 1.1
v v v
P
v v


+ +
+ +
= = = =
+ + + +

The premium in Example 2 is lower than that in Example 1. Premiums in this example
earn interest. So the insured pays less premium.

You can also calculate P by solving the following equation:

Future value @ 3 = t of the total premiums collected
= Future value @ 3 = t of the death benefits paid

Alternatively, the equivalence principle requires that the collected premiums and death
benefits paid have identical values at 3 = t .
( ) ( ) ( ) ( ) ( )
3 2 2
10 1 9 1 6 1 100 1 3 1 5 @10% P i i i i i
( (
+ + + + + = + + + +


( ) ( )
( ) ( ) ( )
2
3 2
1 3 1 5
100
10 1 9 1 6 1
i i
P
i i i
+ + + +
=
+ + + + +
( ) ( )
( ) ( ) ( )
2
3 2
1.1 3 1.1 5
951
100 30.8766
30.8
10 1.1 9 1.1 6 1.1
+ +
= = =
+ +

Page 137 of 285 Deeper Understanding: Exam M November 7, 2006 2006 Yufeng Guo
http:/ / www.guo.coursehost.com

The calculation of reserve year by year is a bit tricky. We have to account for the time
value of money.
Policy Year 0 1 2 3
Time frame Immediately
before the
issue of the
policy
0 = t to 1 = t 1 = t to 2 = t 2 = t to 3 = t
Premiums
collected @
beginning of the
year
$0 10(30.8766)
=$308.7662

@ 0 = t
9(30.8766)
=$277.8894

@ 1 = t
6(30.8766)
=$185.2596

@ 2 = t
Accumulated
value of the
premium @ end
of the year
$0 308.7662(1.1)
=$339.6428

@ 1 = t
$277.8894(1.1)
=$305.6783

@ 2 = t
185.2596(1.1)
=$203.7856

@ 3 t =
Death benefit
paid @ end of
the year
$0 $100

@ 1 = t
$300

@ 2 = t
$500

@ 3 t =
Premium in
excess of death
benefit
$0 $239.6428

@ 1 = t
$5.6783

@ 2 = t
- $296.2144
@ 3 t =
The value of the
reserve savings
account @ the
end of the year
$0 $239.6428

@ 1 = t
$239.6428(1.1)
+$5.6783
=$269.2854

@ 2 = t
$269.2854(1.1)
$296.2144
= - $0.0005 = $0
@ 3 t =
# of insureds @
end of the year
to share the
reserve savings
account
10 @ 0 = t 9 @ 1 = t 6 @ 2 = t 1 @ 3 = t
Reserve per
person @ end of
the year
$0 /10 = $0

@ 0 t =
$239.62428/9
=$26.6270
@ 1 t =
$269.2854/6
= $44.8809
@ 2 t =
$0

@ 3 t =
Once again, we see the reserve starts off with zero before any policy is issued. It ends
with zero after the last policy comes off the books.

Another definition of reserve at time t
Lets think through how we calculate the value of the reserve savings account at time t .
To find the value of the reserve savings account at time t , we look at the premium
payment history during [ ] t , 0 . For each premium collected during [ ] t , 0 , we use a portion
Page 138 of 285 Deeper Understanding: Exam M November 7, 2006 2006 Yufeng Guo
http:/ / www.guo.coursehost.com

of it to pay the death benefit. Anything left is above and beyond the death benefit and is
deposited in the reserve savings account. So we have:

Value of the reserve savings account @t
= Future value @t of the overpaid premiums during [ ] t , 0
= Future value @t of the premiums paid during [ ] t , 0
- Future value @t of the death benefits paid during [ ] t , 0
From the insurance companys viewpoint, the future value @t of premiums paid during
[ ] t , 0 is the revenue earned during [ ] t , 0 . The future value @t of the death benefits paid
during [ ] t , 0 is the cost incurred during [ ] t , 0 . The value of the reserve savings account
@t is simply the future value of the profit earned during [ ] t , 0 .

Value of the reserve savings account @t
= Future value @t of the profit earned during [ ] t , 0
Reserve per policy @t
= Future value @t of the profit earned during [ ] t , 0 / # of polices @t
= Profit per policy @t
Example 1

Facts:
3-year term insurance
Premiums are collected at the beginning of the year
Death benefit of $100 is paid at the end of the year
10
x
l =

1
9
x
l
+
=

2
6
x
l
+
=

3
1
x
l
+
=
Interest rate 10% i = .
Calculate the benefit reserve at 0, 1, 2, 3 t = , using the profit-related definition of reserve.

Page 139 of 285 Deeper Understanding: Exam M November 7, 2006 2006 Yufeng Guo
http:/ / www.guo.coursehost.com

Solution

Time t
0 1 2 3
# of people alive 10 9 6 1
Premiums collected $10P $9P $6P
# of deaths 1 3 5
Death benefits paid $100 $300 $500
First, we calculate the level premium P . The equivalence principle requires that the
insurance company earns zero profit. So the present value of premiums collected is equal
to the present value of death benefits paid.

( ) ( )
2 2 3
10 9 6 100 3 5 @10% P v v v v v + + = + +
( ) ( )
( ) ( )
1 2 3
2 3
2 1 2
1.1 3 1.1 5 1.1
3 5 7.145
100 100 100 $30.8766
10 9 6 23.1405 10 9 1.1 6 1.1
v v v
P
v v


+ +
+ +
= = = =
+ + + +

Alternatively, the equivalence principle requires that the collected premiums and death
benefits paid have identical values at 3 = t .
( ) ( ) ( ) ( ) ( )
3 2 2
10 1 9 1 6 1 100 1 3 1 5 @10% P i i i i i
( (
+ + + + + = + + + +


( ) ( )
( ) ( ) ( )
( ) ( )
( ) ( ) ( )
2 2
3 2 3 2
1 3 1 5 1.1 3 1.1 5
951
100 100 30.8766
30.8
10 1 9 1 6 1 10 1.1 9 1.1 6 1.1
i i
P
i i i
+ + + + + +
= = = =
+ + + + + + +

Next, we calculate the value of the reserve savings account and the reserve per each
remaining policy.
@ 0 = t Immediately before any policy is issued
Revenue = $0
Cost = $0
Profit=$0
Value of the reserve savings account = profit = $0
Reserve per policy = $0
Page 140 of 285 Deeper Understanding: Exam M November 7, 2006 2006 Yufeng Guo
http:/ / www.guo.coursehost.com

@ 1 = t Ending of the 1
st
policy year and immediately before the start of the 2
nd
policy
year (at this time, the 2
nd
premium is not collected yet)
Revenue: ( ) ( )( ) 10 1 10 30.8766 1.1 $339.6426 P i + = =
Cost: $100
Profit: $239.6426
Value of the reserve savings account: $239.6426
# of remaining policies to use this reserve saving account: 9
Reserve per policy @ 1 = t : $239.6426/ 9 = $26.6270
@ 2 = t Ending of the 2
nd
policy year and immediately before the start of the 3
rd
policy
year (at this time, the 3
rd
premium is not collected yet)
Revenue:
( ) ( ) ( ) ( ) ( )( )
2 2
10 1 9 1 10 30.8766 1.1 9 30.8766 1.1 $679.2852 P i P i + + + = + =
Cost: ( ) ( ) 100 1 300 100 1.1 300 $410 i + + = + =
Profit: 679.2852 410 =$269.2852
Value of the reserve savings account: $269.2852
# of remaining policies to use this reserve savings account: 6
Reserve per policy @ 1 = t : $239.6426 / 6 = $44.8809
@ 3 = t Ending of the 3
rd
policy year all policies have expired
Revenue:
( ) ( ) ( )
3 2
10 1 9 1 6 1 P i P i P i + + + + +
( ) ( ) ( ) ( ) ( )( )
3 2
10 30.8766 1.1 9 30.8766 1.1 6 30.8766 1.1 $950.9993 = + + =
Cost: ( ) ( ) ( ) ( )
2 2
100 1 300 1 500 100 1.1 300 1.1 500 $951 i i + + + + = + + =
Profit: 950.9993 - 951 $0 ~
Value of the reserve saving account: $0
Reserve per policy @ 1 = t : $0
Page 141 of 285 Deeper Understanding: Exam M November 7, 2006 2006 Yufeng Guo
http:/ / www.guo.coursehost.com

If we ignore any rounding errors, we get the same reserves if we interpret reserves as the
profit earned per remaining policy.

I recommend that you use the profit interpretation to solve reserve problems on the exam.
Revenue, costs, and profit are simple concepts to understand and apply.

Retrospective and prospective method

So far, our method for reserve calculation is retrospective. To find the value of the
reserve savings account at time t , we look back to the past and deposit all the overpaid
premiums collected during [ ] t , 0 to the reserve savings account. Then we let this savings
account accumulate with interest to time t .
Next, well learn how to calculate reserve using the prospective method.

Retrospective method

Value of reserve savings account @t
= Accumulating excess premiums collected during [ ] t , 0 to time t
= Accumulating profit earned during [ ] t , 0 to time t
Prospective method

Value of reserve savings account @t
= Discounting premium shortfalls during [ ] + , t to time t
= Discounting loss incurred during [ ] + , t to time t
Under the prospective method, we look forward into the future and identify all the extra
premiums needed to pay the future death benefits incurred during [ ] + , t . Then we
discount all these premium shortfalls to time t .
Page 142 of 285 Deeper Understanding: Exam M November 7, 2006 2006 Yufeng Guo
http:/ / www.guo.coursehost.com

Example 2

Facts:
3-year term insurance
Premiums are collected at the beginning of the year
Death benefit of $100 is paid at the end of the year
10
x
l =

1
9
x
l
+
=

2
6
x
l
+
=

3
1
x
l
+
=
Interest rate 10% i = .
Calculate the benefit reserve at 0, 1, 2, 3 t = using the prospective method.

Solution

Time t
0 1 2 3
# of people alive 10 9 6 1
Premiums collected $10P $9P $6P
# of deaths 1 3 5
Death benefits paid $100 $300 $500
The equivalence principle:

( ) ( )
2 2 3
10 9 6 100 3 5 @10% P v v v v v + + = + +
( ) ( )
( ) ( )
1 2 3
2 3
2 1 2
1.1 3 1.1 5 1.1
3 5 7.145
100 100 100 $30.8766
10 9 6 23.1405 10 9 1.1 6 1.1
v v v
P
v v


+ +
+ +
= = = =
+ + + +

Page 143 of 285 Deeper Understanding: Exam M November 7, 2006 2006 Yufeng Guo
http:/ / www.guo.coursehost.com

@ 0 = t Immediately before any policy is issued
Future: [ ] 3 , 0
Present value @ 0 = t of the future costs (death benefits) during [ ] 3 , 0
( )
2 3
100 3 5 $714.5 v v v + + =
Present value @ 0 = t of future revenue (PV of premium collected) during [ ] 3 , 0
( ) ( )
2 2
10 9 6 30.8766 10 9 6 $714.5 P v v v v + + = + + =
Present value @ 0 = t of the profit earned during [ ] 3 , 0 : $0
Value of the reserve saving account = $0
@ 1 = t
Future period: [ ] 3 , 1
PV @ 1 = t of the cost incurred during [ ] 3 , 1 :
( ) ( )
2 1 2
300 500 300 1.1 500 1.1 $685.9504 v v

+ = + =
PV @ 1 = t of the revenue collected during [ ] 3 , 1 :
( ) ( )
1
9 6 30.8766 9 6 1.1 $446.3072 P v

(
+ = + =


PV @ 1 = t of the loss (premium shortfalls) incurred during [ ] 3 , 1 :
$685.9504 - $446.3072 =$239.6432
Value of the reserve savings account: $239.6432
# of remaining policies to use this reserve savings account: 9
Reserve per policy @ 1 = t : $239.6432/ 9 = $26.6270
Page 144 of 285 Deeper Understanding: Exam M November 7, 2006 2006 Yufeng Guo
http:/ / www.guo.coursehost.com

@ 2 = t
Future period: [ ] 3 , 2
PV @ 2 = t of the cost incurred during [ ] 3 , 2 :
( )
1
500 500 1.1 $454.5455 v

= =
PV @ 2 = t of the revenue collected during [ ] 3 , 2 :
( ) 6 6 30.8766 $185.2596 P = =
PV @ 2 = t of the loss (premium shortfalls) incurred during [ ] 3 , 2 :
$454.5455 - $185.2596 =$296.2859
Value of the reserve savings account: $296.2859
# of remaining policies to use this reserve savings account: 6
Reserve per policy @ 2 = t : $296.2859/ 6 = $44.8810
@ 3 = t Ending of the 3
rd
policy year all policies have expired
Future period: [ ] + , 3
PV @ 3 = t of the cost incurred during [ ] + , 3 : $0
PV @ 3 = t of the revenue collected during [ ] + , 3 : $0
PV @ 3 = t of the loss (premium shortfalls) incurred during [ ] + , 3 : $0
Value of the reserve savings account: $0
Reserve per policy @ 3 = t : $0
You see that the prospective method generates the identical result (if we ignore any
rounding errors).

Page 145 of 285 Deeper Understanding: Exam M November 7, 2006 2006 Yufeng Guo
http:/ / www.guo.coursehost.com

Steps to calculate reserve at time t for a fully discrete insurance

Step 1 Draw a cash flow diagram
Step 2 Count how many people are alive and thus paying premiums
at 0, 1, 2,..., 1 t n = .
Step 3 Count how many people die year by year during [ ] 1 , 0 , [ ] 2 , 1 ,,
[ ] t t , 1 . They get death benefits.
Step 4 Calculate the level benefit premium by forcing the insurance
company to make zero profit (the equivalence principle). This
gives us the benefit premium.
Step 5 Retrospective Method
Calculate the profit earned by the insurance company during
[ ] t , 0 . Accumulate this profit to time t . This gives us the value
of the reserve savings account at time t .
Prospective Method
Calculate the loss incurred by the insurance company
during [ ] + , t . Discount this loss to time t . This gives us the
value of the reserve savings account at time t .
Step 6 Divide the value of the reserve savings account calculated in
Step 5 by the # of policies still in force at time t . This
gives us the reserve per policy at time t .
Problem 1

3-year fully discrete term insurance issued to ( ) x
k
1 k
b
+ x k
q
+
0 1 0.4
1 2 0.5
2 3 0.7
The interest rate % 10 = i
Calculate the reserve at 1 t =
Page 146 of 285 Deeper Understanding: Exam M November 7, 2006 2006 Yufeng Guo
http:/ / www.guo.coursehost.com

Solution

First, we calculate the number of people alive at . 3 , 2 , 1 , 0 = t At time zero, one person is
alive (you can think of 1 person as actually representing 1 billion people).

At time 1, the # of people alive: 1 1 0.4 0.6
x x
p q = = =
At time 2, the # of people alive: ( ) ( )
1 1
1 0.6 1 0.5 0.3
x x x x
p p p q
+ +
= = =
At time 3, the # of people alive: ( ) ( )
1 2 1 2
1 0.3 1 0.7 0.09
x x x x x x
p p p p p q
+ + + +
= = =
Next, lets figure out the # of deaths year by year.

# of deaths during the 1
st
policy year [ ] 1 , 0
= # of people alive @ 0 = t - # of people alive @ 1 = t = 1 1.6 = 0.4

# of deaths during the 2
nd
policy year [ ] 2 , 1
= # of people alive @ 1 = t - # of people alive @ 2 = t = 0.6 0.3 = 0.3

# of deaths during the 3
rd
policy year [ ] 3 , 2
= # of people alive @ 2 = t - # of people alive @ 3 = t = 0.3 0.09 = 0.21

Time t
0 1 2 3
# of people alive 1 0.6 0.3 0.09
Premiums collected $P $0.6P $0.3P
# of deaths 0.4 0.3 0.21
Death benefits per death $1 $2 $3
Total death benefits paid $0.4 $0.6 $0.63
Next, we calculate the benefit premium by forcing the insurer to make zero profit. We
need to consider the time value of money.

PV of future premiums collected - PV of future death benefits paid = 0

( )
2 2 3
1 0.6 0.3 0.4 0.6 0.63 0 @ 10% P v v v v v i + + + + = =
2 3
2
0.4 0.6 0.63 1.33283
$0.74319
1 0.6 0.3 1.79339
v v v
P
v v
+ +
= = =
+ +

Page 147 of 285 Deeper Understanding: Exam M November 7, 2006 2006 Yufeng Guo
http:/ / www.guo.coursehost.com

Retrospective Method:

Value of the reserve savings account @ 1 = t
= Profit earned during the 1
st
policy year
( ) ( ) 1 0.4 0.74319 1.1 0.4 0.41751 P i = + = =
Reserve per policy in force @ 1 = t
= Value of the reserve savings account @ 1 = t / # of policies in force @ 1 = t
= Value of the reserve savings account @ 1 = t / # of people alive @ 1 = t
= 0.41751 / 0.6 = $0.69585

Prospective Method:

Value of the reserve savings account @ 1 = t
= PV @ 1 = t of the loss incurred during the 2
nd
and 3
rd
policy years

PV @ 1 = t of the costs incurred during the 2
nd
and 3
rd
policy years
( ) ( )
2 1 2
0.6 0.63 0.6 1.1 0.63 1.1 1.06612 v v

= + = + =
PV @ 1 = t of the premiums collected during the 2
nd
and 3
rd
policy years
( ) ( )
1
0.6 0.3 0.74319 0.6 0.3 1.1 0.64860 P v

(
= + = + =


PV @ 1 = t of the losses incurred during the 2
nd
and 3
rd
policy years
= 1.06612 0.64860 = $0.41751

Reserve per policy in force @ 1 = t
= Value of the reserve savings account @ 1 = t / # of people alive @ 1 = t
=0.41751 / 0.6 = $0.69585

We can see that the retrospective method is faster in this problem.

Problem 2

Insurance
3-year fully discrete endowment issued to ( ) x
Death benefit $1,000 plus benefit premium
Endowment benefit $1,000
Interest rate 10%
Death rate
0.2, 0,1, 2
x n
q n
+
= =
Calculate the benefit reserve at 1 t =
Page 148 of 285 Deeper Understanding: Exam M November 7, 2006 2006 Yufeng Guo
http:/ / www.guo.coursehost.com

Solution

First, lets track down the number of people alive year by year.

@ 0 t = # of people alive = 1
@ 1 t = # of people alive 1 1 0.2 0.8
x x
p q = = = =
@ 2 t = # of people alive ( )
1
0.8 1 0.2 0.64
x x
p p
+
= = =
@ 3 t = # of people alive ( )
1 2
0.64 1 0.2 0.512
x x x
p p p
+ +
= = =
Next, lets find the number of deaths year by year.

# of deaths during Policy Year 1
= # of people alive @ 0 t = - # of people alive @ 1 t = = 1 0.8 = 0.2

# of deaths during Policy Year 2
= # of people alive @ 1 t = - # of people alive @ 2 t = = 0.8 0.64 = 0.16

# of deaths during Policy Year 3
= # of people alive @ 2 t = - # of people alive @ 3 t = = 0.64 0.512 =0.384

Time t
0 1 2 3
# of people alive 1 0.8 0.64 0.512
Premiums collected $P $0.8P $0.64P
# of deaths 0.2 0.16 0.128
death benefits paid ( ) 0.2 1000 P + ( ) 0.16 1, 000 P + ( ) 0.128 1000 P +
# of people survived @ 3 t = 0.384
survival benefit paid $384
Equivalence principle:

( )
2
1 0.8 0.64 P v v + +
( ) ( ) ( )
2 3 3
0.2 1, 000 0.16 1, 000 0.128 1000 384 @10% P v P v P v v = + + + + + +
Page 149 of 285 Deeper Understanding: Exam M November 7, 2006 2006 Yufeng Guo
http:/ / www.guo.coursehost.com

( ) ( )
2 3
2 3
0.2 0.16 0.512 0.698723
1, 000 1, 000 $378.5104
1 0.8 0.2 0.64 0.16 0.128 1.845980
v v v
P
v v v
+ +
= = =
+ +

Retrospective Method:

Value of the reserve savings account @ 1 = t
= Profit earned during the 1
st
policy year
( ) ( ) ( ) ( ) 1 0.2 1, 000 378.5104 1.1 0.2 1, 000 378.5104 $140.66 P i P = + + = + =
Reserve per policy in force @ 1 = t
= Value of the reserve savings account @ 1 = t / # of people alive @ 1 = t
=$140.66 / 0.8 = $175.82

Prospective Method:

Value of the reserve savings account @ 1 = t
= PV @ 1 = t of the losses incurred during the 2
nd
and 3
rd
policy years

PV @ 1 = t of the costs incurred during the 2
nd
and 3
rd
policy years:

( ) ( )
2 2
0.16 1, 000 0.128 1000 384 P v P v v + + + +
( ) ( )
2 2
0.16 1, 000 378.5104 0.128 1000 378.5104 384 v v v = + + + +
$663.6919 =
PV @ 1 = t of the premiums collected during the 2
nd
and 3
rd
policy years:

( ) ( )
1
0.8 0.64 378.5104 0.8 0.64 1.1 $523.0326 P v

(
+ = + =


PV @ 1 = t of the losses incurred during the 2
nd
and 3
rd
policy years
= 663.6919 523.0326 = $140.66

Reserve per policy in force @ 1 = t
= Value of the reserve savings account @ 1 = t / # of people alive @ 1 = t
=$140.66 / 0.8 = $175.82

Page 150 of 285 Deeper Understanding: Exam M November 7, 2006 2006 Yufeng Guo
http:/ / www.guo.coursehost.com

Problem 3

Insurance fully discrete 30-year deferred whole life
Insured Age 20
Death benefit $1
Premium Level annual benefit premiums over first 30 years
Interest rate 6%
Mortality Illustrative Life Table
Calculate the benefit reserve at the end of Year 10.

Solution

First, we need to find the level annual benefit premiums P .
Age 20 50
T 0 30 +
# of people alive
20
l
50
l
PV @ 0 t = of
20
20:30
l P a
premiums collected
during first 30 years
PV @ 30 t = of death benefits paid
50 50
l A
during [ ] 30, t e +
Lets walk through the above numbers. First, lets look at
50 50
l A . This insurance is a
whole life deferred 30 years. So each survivor at age 50 gets a whole life insurance policy.
Those who die before reaching age 50 get nothing. The number of people alive at age 50
is
50
l . So we have a total of
50
l whole life insurance policies at age 50. The cost of each of
these whole life insurance policies is
50
A . So the total cost of such whole life policies is
50 50
l A at 30 t = .
Next, lets look at
20
20:30
l P a . If we have only one insured age 20 who pays $1 annually
in advance for 30 years as long as he is alive, the present value of the premiums paid is
Page 151 of 285 Deeper Understanding: Exam M November 7, 2006 2006 Yufeng Guo
http:/ / www.guo.coursehost.com

20:30
a . Now we have
20
l insured paying $P for 30 years as long as they live, so the present
value of the premiums paid is
20
20:30
l P a .
We then apply the equivalence principle @ 0 t = . We force the insurance company to
make zero profit over the life of the insurance policy:

30
20 50 50
20:30
l P a l A v =
30
50 50
20
20:30
l A v
P
l a
=


Alternatively, we can apply the equivalence principle @ 30 t = . We force the premiums
and the death benefits to have identical future values at 30 t = .
( )
30
20 50 50
20:30
1 l P a i l A + =
30
50 50
20
20:30
l A v
P
l a
=


Next, we calculate the reserve savings account value @ 10 t =
Age 20 30 50
T 0 10 30 +
Value of the reserve savings account @ 10 t =
= Accumulating profit earned during [ ] 0, 10 t e to 10 t =
Accumulating premiums collected during [ ] 0, 10 t e to 10 t =
= ( ) ( )
30
10 10
20:10 20 50 50
20 20 50 50
20:10 20:10
20
20:30 20:30
1 1
a
l A v
l P i l i l A v
l a a
a a
| |
+ = + =
|
|
\ .



Accumulating death benefit incurred [ ] 0, 10 t e to 10 t = :
= $0 (1
st
claim occurs after 30 t = ; no claim occurs before 30 t = )
Accumulating profit earned during [ ] 0, 10 t e to 10 t =
=
20:10 20
50 50
20:30
a
l A v
a


Page 152 of 285 Deeper Understanding: Exam M November 7, 2006 2006 Yufeng Guo
http:/ / www.guo.coursehost.com

So the value of the reserve savings account @ 10 t = is
20:10 20
50 50
20:30
a
l A v
a

.
# of insurance policies still in force @ 10 t = :
30
l
Reserve @ 10 t = per policy in force:

20:10 20
50 50
20:10 20 20:30 50
50
30 30
20:30
a
l A v
a
a
l
A v
l l a
=


We can easily find
30
l ,
50
l ,
50
A from the Illustrative Life Table. The tricky part is to
calculate
20:10
a and
20:30
a .
10
20 10 20 30
20:10
a a p v a = +
10 10 30
20 10 20 30 20 30
20:10
20
l
a a p v a a v a
l
= =
30
20 30 20 50
20:30
a a p v a = +
30 30 50
20 30 20 50 20 50
20:30
20
l
a a p v a a v a
l
= =
10 30
20 30
20:10 20 20 50 50 20
50 50
30 50
30 30
20:30
20 50
20
l
a v a
a
l l l
A v A v
l
l a l
a v a
l


From the Illustrative Life Table, we have:

20
l
9,617,802
30
l
9,501,381
50
l
8,950,901
50
A
0.24905
20
a 16.5133
30
a 15.8561
50
a 13.2668
Page 153 of 285 Deeper Understanding: Exam M November 7, 2006 2006 Yufeng Guo
http:/ / www.guo.coursehost.com

Plugging in all the numbers, we have:
10 30
20 30
20:10 20 20 50 50 20
50 50
30 50
30 30
20:30
20 50
20
0.04
l
a v a
a
l l l
A v A v
l
l a l
a v a
l

= =



Problem 4

Insurance fully discrete 30-year deferred life annuity of $1
Insured Age 20
Premium Level annual benefit premiums over first 30 years
Interest rate 6%
Mortality Illustrative Life Table
Calculate the benefit reserve at the end of Year 10.

Solution

This problem is identical to Problem 3 except here the benefits paid are an annuity (as
opposed to death benefits).

Age 20 50
T 0 30 +
# of people alive
20
l
50
l
PV @ 0 t = of
20
20:30
l P a
premiums collected
during first 30 years
PV @ 30 t = of annuity benefits paid
0 50 5
l a
during [ ] 30, t e +
Its
50 50
l A in Problem 3
Page 154 of 285 Deeper Understanding: Exam M November 7, 2006 2006 Yufeng Guo
http:/ / www.guo.coursehost.com

We dont need to reinvent a solution here. We simply use the solution to Problem 3,
except that we need to replace
50
A with
50
a .
So the reserve 10 t = is:
50 50
10 30
20 30
20:10 20 20 50 50 20
30 50
30 30
20:30
20 50
20
2.10713362 2.11
l
a v a
a
l l l
v v a a
l
l a l
a v a
l

= = ~



Problem 5

Insurance Fully discrete whole life with premiums annually increasing
Insured Age 40
Death benefits $1,000
Annual premiums The next premium is always 10% larger than the previous
premium
Mortality De Moivres law with 120 e =
Interest rate 10%
Calculate the reserve at the end of Year 10.

Solution

First, you need to understand that under De Moivres law

( )
1
f t
x e
=

(a constant) where 0 t e s s
( )
1 1
1
1 1 1
n n
n x n x
n n
n n
q q f t dt dt
x x x e e e
+ +
+
+
= = = =

} }
(a constant)
where 0,1,..., 1 n x e =
1 n x n x
q q
+
is a constant means that each year, a constant fraction of the original
population
x
l dies each year. In other words, the # of deaths each year is
1
x
l
x e
| |
|

\ .
.
In this problem, 40 x = and 120 e = . So

( )
1 1 1
120 40 80
f t
x e
= = =


1
1
80
n x n x
q q
+
=
Page 155 of 285 Deeper Understanding: Exam M November 7, 2006 2006 Yufeng Guo
http:/ / www.guo.coursehost.com

1
1
80
n x n x
q q
+
= means that
1
80
of the starting population
x
l dies each year. So the
number of deaths each year is
1
80
x
l
@ 0 t = we have
x
l insureds alive.
@ 1 t = we have
1 79
80 80
x x x
l l l = insureds alive.
@ 2 t = we have
2 78
80 80
x x x
l l l = insureds alive.

@ 79 t = we have
79 1
80 80
x x x
l l l = insureds alive.
@ 80 t = we have
80
0
80
x x
l l = insureds alive (everyone is dead).
Next, we draw a diagram to track down population:

Age 40 41 42 119 120
T 0 1 2 79 80
# people alive
x
l
79
80
x
l
78
80
x
l
1
80
x
l 0
# of deaths
1
80
x
l
1
80
x
l
1
80
x
l
1
80
x
l
To make things simple, we arbitrarily set 80
x
l = :
Age 40 41 42 119 120
T 0 1 2 79 80
# people alive 80 79 78 1 0
# of deaths 1 1 1 1
Page 156 of 285 Deeper Understanding: Exam M November 7, 2006 2006 Yufeng Guo
http:/ / www.guo.coursehost.com

General diagram for De Moivres law:

Age x 1 x + 2 x + 1 e e
T 0 1 2 1 x e x e
# people alive x e 1 x e 2 x e 1 0
# of deaths 1 1 1 1
Please memorize the above diagram. Its a time saver in the exam when you need to
calculate premiums or reserve under De Moivres law.

Come back to our current problem ...

Age 40 41 42 119 120
T 0 1 2 79 80
# people alive 80 79 78 1 0
Premiums 80y ( ) 79 1.1 y
( )
2
78 1.1 y
( )
79
1.1 y
# of deaths 1 1 1 1
Death benefits $1,000 $1,000 $1,000 $1,000
In the above diagram, y is the 1
st
year premium.

We apply the equivalence principle @ 0 t = :
PV @ 0 t = of the premiums collected during the life of the policy
= PV @ 0 t = of the death benefits paid during the life of the policy

( ) ( ) ( )
2 2 79 79
80
80 79 1.1 78 1.1 ... 1.1 1, 000
i
y y v y v y v a + + + + =
Page 157 of 285 Deeper Understanding: Exam M November 7, 2006 2006 Yufeng Guo
http:/ / www.guo.coursehost.com

In the above equation,
80 i
a is PV of an 80-year annuity immediate (an Exam FM symbol).
It discounts with interest and doesnt discount with survivorship.

Because
1
1.1 v

= , we have ( ) 1.1 1 v = . How nice!

( )
80 10%
80 79 78 ... 1 1, 000 y a + + + + =
( )( )
1
80 79 78 ... 1 80 81 3, 240
2
+ + + + = =
The general formula is
( ) 1
1 2 ...
2
n n
n
+
+ + + = .
80 10%
1, 000
3, 240
y a =
Next, lets calculate the reserve @ 10 t = .
PV @ 0 t = of the premiums collected during the first 10 policy years
( ) ( )
71 80
80 79 78 ... 71 10 755
2
y y y
+
+ + + + = =
PV @ 0 t = of the premiums collected during the first 10 policy years
10 10%
1, 000 a
Discounting the profit earned during the first 10 years back to time zero:

10 10% 10 10% 80 10% 10 10%
1, 000
755 1, 000 755 1, 000 755 1, 000
3, 240
y a y a a a
| |
= =
|
\ .

Page 158 of 285 Deeper Understanding: Exam M November 7, 2006 2006 Yufeng Guo
http:/ / www.guo.coursehost.com

Recursive formulas for reserve

Lets examine how reserve changes during the policy year t . At time t , the reserve is
( ) R t . During the policy year t , the premium is ( ) P t . Whats the reserve at 1 t + ?
Age x x t + 1 x t + +
Time ( ) T x
t 1 t +
# of people alive
x t
l
+ 1 x t
l
+ +
(or # of policies in force)
Value of reserve savings account @t ( )
x t
R t l
+
Premiums collected in policy yr t ( )
x t
P t l
+
# of deaths in policy yr t
1 x t x t
l l
+ + +

Death benefits paid in policy yr t ( )


1 1 x t x t t
l l b
+ + + +

Total value of the reserve savings account @ 1 t + :


( ) ( ) ( ) ( ) ( )
1 1
1 1
x t x t x t x t t
l R t i P t l i l l b
+ + + + + +
+ + + (

Lets walk through the above expression. At time t , the reserve is ( ) R t . Remember this
is per policy share of the reserve account value. Because we have a total of
x t
l
+
policies
in force, the total dollar value of the reserve savings account at t is ( )
x t
R t l
+
. This reserve
savings account accumulates with interest from t to 1 t + and becomes ( ) ( ) 1
x t
l R t i
+
+ at
1 t + .
In addition to the existing money in the reserve savings account, during the policy year t ,
we have new reserve coming in. During the policy year t , we collect ( )
x t
P t l
+
premium
dollars. We also pay ( )
1 1 x t x t t
l l b
+ + + +
death benefits, where
1 t
b
+
is the death benefit paid
per death claim.
Page 159 of 285 Deeper Understanding: Exam M November 7, 2006 2006 Yufeng Guo
http:/ / www.guo.coursehost.com

The excess of premiums over death benefit is:

( ) ( ) ( )
1 1
1
x t x t x t t
P t l i l l b
+ + + + +
+
So the value of the total reserve savings account at 1 t + is:

( ) ( ) ( ) ( ) ( )
1 1
1 1
x t x t x t x t t
l R t i P t l i l l b
+ + + + + +
+ + + (

At 1 t + , the total # of policies still in force is
1 x t
l
+ +
. As a result, the per policy share of the
reserve savings account at 1 t + is:

( )
( )( ) ( ) ( ) ( )
1 1
1
1 1
1
x t x t x t x t t
x t
l R t i P t l i l l b
R t
l
+ + + + + +
+ +
+ + + (

+ =
We can rewrite the above formula into:

( )
( ) ( ) ( ) ( )
1 1
1
1
1
x t x t x t t
x t
P t R t l i l l b
R t
l
+ + + + +
+ +
+ + ( (

+ =
( ) ( ) ( ) ( )
1 1
1
1
x t x t x t t
x t
x t
x t
P t R t l i l l b
l
l
l
+ + + + +
+
+ +
+
+ + ( (

=
( ) ( ) ( )
1
1
1
1
x t x t
t
x t
x t
x t
l l
P t R t i b
l
l
l
+ + +
+
+
+ +
+

+ + (

=
But
1 1 x t x t x t
x t x t
x t x t
l l l
p q
l l
+ + + + +
+ +
+ +

= =
( )
( ) ( ) ( )
1
1
1
x t t
x t
P t R t i q b
R t
p
+ +
+
+ + (

+ =
A faster way to prove and memorize the above formula is to consider only one policy in
force at time t . For each policy in force at time t , the insurer has earned ( ) R t profit
from its previous operation from time zero to time t . Then at time t , the insurer receives
Page 160 of 285 Deeper Understanding: Exam M November 7, 2006 2006 Yufeng Guo
http:/ / www.guo.coursehost.com

( ) P t new premium. So for each policy in force at time t , the insurer has a total wealth of
( ) ( ) R t P t + . This amount accumulates with interest to 1 t + . Then at 1 t + , the insurer has
x t
q
+
death claims and pays out
1 x t t
q b
+ +
death benefits. The total operating profit at 1 t + is
( ) ( ) ( )
1
1
x t t
P t R t i q b
+ +
+ + (

. This profit is shared by
x t
p
+
surviving policies at 1 t + . So
the profit per policy at 1 t + is:

( )
( ) ( ) ( )
1
1
1
x t t
x t
P t R t i q b
R t
p
+ +
+
+ + (

+ =
We can rewrite the above formula into:

( ) ( ) ( ) ( )
1
1 1
x t t x t
P t R t i q b R t p
+ + +
+ + = + + (


( ) ( ) ( )
1
1
x t t x t
P t R t q b v R t p v
+ + +
+ = + +
Because 1
x t x t
p q
+ +
+ = , the above formula can also be written as:

( ) ( ) ( ) ( ) ( )
1
1 1 1
t x t
R t P t R t i b R t q
+ +
+ = + + + ( (

]
Or ( ) ( ) ( ) ( ) ( ) 1 1 1
x t
R t P t R t i q NAR t
+
+ = + + + (


( ) ( )
1
1 1
t
b R t NAR t
+
+ = + is called the net amount of risk. The net amount of risk is the
excess of death benefit over the reserve. If a policyholder is alive at 1 t + , his share of the
reserve savings account is ( ) 1 R t + . However, if he dies, the insurer needs to pay him
1 t
b
+
.
So ( )
1
1
t
b R t
+
+ is the risk facing the insurer.

Example May 2000 #3

For a fully discrete two-year term insurance of 400 on ( ) x :
0.1 i =
1
: 2
400 74.33
x
P =
1
1
: 2
400 16.58
x
V =
Page 161 of 285 Deeper Understanding: Exam M November 7, 2006 2006 Yufeng Guo
http:/ / www.guo.coursehost.com

The contract premium equals the benefit premium.

Calculate the variance of the loss at issue.

Solution

First, we need to find
x
q and
1 x
q
+
.
We know that the reserve at 1 t = is $16.58. This is the profit per policy accumulated
from 0 t = to 1 t = .
Lets consider the time interval [ ] 0, 1 t e . The insurer collects revenue of 74.33 (premium)
at 0 t = and pays the death benefit of 400
x
q at 1 t = . So at 1 t = , the insurers profit is

( ) 74.33 1 400
x
i q +
This profit is shared by
x
p remaining policies at 1 t = . The per policy profit at 1 t = is

( ) 74.33 1 400
x
x
i q
p
+
This should be the reserve of $16.58 at 1 t = .
( ) 74.33 1 400
16.58
x
x
i q
p
+
= , 0.17
x
q =
Next, we consider the time interval [ ] 1, 2 t e . The insurers profit at 2 t = should be zero.
This is a two-year term insurance and the premiums are calculated using the equivalence
principle. As a result, the insurer makes zero profit when the final death claim is paid off
at 2 t = .
At 1 t = , for each policy in force, the insurer has 16.58 profit from its previous operation.
Then the insurer immediately collects a new premium of 74.33. The sum of these
accumulates with interest to 2 t = . Then at 2 t = , the insurer pays death claims of 400
x
q
Total profit at 2 t = :
( )( )
1 1
death benefits accumulate reserve and premium
paid at 2
with interest from 1 to 2
74.33 16.58 1 400 0, 0.25
x x
t
t t
i q q
+ +
=
= =
+ + = =


Page 162 of 285 Deeper Understanding: Exam M November 7, 2006 2006 Yufeng Guo
http:/ / www.guo.coursehost.com

Now, we can calculate ( ) Var L .
Well use the BA II Plus or BA II Plus Professional 1-V Statistics Worksheet to find
( ) Var L . First, we need to list all the possible values of L and the associated probabilities:

Scenarios Probability PV of loss at 0 t =
( ) x dies in the 1
st
year
0.17 400 74.33 289.3 v =
( ) x dies in the 2
nd
year ( ) 1 0.17 0.25 0.2075 = ( )
2
400 74.33 1 188.68 v v + =
( ) x dies after 2 t = ( ) ( ) 1 0.17 1 0.25 0.6225 = ( ) 74.33 1 141.90 v + =
Check: the sum of all probabilities should be one. If the sum is not one, we have not
included all the possible scenarios.

0.17+0.2075+0.6225=1 (OK)

Next, we enter the following data pairs (X,Y) in the Statistics Worksheet:

Probability X Y
Scaled up probability
=probability*10,000
0.17 289.3 1,700
( ) 1 0.17 0.25 0.2075 =
188.68 2,075
( ) ( ) 1 0.17 1 0.25 0.6225 =
-141.9 6,225
In other words, we enter

X01=289.3, Y01=1,700
X02=188.68, Y02=2,075
X03=-141.9, Y03=6,225

We get:

0.00065 X =
(this is not exactly zero because we rounded present values of loss at 0 t = )
184.7958605
X
o =
2
34,149.51007
X
o =
Page 163 of 285 Deeper Understanding: Exam M November 7, 2006 2006 Yufeng Guo
http:/ / www.guo.coursehost.com

Reserve for variable death benefit

#21 Fall 2003

We are asked to find the reserve at 21 t = . If we can find the reserve at 20 t = , we can
use the recursive formula to find the reserve at 21 t = .
If we look at the segment of the insurance from 0 t = to 21 t = , we find that the premium
charged during this segment is a whole life premium for age ( ) 40 where the death
benefit is $1,000. Well also find that the death benefit during this segment is $1,000.

So the insurance from 0 t = to 21 t = is really a segment of whole life insurance of
$1,000 issued to ( ) 40 . As a result, the reserve at 20 t = is the reserve for a whole life
insurance policy of $1,000 issued to ( ) 40 .
60
20 20 40
40
11.1454
1, 000 1, 000 1 1, 000 1 247.78
14.8166
a
V V
a
| |
| |
= = = =
| |
\ .
\ .


We can easily calculate
40
1, 000P .
40
40
40
161.32
1, 000 1, 000 10.89
14.8166
A
P
a
= = =

Next, we apply the recursive reserve formula from 20 t = to 21 t = .


( ) ( ) ( )
20 40 60 60 21
5, 000 1 21 V P i DB q p V + + = +
( ) ( ) ( )
21
247.78 5 10.8878 10.6 5, 000 0.1376 1 0.1376 V + = +
21
255.06 V =
Page 164 of 285 Deeper Understanding: Exam M November 7, 2006 2006 Yufeng Guo
http:/ / www.guo.coursehost.com

How to calculate reserve for a fully continuous insurance

If the insurance is fully continuous, then you use integration, instead of summation, to
calculate the profit per policy at time t .
May 2000 #9

For a 10-year deferred whole life annuity of one on (35) payable continuously:

Mortality follows De Moivres law with 85.

0 i =
Level benefit premiums are payable continuously for 10 years.

Calculate the benefit reserve at the end of five years.

Solution

Age 35 45 85
Time 0 10 50

pay premium benefit period
# of people 50 40 0
alive

In De Moivres law, the # of deaths occurs at a constant speed. The starting age is 35 and
the max age is 85. So 50 people died from 0 t = to 50 t = . To simplify our calculation
and thinking, we assume we start off with fifty people aged 35. In other words, we
assume that fifty customers each aged 35 purchased this 10-year deferred whole life
annuity of $1. Then at any time t , the # of people alive is 50 t
Page 165 of 285 Deeper Understanding: Exam M November 7, 2006 2006 Yufeng Guo
http:/ / www.guo.coursehost.com

Once again,

As a general rule for simplification, we can assume that the starting population of ( ) x is
x e . This way, all these x e will die off at t x e = .
At any time t , the # of people alive is
t
l x t e = where 0 t x e s s .
First, lets calculate the benefit premium. Only those still alive at age 45 will get paid
continuously at an annual rate of $1 per year. Those who died before reaching age 45
paid premiums but didnt get anything. This is similar to life insurance. Annuity is also
self-supporting. The difference between life insurance and annuity is this:

In life insurance, those who live long (who paid a lot of premium) subsidize those who
die early (who paid less premium).
In annuity, those who die early (who received fewer annuity payments) subsidize those
who live long (who received more annuity payments).
Lets calculate the cost of the annuity payments. At any time t where 10 50 t s s , the
number of people alive (thus still receiving annuity payments) is 50
t
l t = . So during a
time interval [ ] , t t dt + , the number of people alive is 50
t
l t = . Each of them receives
$1 per year continuously. So during [ ] , t t dt + , each of the 50 t people receives dt
dollars. So the insurer will pay ( ) 50 t dt dollars during [ ] , t t dt + . So the PV at 0 t = of
the total benefits is:

( )
50
10
50
t
t e dt
o

}
This is a tricky integration, but fortunately SOA sets 0 o = . So the PV of the benefits at
0 t = is:

( ) ( )
0 50 0
2 2
40 10 40
1 1
50 40 800
2 2
t dt s ds s
(
= = = =
(

} }

Next, lets calculate the PV of future premiums. Let P represent the benefit premium.
Premiums are collected during 0 t = to 10 t = . Consider a time interval [ ] , t t dt + . The
Page 166 of 285 Deeper Understanding: Exam M November 7, 2006 2006 Yufeng Guo
http:/ / www.guo.coursehost.com

number of people alive during this interval is 50
t
l t = . Each of them pays $ P
premiums per year continuously. So the PV at 0 t = of the premiums collected is:

( )
10
0
50 450 P t dt P =
}
Applying the equivalence principle:

450 800 1.778 P P = =
Finally, we are ready to calculate the reserve at 5 t = . The accumulated value of all the
premiums collected at 5 t = is:

( )( ) ( ) ( )
5 5 5
5
0 0 0
50 1 50 1.778 50 422. 23
t
P t i dt P t dt t dt

+ = = =
} } }

However, no benefits are paid out during [ ] 0, 5 t e . So the accumulated value of the
reserve fund is $422.23.

To find the reserve at 5 t = , we use the following formula:

( )
Reserve fund value Reserve fund value
5
# of surviving policies # of people alive
R t = = =
The number of people alive at 5 t = is: 50 5 = 45. So the reserve at 5 t = is:

( )
422. 23
5 9. 3829
45
R t = = =
I didnt use any fancy formulas or notations to solve this problem. I recommend that you
understand my solution. This forces you to dig deeper into the problem and understand
the essence of the calculations. Once you understand my solution, then you can use the
SOA solution if you think its faster.

Page 167 of 285 Deeper Understanding: Exam M November 7, 2006 2006 Yufeng Guo
http:/ / www.guo.coursehost.com

Chapter 9 Asset share

Asset share at time t is the expected profit earned during [ ] t , 0 divided by the number of
survivors at t .
Asset share calculations are typically done retrospectively. In other words, generally we
dont divide the expected prospective profit to be earned during [ ] , t by the number of
survivors at time t to get the asset share. Rather, well accumulate the past profit earned
to time t and then divide this profit by the number of survivors to calculate the asset
share.

The reason for using the retrospective profit lies in the goal of asset share calculation.
Asset share calculation is meant to give the insurance company a good measure of how
much profit has been earned historically for each surviving policy, not how much profit is
to be earned in the future.

Because the actual experience varies year by year, the prospective profit and the
retrospective profit may differ. For example, an insurer may earn 6% on its investment
for the first two years and 7% Year 3 and beyond. Because the interest rate varies year
by year, the profit that has actually emerged is different from the profit to be earned in the
future.

In addition, asset share calculation uses contract premiums. Asset share calculation also
considers expenses (e.g. costs associated with collecting premium and investigating
claims) and lapses (a policyholder may back out from an insurance contract and stops
paying premiums). Once again, the goal of the asset share is to calculate realistically how
much profit has emerged per policy in force. As a result, asset share calculation needs to
consider the contract premium (gross premium), expenses, and lapses.

In contrast, benefit reserve is a theoretical calculation. It uses simplified assumptions that
expenses are zero and the lapse rate is zero. It uses benefit premium, not contract
premium. Both the retrospective and the prospective method can be used. Benefit reserve
at time t is the expected profit earned during [ ] t , 0 or the expected loss during [ ] , t
divided by the number of survivors at t . Finally, the interest rate used in the asset share
calculation may be different from the interest rate used in the benefit reserve calculation.

For Exam M purposes, SOA may use the same interest rate for both the asset share
calculation and the benefit reserve calculation, though in the real world the two interest
rates are often different. In addition, SOA problems may set the lapse rate to zero.

Page 168 of 285 Deeper Understanding: Exam M November 7, 2006 2006 Yufeng Guo
http:/ / www.guo.coursehost.com

Example 1 (M #16, May 2005 modified)

For a fully discrete whole life insurance of 1,000 on ( ) x :
Death is the only decrement.
Annual benefit premium is 80.
Annual contract premium is 100.
Expense in Year 1, payable at the start of the year, is 40% of the contract premium.
10% i =
5%
x
q =
Calculate the benefit reserve and the asset share at the end of Year 1.

Solution

Benefit reserve

Policy Year 1
Time 0 1
# of people alive 1 0.95
Benefit premium collected $80
# of deaths 0.05
Death benefits paid 0.05(1,000)=$50
Accumulated profit @ 1 = t : 80(1.1)-50=$38
# of survivors @ 1 = t : 0.95

Benefit reserve @ 1 = t : $38 / 0.95=$40

Asset share

Policy Year 1
Time 0 1
# of people alive 1 0.95
Contract Premium collected $100
# of deaths 0.05
Death benefits paid 0.05(1,000)=$50
Expense 40%(100)=$40
Accumulated profit @ 1 = t : (100-40)(1.1)-50=$16
# of survivors @ 1 = t : 0.95

Asset share @ 1 = t : $16 / 0.95=$16.84
Page 169 of 285 Deeper Understanding: Exam M November 7, 2006 2006 Yufeng Guo
http:/ / www.guo.coursehost.com

Example 2 (M #16, May 2005 modified)

For a fully discrete whole life insurance of 1,000 on ( ) x :
Decrements: deaths and withdrawals.

Annual benefit premium is 80. Annual contract premium is 100.

Acquisition expense in Year 1, payable at the start of the year, is 40% of the contract
premium.

Lapse handling expense: $20 per lapse

Surrender benefit: $100 if the policy is surrendered at the end of Year 1 (if a policyholder
backs out from a life insurance policy, he gets surrender benefits)

% 10 = i (well use the same interest rate for simple calculation)
% 5 =
d
x
q (death rate)
% 10 =
w
x
q (lapse rate, w=withdrawal)

Assume the lapse occurs at the end of the year.

Calculate the benefit reserve and the asset share at the end of Year 1.

Solution

Benefit reserve ignoring lapse and expenses and using the benefit premium

Policy Year 1
Time 0 1
# of insured 1 0.95
Benefit premium collected $80
# of deaths 0.05
Death benefit paid 0.05(1,000)=$50
Accumulated profit @ 1 = t : 80(1.1)-50=38
# of survivors @ 1 = t : 0.95
Benefit reserve @ 1 = t : 38/0.95=$40

Page 170 of 285 Deeper Understanding: Exam M November 7, 2006 2006 Yufeng Guo
http:/ / www.guo.coursehost.com

Asset share considering lapse and expenses and using the contract premium

Policy Year 1
Time 0 1
# of insured 1
( )( )
1 1
d w
x x
q q
=(1-5%)(1-10%)=0.855
Contract Premium collected $100
# of deaths 0.05
Death benefits paid 0.05(1,000)=$50
Acquisition Expense 40%(100)=$40
Lapse handling expense
( ) 20 20 .1 $2
w
x
q = =
Surrender benefits paid
( ) 100 100 .1 $10
w
x
q = =
Accumulated profit @ 1 = t : (100-40)(1.1)-50-2-10=$4
# of survivors @ 1 = t : 0.855

Asset share @ 1 = t : $4 / 0.855=$4.68

Page 171 of 285 Deeper Understanding: Exam M November 7, 2006 2006 Yufeng Guo
http:/ / www.guo.coursehost.com

Chapter 10 Expense loaded premium

Insurance companies cant make zero profit by charging benefit premiums as indicated
by the equivalence principle. Insurers need to charge more than benefit premiums to pay
for operating expenses. In addition, insurers need to earn a profit so they can grow.

Ill use an example to explain how to calculate expense loaded premium. If you
understand this example, you should be ready to be tested about expense loaded
premiums on Exam M.

Previously tested problem: #4, May 2005

Problem 1
Insurance
Fully discrete 30 pay whole life issued to ( ) 35
Death benefit $1,000
Acquisition expense at issue $20 per policy
Maintenance expense at the
beginning of each policy year
$4 per policy year
Premium collection expense 10% of each years expense-loaded premium
Mortality Illustrative Life Table
Interest rate 6%
Calculate
Expense-loaded premium (gross premium)
Identify each component of the expense-loaded premium

Solution

Step 1 Identify all cash flows

Age 35 36 37 64 65
t 0 1 2 34 35
Gross
premium
$P P P P
Acquisition
expense
$20
Maintenance
expense
$4 4 4 .. 4 4 4 4
Premium
collection
expense
0.1P 0.1P 0.1P 0.1P
PV of death
benefits
35
1, 000A
Page 172 of 285 Deeper Understanding: Exam M November 7, 2006 2006 Yufeng Guo
http:/ / www.guo.coursehost.com

Step 2 PV of gross premiums = PV of death benefits + PV of all expenses.

PV of gross premiums:
35:30
Pa
PV of death benefit:
35
1, 000A
PV of acquisition expense: 20
PV of maintenance expense:
35
4a
PV of premium collection expense:
35:30
0.1Pa
35 35
35:30 35:30
1, 000 20 4 0.1 Pa A a Pa = + + +
35 35
35:30
1, 000 20 4
0.9
A a
P
a
+ +
=


From the Illustrative Life Table,

35
1, 000 128.72 A =
35
15.3926 a =
35 30 35 65
35:30
a a E a =
65
9.8969 a =
30 30 30 65
30 35 30 35
35
7, 533, 964
1.06 0.1392408
9, 420, 657
l
E v p v
l

= = = =
35 30 35 65
35:30
15.3926 0.1392408 9.8969 14.01455 a a E a = = =
( )
( )
35 35
35:30
1, 000 20 4 128.72 20 4 15.3926
16.672
0.9 0.9 14.01455
A a
P
a
+ + + +
= = =


Step 3 Identify each component of the gross premium

35 35
35:30 35:30
1, 000 20 4 0.1 Pa A a Pa = + + +
Page 173 of 285 Deeper Understanding: Exam M November 7, 2006 2006 Yufeng Guo
http:/ / www.guo.coursehost.com

35 35
35:30
35:30
1, 000 20 4 0.1 A a Pa
P
a
+ + +
=


35
35
35:30 35:30 35:30 premium collection
expense
benefit premium acquisition maintenance
expense premium expense premium
4
1, 000 20
0.1
a
A
P
a a a
= + + +



premium

35
35:30
benefit premium
1, 000 128.72
9.185
14.01455
A
a
= =


35:30
acquisition
expense premium
20 20
1.427
14.01455 a
= =


35
35:30
maintenance
expense premium
4
4 15.3926
4.393
14.01455
a
a

= =


premium collection
expense premium
0.1 0.1 16.672 1.667 P = =


Total gross premium: 9.185+1.427+4.393+1.667=16.672

In this example, we assume that the insurer makes zero profit. If the insurers profit goal
is to earn 5% of the gross premiums collected, then we need to write the equation as:

PV of gross premiums = PV of death benefit + PV of all expenses + PV of profits

PV of gross premiums:
35:30
Pa
PV of the profits:
( )
35:30
5% Pa
PV of death benefit:
35
1, 000A
Page 174 of 285 Deeper Understanding: Exam M November 7, 2006 2006 Yufeng Guo
http:/ / www.guo.coursehost.com

PV of acquisition expense: 20
PV of maintenance expense:
35
4a
PV of premium collection expense:
35:30
0.1Pa
( ) 35 35
35:30 35:30 35:30
1, 000 20 4 0.1 5% Pa A a Pa Pa = + + + +
35 35
35:30
1, 000 20 4
0.85
A a
P
a
+ +
=


Premium components are calculated as follows:

( ) 35 35
35:30 35:30 35:30
1, 000 20 4 0.1 5% Pa A a Pa Pa = + + + +
35
35
35:30 35:30 35:30 premium collection
expens
benefit premium acquisition maintenance
expense premium expense premium
4
1, 000 20
0.1
a
A
P P
a a a
= + + +



profit loading
e premium
5% P +

Page 175 of 285 Deeper Understanding: Exam M November 7, 2006 2006 Yufeng Guo
http:/ / www.guo.coursehost.com

Chapter 11 Multiple decrement model

Examples of multiple decrements

Example #1 - Membership of a pension plan.

Corporations in the United States and many other countries offer pension benefits to their
employees. Each year a company contributes a certain amount of money to an
employees pension account. When an employee retires, he gets money from his pension
plan on a regular basis (such as annually or monthly). Each year, a company that offers
pension benefits needs to figure out how much money it needs to contribute to how many
employees. The number of employees for whom the company needs to contribute money
annually is affected by the following factors:

Death. If an employee dies, then no more contributions are made. Some
companies pay a death benefit to a deceased employees family; other companies
dont pay anything to a deceased employees family. Whatever the case, the
company doesnt need to contribute more money to a deceased employees
pension plan.

Changing jobs. Skilled employees often shop around for better paying jobs. If an
employee leaves his current company for a better job elsewhere, then the current
company doesnt need to contribute any more money to this employees pension
account. Generally, if an employee has worked in a company for a certain number
of years (such as five years) and then leaves for a new job elsewhere, he can keep
his pension benefits (called vested) in his current employer even though he
wont work there any more. However, the current employer wont contribute any
more money to this employees pension plan.

Disability. If an employee becomes disabled and hence leaves the company for
good, he may lose his pension benefits at some companies or be able to keep his
pension benefits at other companies. Either way, the employer wont make more
contributions to his pension account.

Retirement. If an employee retires, he starts receiving benefits and no additional
contributions will be made to his pension account by his employer.

In this example, the membership of pension plans is subject to the following four
decrements:
Death
Changing jobs
Disability
Retirement
Page 176 of 285 Deeper Understanding: Exam M November 7, 2006 2006 Yufeng Guo
http:/ / www.guo.coursehost.com

Pension actuaries need to set up a multiple decrement model to predict the continuous
change of membership and determine how much an employer needs to contribute to the
pension plan on an ongoing basis.

Example 2 Individual insurance

A life insurance contract terminates if a policyholder dies or surrenders his policy. If a
policyholder dies, the insurance company needs to pay a death benefit. If a policyholder
surrenders his policy, the insurance company needs to pay the surrender cash value to the
policyholder.

A life insurance policy is subject to two decrements:
Death
Surrender

To determine premium rates, actuaries need to set up a multiple decrement model to
project the death rate and surrender rate over the lifetime of a policy.

Building a multiple decrement model

If theres only one decrement (such as death), we use one random variable ( ) T x to keep
track of the future lifetime of someone aged ( ) x . Now we have multiple decrements. One
variable alone, ( ) T x , is not enough. We need an additional variable J to keep track of
the various decrements.

Consider someone aged ( ) x . His future lifetime (i.e. number of years before hes finally
hit by a cause of decrement) is ( ) T x . At any moment, ( ) x is subject to one of m
mutually exclusive causes of decrement numbered from 1 to m. The joint pdf
(probability density function) of T and J is:

( ) ( )
,
, ,
T J
f t j dt P t T t dt J j = < s + = where 1, 2,..., j m =
This represents the probability that ( ) x will be hit by cause j during a very short
interval ( ] , t t dt + .
The probability that ( ) x will be hit by cause j before time t is:

( )
( ) ( )
,
0
, ,
t
j
t x T J
q f s j ds P T t J j = = s =
}
where 1, 2,..., j m =
Page 177 of 285 Deeper Understanding: Exam M November 7, 2006 2006 Yufeng Guo
http:/ / www.guo.coursehost.com

Marginal distribution of J :
( ) ( ) ( )
,
0
,
J T J
f j f s j ds P J j
+
= = =
}
This represents the probability that ( ) x will be hit by cause j during his life time.

Marginal distribution of T :
( ) ( )
,
1
,
m
T T J
j
f t f s j
=
=

This represents the probability that ( ) x will be hit by any cause of decrement during
( ] , t t dt + .
Most likely, you wont be directly tested on the marginal distribution of J or T , but its
good to know their algebraic expressions.

Up to this point we considered one individual cause of decrement. Now lets consider the
probability of being hit by any one of the causes of decrement.

The probability that ( ) x is finally hit by any of the m mutually exclusive causes of
decrement during the time interval [ ] 0, t is:

( )
( ) ( ) ( )
0
t
t x T T
q P T t F t f s ds
t
= s = =
}
The probability that ( ) x has not been hit by any of the m mutually exclusive causes of
decrement during the time interval [ ] 0, t is:

( )
( )
( )
1
t x t x
p P T t q
t t
= > =
Conceptually, we can think of ( ) t as one big cause of decrement produced by combining
all the m mutually exclusive causes of decrement into one cause. So ( ) t represents a
single decrement; everything you learned from Actuarial Mathematics Chapter 1 about a
single decrement table applies to ( ) t .
For example,
Page 178 of 285 Deeper Understanding: Exam M November 7, 2006 2006 Yufeng Guo
http:/ / www.guo.coursehost.com

( )
( ) ( )
T t x t x
d d
f t q p
dt dt
t t
= =
( )
( )
( )
( )
T
x
t x
f t
t
p
t
t
=
( ) ( )
( )
0
exp
t
t x x
p s ds
t t

(
=
(

}
( ) ( ) ( )
( )
0
exp
t
t x s x x
q p s ds
t t t

(
=
(

}
The total force of decrement:

( )
( )
( )
( )
( )
( ) ( )
1
ln
1
T
x t x t x
T t x
f t
d d
t p p
F t dt dt p
t t t
t

(
= = =

The force of decrement due to cause j :


( )
( )
( )
( )
( )
( )
, ,
, ,
1
j T J T J
x
T t x
f t j f t j
t
F t p
t
= =

Relationship between
( )
( )
x
t
t
and
( )
( )
j
x
t :
( )
( )
( )
( )
( )
( )
( )
( )
( )
( )
1 2
1
...
m
j m
x x x x x
j
t t t t t
t

=
= = + + +

Probability of decrement due to cause j under the multiple decrement table:



( )
[ ]
( ) ( )
( )
0
,
t
j j
t x s x x
q P T t J j p s ds
t
= s = =
}
The associated single decrement table

Under a multiple decrement table, mmutually exclusive causes of decrement operate
continuously. For the purpose of building a multiple decrement table, however, often we
Page 179 of 285 Deeper Understanding: Exam M November 7, 2006 2006 Yufeng Guo
http:/ / www.guo.coursehost.com

want to isolate each cause of decrement by silencing all the other decrements. This gives
us an associated single decrement table.

In our pension membership example, the multiple decrement table has four causes:

Death
Changing jobs
Disability
Retirement.

For each of these four causes, we can construct an associated single decrement table. For
example, to build a single decrement table for the cause of death, we silence the other
three causes (changing jobs, disability, and retirement). The only decrement left is death.
So under the associated single decrement table for death, a person either dies or lives on.
The only way his membership can terminate is if he dies. He cant change his job; he has
zero chance of becoming disabled; he wont retire (remember that we silenced the other
three decrements).

Similarly, we can build an associated single decrement table for changing jobs. Here
every existing member of a pension plan either changes jobs or continues working for his
current employer. The only way his membership terminates is when he changes his job.
The other three causes of decrement (death, disability, and retirement) are silenced and
dont exist anymore.

So we follow this line of thinking and build four separate associated single decrement
tables respectively for death, changing jobs, disability, and retirement.

Please note that in reality it is impossible to have only one decrement at work and have
all other decrements removed. For example, in our pension membership example, we
really cant isolate death by silencing the remaining three causes of decrement.

However, the concept of an associated single decrement table is still useful. For example,
we can use a general populations mortality table to approximate the single decrement
table for death. By using the concept of an associated single decrement table, we find a
way to construct a multiple decrement table.

The link between a multiple decrement table and its associated single decrement tables is:

Page 180 of 285 Deeper Understanding: Exam M November 7, 2006 2006 Yufeng Guo
http:/ / www.guo.coursehost.com

( ) ( ) ( ) ( ) ' 1 ' 2 '
...
m
t x t x t x t x
p p p p
t
=
To remember the above equation, remember the phrase: To survive all decrements, you
must survive each single decrement.
Imagine you release each decrement separately. The first time, you release decrement #1
and silence all the other decrements. So only decrement #1 goes to work and hits some
people. Next, you put decrement #2 to work and silence all the other decrements.
Decrement #2 goes to work and hits some people. And this process goes on. For someone
to survive, he must not be hit by any single decrement.
The equation
( ) ( ) ( ) ( ) ' 1 ' 2 '
...
m
t x t x t x t x
p p p p
t
= stands true because we have

( )
( )
( )
( )
( )
( )
( )
( )
( )
( )
1 2
1
...
m
j m
x x x x x
j
t t t t t
t

=
= = + + +

( )
( )
( )
( )
( )
( )
( )
( )
1 2
0 0
exp exp ...
t t
m
x x x x
s ds s s s ds
t

( (

= + + +
` ( (
)
} }

( )
( )
( )
( )
( )
( )
1 2
0 0 0
exp exp ... exp
t t t
m
x x x
s ds s ds s ds
( ( (
=
( ( (

} } }

( )
( )
( ) '
0
exp
t
x x
s ds p
t t

(
=
(

}
( )
( )
( ) 1 ' 1
0
exp
t
x x
s ds p
(
=
(

}
( )
( )
( ) 2 ' 2
0
exp
t
x x
s ds p
(
=
(

}
,
( ) ( ) ( ) ( ) ' 1 ' 2 '
...
m
t x t x t x t x
p p p p
t
=
Page 181 of 285 Deeper Understanding: Exam M November 7, 2006 2006 Yufeng Guo
http:/ / www.guo.coursehost.com

Please understand the difference between
( ) j
t x
q and
( ) ' j
t x
q .
( ) j
t x
q represents the probability that someone gets hit by cause j , where j is one of the
m mutually exclusive causes of decrements.
( ) ' j
t x
q represents the probability that someone gets hit by the cause j , where j is the only
cause of decrement.
You should know that
( ) ( ) ' j j
t x t x
q q < . Why? Under the single ( ) ' j table, the cause j keeps
hitting people without being interrupted by any other causes of decrement. As a result,
more people are hit by cause j under the single decrement table ( ) ' j . In contrast, under
the multiple decrement table, we have m mutually exclusive decrements taking turns in
hitting people. At any point in time, cause j and the other ( ) 1 m causes of decrement
compete with each other for a chance to hit somebody. Remember that these m causes of
decrement are mutually exclusive and that at any give point in time only one cause can
work (one person cant get hit by two causes of decrement). As a result, the number of
people hit by cause j is always smaller than the number of people hit by cause j under
the single ( ) ' j table.

Constant Force of Mortality or UDD under the Multiple Decrement
Table

Understand the intuition behind two difficult formulas:
( )
( )
( )
( )
'
ln
ln
j
j
s x
s x s x
s x
p
q q
p
t
t
=
( ) ( )
( )
( )
'
'
j
s x
s x
q
j
q
s x s x
p p
t t
(
=


If the force of each decrement is constant over the age ( ) , 1 x x + , we should intuitively
know that the number of people hit by cause j during age ( ) , x x s + where 0 1 s < s
Page 182 of 285 Deeper Understanding: Exam M November 7, 2006 2006 Yufeng Guo
http:/ / www.guo.coursehost.com

should be proportional to
( ) j
x
. The bigger the force of decrement
( ) j
x
, the greater the
number of people hit by cause j . This leads to

( )
( )
( )
( )
j
j
x
s x s x
x
q q
t
t

= ,
( ) ( )
( )
( )
'
'
j
s x
s x
q
j
q
s x s x
p p
t t
(
=


But what about UDD and the multiple decrement table? Why should
( )
( )
( )
( )
'
ln
ln
j
j
s x
s x s x
s x
p
q q
p
t
t
=
hold?

Lets think things through. We have m mutual exclusive causes of decrement. Each
decrement is UDD under the multiple decrement table. So the total decrement is also
UDD under the multiple decrement table. Then by intuition we know that the number of
people who are hit by the cause of decrement ( ) j in each year should be a constant
fraction of the total number of people who are hit by any of the mmutual exclusive
causes of decrement.

For example, say we have three decrements: death, disability, and retirement. If any of
these events occurs in a persons life, this persons membership to a pension plan
terminates immediately. If death, disability, and retirement are UDD under the multiple
decrement table, then each of the three decrements continuously hits people and
terminates their membership at a constant speed.

For example, death is hitting people at a constant speed of
1
k death(s) per 30 days (so
1
k
people die every 30 days); disability may hit people at a constant speed of
2
k disability
cases per 30 days (so
2
k people become disabled every 30 days); and retirement is
continuously hitting people at a constant speed of
3
k per 30 days (so
3
k people retire
every 30 days).

Then during the time interval [ ] 0, s where the unit is one month, we have:

The # of people hit by deaths is
1
k s
The # of people hit by disability is
2
k s
The # of people hit by retirement is
3
k s
Total # of people hit by any of the 3 decrements is ( )
1 2 3
k k k s + +
If we start off with
( )
x
l
t
people in the pension plan, and if 1,2, and 3 represents death,
disability, and retirement respectively, then

Page 183 of 285 Deeper Understanding: Exam M November 7, 2006 2006 Yufeng Guo
http:/ / www.guo.coursehost.com

( )
[ ]
( )
1
1
# of members who died during 0,
# of members in the pension plan at the beginning of the year
s x
x
s
k s
q
l
t
= =
( )
[ ]
( )
2
2
# of members disabled during 0,
# of members in the pension plan at the beginning of the year
s x
x
s
k s
q
l
t
= =
( )
[ ] # of members died, disabled, or retired during 0,
# of members in the pension plan at the beginning of the year
s x
s
q
t
=
( )
1 2 3
x
k k k
s
l
t
+ +
=
( )
( )
( )
1
1 1
1 2 3 1 2 3
constant one
s x
s x
q sk k
s k k k k k k q
t
= = =
+ + + +

( )
( )
( )
2
2 2
1 2 3 1 2 3
constant two
s x
s x
q sk k
s k k k k k k q
t
= = =
+ + + +

( )
( )
( )
3
3 3
1 2 3 1 2 3
constant three
s x
s x
q sk k
s k k k k k k q
t
= = =
+ + + +

Now you see that
( )
( )
j
s x
s x
q
q
t
should be a constant. Then intuitively we know that

1
k is proportional to
( ) 1
x
, the average force of mortality due to cause 1 during [ ] 0, s ,
2
k is proportional to
( ) 2
x
, the average force of mortality due to cause 2 during [ ] 0, s ,
3
k is proportional to
( ) 3
x
, the average force of mortality due to cause 3 during [ ] 0, s
Now it should be clear that

( )
( )
( ) ( ) ( )
( )
( )
( )
( )
1 2
1
...
j j
j
x x
s x s x s x m
m
i
x x x
x
i
q q q
t t



=
= =
+ + +

Next, we know that
Page 184 of 285 Deeper Understanding: Exam M November 7, 2006 2006 Yufeng Guo
http:/ / www.guo.coursehost.com

( ) ( )
( )
( ) ( ) ' '
1
exp ln
j j j j
s x x x s x
p s p
s
= =
This is from the general formula
( )
exp
t x
p t = .
Also, we have
( ) ( ) ( ) ( ) ( ) ( ) ( )
( )
' ' 1 ' 2 ' 1 2
... exp ...
m m
s x s x s x s x x x x
p p p p s
t

(
= = + + +
(

( ) ( ) ( ) ( ) 1 2 '
1
... ln
m
x x x s x
p
s
t
+ + + =
Finally, we have

( )
( )
( )
( )
( )
( )
'
1
ln
ln
j j j
s x x s x
m
i
s x s x
x
i
q p
q p
t t

=
= =

The above proof is not as rigorous as the proof in the textbook, but it gives us a good feel
for the logic in the formula.

Next, if you know
( )
( )
( )
( )
'
ln
ln
j
j
s x
s x s x
s x
p
q q
p
t
t
= , you shouldnt have trouble deriving
( ) ( )
( )
( )
'
'
j
s x
s x
q
j
q
s x s x
p p
t t
(
=

.
Please note that if each decrement has UDD in the multiple decrement table, it follows
that the total force of decrement also follows UDD.

Please also note that in the exam, you can use the direct formulas

( )
( )
( )
( )
'
ln
ln
j
j
s x
s x s x
s x
p
q q
p
t
t
= ,
( ) ( )
( )
( )
'
'
j
s x
s x
q
j
q
s x s x
p p
t t
(
=


Once you understand the intuition behind these formulas, you shouldnt have trouble
memorizing them.

Page 185 of 285 Deeper Understanding: Exam M November 7, 2006 2006 Yufeng Guo
http:/ / www.guo.coursehost.com

Construct a multiple decrement table

Here we assume that each associated single decrement is uniformly distributed over the
year ( ) , 1 x x + .
Formulas you need to memorize:

For two decrements:
( ) ( ) ( ) 1 ' 1 ' 2
1
1
2
x x x
q q q
(
=
(

( ) ( ) ( ) 2 ' 2 ' 1
1
1
2
x x x
q q q
(
=
(

The derivation is really simple. To see why
( ) ( ) ( ) 1 ' 1 ' 2
1
1
2
x x x
q q q
(
=
(

, notice

( ) ( )
( )
( ) ( )
( )
( ) ( )
1 1
1 1 1 ' 1 ' 2
0 0
x x t x x t x t x
q t p dt t p p dt
t
= =
} }

Under UDD,

( )
( )
( ) ( )
( )
( ) 1 ' 1 ' 1 ' 1
x t x x
t p f t q = = (a constant)

( ) ( ) ' 2 ' 1
1
t x x
p t q =
( ) ( ) ( ) ( ) ( )
1
1 ' 1 ' 2 ' 1 ' 2
0
1
1 1
2
x x x x x
q q t q dt q q
(
(
= =
(

}
On the exam, however, you dont want to do this integration, so you need to memorize
the formula.

How to memorize formulas for two decrements

First, as said before,
( ) ( ) 1 ' 1
x x
q q < . So it makes sense that we multiply
( ) ' 1
x
q with a factor
( ) ' 2
1
1
2
x
q . Note that
( ) ' 2
1
1 1
2
x
q < . But why do we apply a factor of
( ) ' 2
1
1
2
x
q ?
Page 186 of 285 Deeper Understanding: Exam M November 7, 2006 2006 Yufeng Guo
http:/ / www.guo.coursehost.com

( ) ' 2
1
1
2
x
q is the average (or expected) survival rate during ( ) , 1 x x + under UDD in the
associated single decrement table ( ) ' 2 . Here we have two causes of decrement
competing for a chance to hit somebody. A person can only be hit by cause #1 if he was
not first hit by cause #2. In other words, if cause #2 already hit someone, this person
permanently leaves the membership and can never be hit by cause #1. Only the survivors
of cause #2 can potentially be hit by cause #1. Under UDD, during ( ) , 1 x x + , the average
survival rate for cause #2 is
( ) ' 2
1
1
2
x
q . As a result,

( ) ( ) ( ) 1 ' 1 ' 2
1
1
2
x x x
q q q
(
=
(

Similarly,
( ) ( ) ( ) 2 ' 2 ' 1
1
1
2
x x x
q q q
(
=
(

.
For three decrements:
( ) ( ) ( ) ( ) ( ) ( ) 1 ' 1 ' 2 ' 3 ' 2 ' 3
1 1
1
2 3
x x x x x x
q q q q q q

(
= + +
`

)
( ) ( ) ( ) ( ) ( ) ( ) 2 ' 2 ' 1 ' 3 ' 1 ' 3
1 1
1
2 3
x x x x x x
q q q q q q

(
= + +
`

)
( ) ( ) ( ) ( ) ( ) ( ) 3 ' 3 ' 1 ' 2 ' 1 ' 2
1 1
1
2 3
x x x x x x
q q q q q q

(
= + +
`

)
I recommend that you dont bother memorizing these difficult formulas. Just understand
how to derive them.
The derivation is not too complex. For example,

( ) ( )
( )
( ) ( )
( )
( ) ( ) ( ) ( ) ( ) ( )
1 1 1
1 1 1 ' 1 ' 2 ' 3 ' 1 ' 2 ' 3
0 0 0
x x t x x t x t x t x x t x t x
q t p dt t p p p dt q p p dt
t
= = =
} } }

( ) ( ) ' 2 ' 2
1
t x x
p t q = ,
( ) ( ) ' 3 ' 3
1
t x x
p t q =
Page 187 of 285 Deeper Understanding: Exam M November 7, 2006 2006 Yufeng Guo
http:/ / www.guo.coursehost.com

( ) ( ) ( ) ( )
1
1 ' 1 ' 2 ' 3
0
1 1
x x x x
q q t q t q dt
( (
=

}
Here
( ) ' 1
x
q ,
( ) ' 2
x
q , and
( ) ' 3
x
q are constants. If you do the integration, you should get:

( ) ( ) ( ) ( ) ( ) ( ) 1 ' 1 ' 2 ' 3 ' 2 ' 3
1 1
1
2 3
x x x x x x
q q q q q q

(
= + +
`

)
Common problems and model solutions

Type 1 Find
( )
f T J j = and
( )
E T J j =
Example

Given
( )
( )
j
x
t j = , where 1, 2, 3, 4 j =
Calculate
( )
2 f T J =
( )
2 E T J = .
Solution

This problem looks scary but is simple. Well use the general formula:

( ) ( ) E Y y f y dy
+

=
}
If we know
( )
1
2
T J
f T J
=
= , we calculate the mean as:

( ) ( )
2
0
2 2
T J
E T J t f T J dt
+
=
= = =
}
( )
( )
( )
( )
( )
0
, 2 , 2
2
2
, 2
f T j f T j
f T J
P j
f T j dt
+
= =
= = =
=
=
}
( ) ( )
( )
( )
( ) 2 '
, 2 , 2
x t x
f T j P T t j t p
t
= = = = =
Page 188 of 285 Deeper Understanding: Exam M November 7, 2006 2006 Yufeng Guo
http:/ / www.guo.coursehost.com

( ) ( )
( )
( )
( )
( )
( )
( )
( )
' 1 2 3 4
0
exp
t
t x x x x x
p s s s s ds
t


(
= + + +
`

)
}
[ ]
10
0
exp 1 2 3 4
t
t
ds e


= + + + =
`
)
}
( )
( )
( )
( ) 2 ' 10
, 2 2
t
x t x
f T j t p e
t


= = = ( )
10
0 0
1
, 2 2
5
t
f T j dt e dt
+ +

= = =
} }

( )
( )
( )
10
10
0
, 2
2
2 10
1
, 2
5
t
t
f T j
e
f T J e
f T j dt

+
=
= = = =
=
}
( ) ( ) ( )
10
2
0 0
2 2 10 0.1
t
T J
E T J t f T J dt t e dt
+ +

=
= = = = =
} }

Please note that
10
10
t
e

is an exponential distribution with a mean of 0.1.



General formulas:

For 1, 2,..., j m = , if
( )
( )
1
1 x
t k = ,
( )
( )
2
2 x
t k = , .,
( )
( )
m
x m
t k =
where
1 2
, ,....,
m
k k k are constants
( )
( )
( )
( ) ( )
1 2
' ...
,
m
t j
x t x j
k k k
f T J j t p k e
t

+ + +
= = =
( )
( )
1 2
...
1 2 0
...
m
k k k s j
j
m
k
f J j k e ds
k k k
+
+ + +
= = =
+ + +
}
( )
( )
( )
( )
( )
( )
1 2
1 2
...
...
1 2
1 2
,
...
...
m
m
k k k t
k k k t j
m
j
m
k e f T J j
f T J j k k k e
k
f J j
k k k
+ + +
+ + +
=
= = = = + + +
=
+ + +

( ) ( ) ( )
( )
1 2
...
1 2
0 0
...
m
k k k t
m T J j
E T J j t f T J j dt t k k k e dt
+ +
+ + +
=
= = = = + + +
} }

1 2
1
...
m
k k k
=
+ + +
Page 189 of 285 Deeper Understanding: Exam M November 7, 2006 2006 Yufeng Guo
http:/ / www.guo.coursehost.com

Please note that ( )
( )
1 2
...
1 2
1 2 0
1
...
...
m
k k k t
m
m
t k k k e dt
k k k
+
+ + +
+ + + =
+ + +
}
. If we set
1 2
...
m
k k k + + + = , then

( )
( )
1 2
...
1 2
0 0
...
m
k k k t t
m
t k k k e dt t e dt

+ +
+ + +
+ + + =
} }

t
e


is an exponential distribution with a mean of
1

. As a result,

( )
( )
( )
1 2
...
1 2
1 2 0 0
1 1
...
...
m
k k k t t
m
m
t k k k e dt t e dt E T
k k k

+ +
+ + +
+ + + = = = =
+ + +
} }

Type 2
( )
f J j T = and ( )
T
f t
Example 1

Given: For 0 120 t s <
( ) 1
1
120
x
t
=

( ) 2
120
x
k
t
=

( )
1
1
1
5
J T
f J T
=
= =
Calculate ( ) 100
T
f
Solution

( )
( )
( )
( )
( )
, ,
1
1, 1,
1
J T J T
J T
f J T t f J T t
f J T
f t P T t
=
= = = =
= = =
=
( )
( ) ( ) 1
,
1,
J T x t x
f J T t p
t
= = = , where
( ) ( )
( )
( )
( )
1 2
0
exp
t
t x x x
p s s ds
t


(
= +
`

)
}
( ) ( )
( )
( )
( )
( )
( ) 1 2
x x t x
f t P T t t t p
t

(
= = = +


Page 190 of 285 Deeper Understanding: Exam M November 7, 2006 2006 Yufeng Guo
http:/ / www.guo.coursehost.com

( )
( )
( )
( ) ( )
( )
( )
( )
( )
( )
( )
( )
( )
( )
( )
1 1
,
1 1 2 1 2 '
1,
1
J T
x t x x
J T
x x x x t x
f J T t
p
f J T
P T t t t t t p
t
t


=
= =
= = = =
= ( + +


Because
( )
1
1
1
5
J T
f J T
=
= = , we have

( )
( )
( )
( )
( )
1
1 2
1 1 1
, 4
5 1 5
x
x x
k
k t t


= = =
+ +
( )
( )
0
5 5 5
exp
120 120 120
t
t x
f t p ds
t t t
t

= =
`

)
}
To avoid doing the integration
0
5
exp
120
t
ds
t

)
}
, you need to memorize the following
shortcuts about the generalized De Moivres Law:

( ) 1
t x x
t
p t
t
o
o

= =
| |
= =
|

\ .

In this case, in the ( ) t table, the total force of mortality is

( )
( )
( )
( )
1 2
5
120
x x
t t
t
+ =

Applying the generalized DeMoivres Law, we have:



( )
5
1
120
t x
t
p
t | |
=
|
\ .
( )
( )
5
5 5
1
120 120 120
t x
t
f t p
t t
t | |
= =
|

\ .
( )
5
5 100
100 1 0.003215%
120 100 120
f t
| |
= = =
|

\ .

Page 191 of 285 Deeper Understanding: Exam M November 7, 2006 2006 Yufeng Guo
http:/ / www.guo.coursehost.com

General formula:

For m causes of decrement,
( )
( )
( )
( )
( )
( )
( )
( )
( )
1 2
...
j
x
J j T m
x x x
t
f J j T t
t t t


=
= = =
+ + +

Example 2

Given:
( )
( )
1
100
x
t =
( )
( )
2
200
x
t =
( )
( )
3
300
x
t =
Find
( )
1
1
J T
f J T t
=
= =
Solution

( )
( )
( )
( )
( )
( )
( )
( )
( )
2
2 1 2 3
200
2 0.5
100 200 300
x
J T
x x x
t
f J T t
t t t


=
= = = = =
+ + + +

Type 3 UDD in the multiple decrement table

Example 1 (SOA)

For a double decrement table, you are given:

x
( )
x
l
t ( ) 1
x
d
( ) 2
x
d
35 1,000 39 41
36 69
37 828
Assume that each decrement is uniformly distributed over each year of age in the double
decrement table. Calculate the absolute rate of decrement due to cause 1 for age 36.

Page 192 of 285 Deeper Understanding: Exam M November 7, 2006 2006 Yufeng Guo
http:/ / www.guo.coursehost.com

Solution

Key formulas to solve this problem:

( ) ( ) ' 1 ' 1
36 36
1 q p = ,
( ) ( )
( )
( )
' 1
36
36
' 1
36 36
q
q
p p
t t
(
=

,
( )
( )
( )
1
1
36
36
36
d
q
l
t
= ,
( )
( )
( )
36
36
36
d
q
l
t
t
t
= ,
( ) ( ) ( ) ( ) 1 2
36 35 35 35
l l d d
t t
=
( ) ( ) ( ) ( ) 1 2
36 35 35 35
1, 000 39 41 920 l l d d
t t
= = =
( ) ( ) ( ) ( ) 1 2
37 36 36 36
l l d d
t t
=
( ) ( ) 1 1
36 36
828 920 69, 23 d d = =
( )
( )
( )
( )
1 1
36 36
36 36
23 1
23 69 4
q d
q d
t t
= = =
+
( )
( )
( )
37
36
36
828
920
l
p
l
t
t
t
= =
( ) ( ) ( )
( )
( )
' 1
36
36
1
4
' 1 ' 1
36 36 36
828
1 1 1 2.6%
920
q
q
q p p
t t | |
(
= = = =
|

\ .

Type 4 UDD in associated single decrement tables, 2 decrements

Example 1

Two decrements, withdrawal and death, are each uniformly distributed in their respective
associated single decrement tables.

x
( )
x
l
t ( ) w
x
d
( ) d
x
d
( ) ' d
x
q
30 10,000 A 50
31 9,500 B C 0.5%
32 9,000
Calculate , , A B C .
Page 193 of 285 Deeper Understanding: Exam M November 7, 2006 2006 Yufeng Guo
http:/ / www.guo.coursehost.com

Solution

( ) ( ) ( )
30 30 30
w d
A d d d
t
= =
( ) ( ) ( )
30 30 31
10, 000 9, 500 500 d l l
t t t
= = =
( ) ( ) ( )
30 30 30
500 50 450
w d
A d d d
t
= = = =
( )
( )
( )
32
31
31
9, 000
9, 500
l
p
l
t
t
t
= =
( ) ( ) ( ) ( ) ( ) ( )
[ ]
' ' ' ' '
31 31 31 31 31 31
1 1 1 0.5%
w d w d w
p p p p q q
t
( (
= = =


( )
[ ]
( ) ' '
31 31
9, 000
1 1 0.5% , 4.7871%
9, 500
w w
q q x
(
= = c


( ) ( ) ( ) ' '
31 31 31
1 1
1 4.7871% 1 0.5% 4.7751%
2 2
w w d
q q q
( | |
= = =
|
(
\ .

( ) ( ) ( ) ' '
31 31 31
1 1
1 0.5% 1 4.7871% 0.48803%
2 2
d d w
q q q
( | |
= = =
|
(
\ .

( ) ( ) ( )
31 31 31
9, 500 4.7751% 453.63
w w
B d l q
t
= = = =
( ) ( ) ( )
31 31 31
9, 500 0.48803% 46.36
d d
C d l q
t
= = = =
Exam 2 (SOA)

Two decrements are considered: mortality and withdrawal.

( ) '
0.03
d
x
q =
( )
0.20
w
x
=
In the respective associated single-decrement tables, mortality is uniformly distributed
between consecutive integral ages and withdrawal has a constant force of decrement
between consecutive integral ages.

Calculate
( ) d
x
q
Page 194 of 285 Deeper Understanding: Exam M November 7, 2006 2006 Yufeng Guo
http:/ / www.guo.coursehost.com

Solution

( ) ( )
( )
( ) ( )
1
' '
0
d d d w
x x t x t x
q t p p dt =
}
Under UDD,
( )
( )
( ) ( ) ' ' d d d
x t x x
t p q = (a constant)

Constant force of mortality:

( )
( )
' 0.2
w
x
w t t
t x
p e e

= =
( ) ( ) ( )
1 0.2 0.2
' ' 0.2
0
1 1
0.03 2.719%
0.2 0.2
d d d t
x x x
e e
q q e dt q


= = = =
}
Type 5 UDD in associated single decrement tables, 3 decrements

Problem 1

Each decrement is uniformly distributed in its associated single decrement table between
integral ages.

( ) ' 1
1
10
x
q = ,
( ) ' 2
1
20
x
q = ,
( ) ' 3
1
30
x
q =
Calculate
( ) 1
x
q
Solution

( ) ( )
( )
( ) ( )
( )
( ) ( ) ( ) ( ) ( ) ( )
1 1 1
1 1 1 ' 1 ' 2 ' 3 ' 1 ' 2 ' 3
0 0 0
x x t x x t x t x t x x t x t x
q t p dt t p p p dt q p p dt
t
= = =
} } }

( ) ( ) ' 2 ' 2
1 1
20
t x x
t
p t q = = ,
( ) ( ) ' 3 ' 3
1 1
30
t x x
t
p t q = =
( )
1 1 2
1
0 0
1 1
1 1 1
10 20 30 10 12 600
x
t t t t
q dt dt
| |
| || |
= = +
| | |
\ .\ .
\ .
} }

1
2 3
0
1
10 24 1, 800
t t
t
(
= +
(

9.5889% =
Page 195 of 285 Deeper Understanding: Exam M November 7, 2006 2006 Yufeng Guo
http:/ / www.guo.coursehost.com

See that you dont need to memorize
( ) ( ) ( ) ( ) ( ) ( ) 1 ' 1 ' 2 ' 3 ' 2 ' 3
1 1
1
2 3
x x x x x x
q q q q q q

(
= + +
`

)
You can quickly do the integration during the exam.
Type 6 multiple decrements some continuous and some discrete

Example 1 (SOA)

Participant data as of 1/1/2003: 1,000 active participants, all exactly age 54.

Number of decrements: withdrawal (w), death (d), and retirement (r)

Timing of decrements:
Retirement occurs at the beginning of the year
withdrawal and death occur at the end of the year

Withdrawals and deaths are uniformly distributed between consecutive integral ages in
their respective single decrement tables.

x
( ) ' w
x
d
( ) ' d
x
d
( ) ' r
x
q
54 4% 0.44% 0%
55 0% 0.49% 15%
56 0% 0.55% 0%
There are no new entrants after 1/1/2003.

Calculate the expected number of deaths from active service during 2003 and 2004.

Solution

We start off with 1,000 active participants. So
( )
1, 000
x
l
t
=
In 2003, the retirement rate is zero. So we have two decrements.

( ) ( ) ( )
( )
' '
1 1
1 0.44% 1 4% 0.4312%
2 2
d d w
x x x
q q q
( (
= = =
( (


Page 196 of 285 Deeper Understanding: Exam M November 7, 2006 2006 Yufeng Guo
http:/ / www.guo.coursehost.com

( ) ( ) ( )
( ) 1, 000 0.4312% 4.312
d d
x x x
d l q
t
= = =
In 2004, there are three decrements. The decrement of retirement occurs only at the
beginning the year; death and withdrawal occur uniformly throughout the year.

First, lets calculate the starting population at 2004 (i.e. the population at the end of year
2003).

( ) ( ) ( ) ( ) ( ) ( ) ( ) ( ) ( ) ' ' ' '
1
1 1
d w d w
x x x x x x x x x
l l p l p p l q q
t t t t t
+
( (
= = =

[ ][ ] 1, 000 1 0.44% 1 4% 955.776 = =
Of these people, 15% will retire, leaving us the following number of active employees:

( ) 955.776 1 15% 812.4096 =
These people will go through two decrements throughout the year. Because death and
withdrawal are UDD, we have:

( ) ( ) ( )
( )
' '
1 1 1
1 1
1 0.49% 1 0% 0.49%
2 2
d d w
x x x
q q q
+ + +
( (
= = =
( (


( ) ( ) ( )
( ) 812.4096 0.49% 3.980807
d d
x x x
d l q
t
= = =
The total expected number of deaths during 2003 and 2004 is:

4.312 3.980807 8.2928 + =
Example 2

# of decrements = 2

In the single decrement table associated with cause( ) 1 ,
( ) ' 1
0.9
x
q = and the decrement is
uniformly distributed over age( ) , 1 x x + .
In the single decrement table associated with cause ( ) 2 ,
( ) ' 2
0.15
x
q = and the decrement
takes place at two points during the year:
( ) ' 2
1
5
x
q at
1
3
t = and
( ) ' 2
4
5
x
q at
2
3
t = .
Calculate
( ) 1
x
q and
( ) 2
x
q
Page 197 of 285 Deeper Understanding: Exam M November 7, 2006 2006 Yufeng Guo
http:/ / www.guo.coursehost.com

Solution

Method 1 (intuitive approach)

t 0 1
3
2
3
1
( ) ' 1
t x
p
1
( ) ' 1
1
1
3
x
q
( )
1
1 0.9
3
=
0.7 =
( ) ' 1
2
1
3
x
q
( )
2
1 0.9
3
=
0.4 =
( ) ' 1
1
x
q
1 0.9 =
0.1 =
Decrement ( ) 2
( ) ' 2
1
5
x
q
( )
1
0.15 3%
5
= =
( ) ' 2
4
5
x
q
( )
4
0.15 12%
5
= =
# of people hit by
cause ( ) 2
( ) 0.7 3% 2.1% = ( ) 0.4 12% 4.8% =
( ) 2
2.1% 4.8% 6.9%
x
q = + =
( ) ( ) ( ) 1 2
x x x
q q q
t
=
( ) ( ) ( ) ( ) ( ) ( )
[ ][ ]
' 1 ' 2 ' 1 ' 2
1 1 1 1 1 1 1 0.9 1 0.15 91.5%
x x x x x x
q p p p q q
t t
( (
= = = = =


( ) ( ) ( ) 1 2
91.5% 6.9% 84.6%
x x x
q q q
t
= = =
Method 2 (Formula-driven approach)

( ) ( )
( )
( ) ( )
( )
( ) ( )
1 1
1 1 1 ' 1 ' 2
0 0
x x t x x t x t x
q t p dt t p p dt
t
= =
} }

( )
( )
( ) ( )
( )
( ) 1 ' 1 ' 1 ' 1
x t x x
t p f t q = = (under UDD)

( ) ( ) ( ) ( ) ( )
1 1
1 ' 1 ' 2 ' 1 ' 2
0 0
x t x t x x t x
q q p dt q p dt = =
} }

Page 198 of 285 Deeper Understanding: Exam M November 7, 2006 2006 Yufeng Guo
http:/ / www.guo.coursehost.com

To find
( )
1
' 2
0
t x
p dt
}
, notice that
( ) ( ) ( )
( )
( )
' 2 ' 2 ' 2
' 2
1
1 0
3
1 1 1 2
1 1 1 0.15 0.97
5 5 3 3
2
1 1 0.15 .85 1
3
t x t x x
x
t
p q q t
q t

s <

= = = = s <

= = s <

( )
( ) ( )
1 2
1 1 3 3
' 2
1 2 0 0
3 3
1 1 1
0.97 0.85 0.97 0.85 94%
3 3 3
t x
p dt dt dt dt = + + = + + =
} } } }

( ) ( ) ( )
( )
1
1 ' 1 ' 2
0
0.9 0.94 84.6%
x x t x
q q p dt = = =
}
( ) ( ) ( ) ( ) ( ) ( )
[ ][ ]
' 1 ' 2 ' 1 ' 2
1 1 1 1 1 1 1 0.9 1 0.15 91.5%
x x x x x x
q p p p q q
t t
( (
= = = = =


( ) ( ) ( ) 2 1
91.5% 84.6% 6.9%
x x x
q q q
t
= = =
Example 3

Number of decrements = 3

In the single decrement table associated with cause ( ) 1 ,
( ) ' 1
0.9
x
q = and the decrement is
uniformly distributed over age( ) , 1 x x + .
In the single decrement table associated with cause ( ) 2 ,
( ) ' 2
0.271
x
q = and the force of
decrement is constant over age( ) , 1 x x + .
In the single decrement table associated with cause ( ) 3 ,
( ) ' 3
0.15
x
q = and decrement takes
place at two points during the year:
( ) ' 3
1
5
x
q at
1
2
t = and
( ) ' 3
4
5
x
q at 1 t = .
Calculate
( ) 3
x
q
Solution
Page 199 of 285 Deeper Understanding: Exam M November 7, 2006 2006 Yufeng Guo
http:/ / www.guo.coursehost.com

Well use the intuitive approach. The formula-driven approach is cumbersome.

t 0 1
2
1
( ) ' 1
t x
p
1
( ) ' 1
1 0.9
1 1 0.55
2 2
x
q = =
( ) ' 1
1 1 0.9 0.1
x
q = =
( ) ' 2
t x
p
1
( )
( )
2 1
1
2
2
1 0.271
x
e

=
( ) 1 0.271
( ) ( ) ' 1 ' 2
t x t x
p p
1
( )
1
2
0.55 1 0.271 0.4696 =
( ) 0.1 1 0.271 0.0729 =
Decrement ( ) 3
( ) ' 2
1
5
x
q
( )
1
0.15 3%
5
= =
( ) ' 2
4
5
x
q
( )
4
0.15 12%
5
= =
# of people hit by
cause ( ) 3
( ) 0.4696 3% 1.485% = ( ) 0.0729 12% 0.8748% =
( ) 3
1.485% 0.8748% 2.2836%
x
q = + =
Type 7 PVDB under the two decrement table

Example 1 (SOA problem modified)

A life insurance benefit of $10,000 is provided to Smith while he is actively employed.

Retirement is assumed to occur in the middle of the year.

Date of birth: 1/1/1938

Absolute rate of retirement:
( ) '
65
50%
r
q =
Absolute rate of mortality:
( ) '
65
4%
d
q =
In the associated single decrement tables, the force of mortality is constant within the
year of age.

7% i = compounded annually.

Calculate the one-year term cost for Smiths death benefit as of 1/1/2003 under the
following two scenarios:
Page 200 of 285 Deeper Understanding: Exam M November 7, 2006 2006 Yufeng Guo
http:/ / www.guo.coursehost.com

Scenario #1 The death benefit is payable at the moment of death.
Scenario #2 The death benefit is payable at the end of the year of death.

Solution

We are asked to find the PV of death benefit for a one-year term insurance. We have two
decrements, but only death triggers the insurance benefits.

Scenario #1 The death benefit is payable at the moment of death. The present value of
one-year term cost if the death benefit is $1:

( )
( )
( ) ( )
( )
( ) ( )
1 1
' '
0 0
d d d r t t
x t x x t x t x
PV t p e dt t p p e dt
t o o


= =
} }

We are given that in the associated single decrement tables the force of mortality is
constant within the year of age. Let
( ) d
represent this constant force of mortality. We
have:

( )
( )
( ) ( )
( )
'
d
d d d t
x t x
t p e



=
( )
( )
( )
1
'
0
d
d r t t
t x
PV e p e dt
o


=
}
Because retirement occurs in the middle of the year, we have

( )
( )
'
'
65
1 0 0.5
1 0.5 0.5 1
r
t x
r
t
p
q t
s <

=

= s s

( )
( )
( )
( )
( )
0.5 1
0 0.5
0.5
d d
d d t t t t
PV e e dt e e dt
o o


= +
} }

( )
( )
( )
( )
( )
( )
( )
0.5 0.5
0.5
0 0
1
d d
d
d
t
d d t t
d
e e dt e dt e
o o
o


o
( (
+ +
( (


= =
`
+ )
} }

( )
( )
( )
( )
( )
( ) ( )
1
0.5
0.5
0.5
0.5
d d
d
d
d t t
d
e e dt e e
o o
o

o
( (
+ +
( (


=
`
+ )
}
( )
( )
( )
( )
( )
( ) ( )
0.5 0.5 0.5
1
d d d
d d
d d
PV e e e
o o o
o o
( ( (
+ + +
( ( (


= +
` `
+ + ) )
Page 201 of 285 Deeper Understanding: Exam M November 7, 2006 2006 Yufeng Guo
http:/ / www.guo.coursehost.com

( ) ( ) ' '
65 65
4% 1 4% 96%
d d
q p = = =
( )
( ) ( ) '
65
96% ln 0.96
d
d d
e p

= = =
( ) ln 1 ln1.07 i o = + =
( )
0.96
ln
ln0.96 ln1.07
1.07
0.96
1.07
d
e e e
o
(
(
(


+
= = =
( )
0.5
0.96
1.07
d
e
o
(
(
(

+
=
ln 0.96 0.96 0.96 0.96
1 0.5 $0.029277
ln 0.96 ln1.07 1.07 1.07 1.07
PV
( | | | |

= + =
( | |
| |
+
(
\ . \ .

But the death benefit is $10,000. So the present value of one-year term cost is:

$0.029277 10, 000 $292.77 =
Scenario #2 The death benefit is payable at the end of the year when death occurs. The
present value of one-year term cost if the death benefit is $1:

( )
( )
( )
1
0
1
1
d
x t x
PV t p dt
i
t
=
+
}
( )
( )
( )
( )
( )
0.5 1
0 0.5
1
0.5
1
d d
d d t t
PV e dt e dt
i



(
= +
(
+

} }

( ) ( ) ( )
0.5 0.5 1 0.5
1
1 1
d d d
PV e e e
i i
o o
( ( (
+ +
( ( (


= +
` `
+ +
) )
( )
0.5ln0.96 0.5ln0.96 ln0.96
1
1 0.5 $0.028133
1.07
e e e
(
= + =


( )
1
1 0.96 0.5 0.96 0.96 $0.028133
1.07
(
= + =


Page 202 of 285 Deeper Understanding: Exam M November 7, 2006 2006 Yufeng Guo
http:/ / www.guo.coursehost.com

But the death benefit is $10,000. So the present value of one-year term cost is:

$0.028133 10, 000 $281.33 =
Page 203 of 285 Deeper Understanding: Exam M November 7, 2006 2006 Yufeng Guo
http:/ / www.guo.coursehost.com

Chapter 12 Markov Chain Problems

Historically, Markov Chain problems have always been a headache to most candidates in
Course 3. Fortunately, SOA removed many of the theoretical (often complex) topics
about the Markov Chain from the syllabus for Exam M. Now Exam M focuses on using
Markov Chain transition matrices to calculate the present value of cash flow associated
with the transition between states.

To master Markov Chain problems, youll need to do two things. First, learn how to
translate a pure math problem into a business problem. Second, learn some fundamentals
about matrices.

Tackling Markov Chain problems using a business-oriented approach

One strategy to tackle Markov Chain problems is to translate pure math problems into
business problems. Business problems are intuitive and easy to grasp. Its easier to think
in terms of business than in terms of pure math. Once you understand the business logic,
youll solve Markov Chain problems with confidence and ease.

Example 1 Course 3, Nov 04 # 14

For a Markov model for an insured population:

Annual transition probabilities between health states of individuals are as follows;

Healthy Sick Terminated
Healthy 0.7 0.1 0.2
Sick 0.3 0.6 0.1
Terminated 0.0 0.0 1.0
The mean annual health care cost each year for each health state is

Mean
Healthy 500
Sick 3,000
Terminated 0
Transition occurs at the end of the year
0 i =
Page 204 of 285 Deeper Understanding: Exam M November 7, 2006 2006 Yufeng Guo
http:/ / www.guo.coursehost.com

A contract premium of 800 is paid each year by an insured not in the terminated state.

Calculate the expected value of the contract premiums less healthcare costs over the first
3 years for a healthy newly-insured.

Solution

First, we need to understand what problem we need to solve. Here we have a newly
insured who is healthy. The question essentially asks us to find out

how much premium an insurance company will collect from this healthy insured
for the first 3 years (Item 1)

how much the insurance company has to pay this healthy insured for the first 3
policy years (Item 2)

The question asks us to find Item 1 Item 2.

Next, we need to translate the matrix into business language. Lets look at the first row
(0.7 0.1 0.2). These numbers mean the following:

If we have one healthy person at the beginning of a year, then at the end of the year this
person has

0.7 chance of still being healthy (and stays in the insurance companys book of
business)

0.1 chance of being sick (and stays in the insurance companys book of business)

0.2 chance of terminating his insurance policy (no longer in the insurance
companys book of business)

Similarly, the second row of the matrix (0.3 0.6 0.1) means that if we have one sick
person at the beginning of the year, then at the end of the year this person has

0.3 chance of becoming healthy (and stays in the insurance companys book of
business)

0.6 chance of being sick (and stays in the insurance companys book of business)

0.1 chance of terminating his insurance policy (no longer in the insurance
companys book of business)

Similarly, the third row of the matrix (0 0 1) means that if we have one terminated
person at the beginning of the year, then at the end of the year this person has
Page 205 of 285 Deeper Understanding: Exam M November 7, 2006 2006 Yufeng Guo
http:/ / www.guo.coursehost.com

0 chance of being healthy (this person terminated his policy and is no longer in
the insurance companys list of all the healthy policyholders)

0 chance of being sick (this person terminated his policy and is no longer in the
insurance companys list of all the sick policyholders)

1 chance of terminating his insurance policy (this person terminated his policy
and will continue to be in the insurance companys list of policyholders who have
terminated their insurance policies)

So the Markov Chain is really a process of transformation that takes place on a regular
basis (once every minute, every month, every quarter, etc). In this problem, the
transformation takes place every year. Every insured goes through an annual
transformation and updates his insurance status. If he is originally healthy, after the
annual transformation, he can become healthy, sick, or terminated.

To help us conveniently keep track of this complex annual transformation, well treat
probabilities as representing a partial person. For example, the first row (0.7 0.1 0.2)
means that one healthy person at the beginning of the year will be transformed into the
following three partial people:

0.7 healthy person (so the # of healthy people at the end of the year is 0.7)

0.1 sick person (so the # of the sick people at the end of the year is 0.1)

0.2 terminated (so the # of people who just terminated the policy at the end of the
year is 0.2)

Similarly, the second row of the matrix (0.3 0.6 0.1) means that if we have one sick
person at the beginning of the year, he will be transformed, at the end of the year, into the
following 3 partial people:

0.3 healthy person
0.6 sick person
0.1 person who is no longer in the insurance companys book of business

The third row (0 0 1) means that a policy, once terminated, will continue to be
terminated.

If you can map out the business meaning behind the matrix as we did above, you are on
the right track. Next, well keep track of how the healthy insured at t=0 will evolve in the
first 3 years of the policy.

At t=0, this healthy person bought the insurance policy and pays a premium of $800.

Page 206 of 285 Deeper Understanding: Exam M November 7, 2006 2006 Yufeng Guo
http:/ / www.guo.coursehost.com

At t=1, this person has 0.7 chance of still being health (thus paying premium), 0.1 chance
of being sick (thus paying premium), and 0.2 chance of terminating his policy (thus not
paying premium). The total chance of him paying premium is 0.8. So the total premium
collected from this person at t=1 is $800(0.8)=$640.

Alternatively, we can think that at t=1, we have 3 fractional people 0.7 person healthy,
0.2 person sick, and 1 person terminated. We can collect premium only from the healthy
and the sick. So the total premium collected at t=1 is: 0.7(800)+0.1(800)=$640.

At t=2. This is tricky. At t=1, we have two fractional people stay in the book of business,
0.7 person healthy and 0.1 sick. Each of these two fractional people will go through an
annual transformation during the 2
nd
year. The transformation rule is the same: Each
health person splits into 0.7 healthy person, 0.1 sick person, and 0.2 terminated person;
each sick person splits into 0.3 healthy person, 0.6 sick person, and 0.1 terminated person;
each terminated person stays unchanged.

Lets apply the transformation (or splitting) rule to the 0.7 healthy person at t=1. The new
people we have at t=2 are:

0.7(0.7)=0.49 health person (each paying premium of $800)
0.7(0.1)=0.07 sick person (each paying premium of $800)
0.7(0.2)=0.14 terminated person (no longer paying premium)

The total premium collected after this split is 800(0.49+0.07)=$448

Lets apply this splitting rule to the 0.1 healthy person at t=1. The new people we have at
t=2 due to this split are:

0.1(0.3)=0.03 health person (each paying premium of $800)
0.1(0.6)=0.06 sick person (each paying premium of $800)
0.1(0.1)=0.01 terminated person (no longer paying premium)

The total premium collected after this split is 800(0.03+0.06)=$72

Lets apply this process to the 0.2 terminated person at t=1. This person wont split. He
will stay the same at t=2 and wont pay any premium.

So the total premium collected at t=2 is 448+72=$520

So the total premium collected at t=0, 1, 2:

800+640+520=$1,960

Page 207 of 285 Deeper Understanding: Exam M November 7, 2006 2006 Yufeng Guo
http:/ / www.guo.coursehost.com

Lets draw a diagram of the above transformation process:

Only the red nodes pay premiums.
0.7 H the # of Hs=0.7(0.7)=0.49
0.1 S the # of Ss=0.7(0.1)=0.07
0.7 H 0.2 T
0.3 H the # of Hs=0.1(0.3)=0.03
1 H 0.1 S 0.6 S the # of Hs=0.1(0.6)=0.06
0.1 T
0 H
0.2 T
0 S
1 T
t=0 t=1 t=2
Its always a good idea not to further transform a terminated policy. This not only keeps
your diagram simple, but it also prevents errors.

In the above diagram, the bottom node at t=1 always stays at T and will never transform
anymore (a terminated policy stays terminated). This node is called absorbing node. We
shouldnt have further developed this absorbing node. So the above diagram needs to be
improved. If you further develop this absorbing node, errors may creep in.

For example, say you need to count the # of deaths during the 1
st
two years. If you use the
above diagram, you may conclude that

There is 0.2 death at t=1 (This is correct.)
There is 0.2+0.1+1(0.2)=0.5 death at t=2. (This is wrong.)
So the total death is 0.2+0.5=0.7 (Wrong because the death count at t=2 is wrong.)

The death count at t=2 is wrong. The # of deaths at t=2 is 0.2+0.1=0.3, not 0.5. The other
0.2 death at the bottom node at t=3 (the blue number in this diagram) is just the
continuation of the 0.2 death at t=1. So you double counted.
Page 208 of 285 Deeper Understanding: Exam M November 7, 2006 2006 Yufeng Guo
http:/ / www.guo.coursehost.com

So we need to remove the absorbing node (the bottom node at t=1). This gives the next
diagram:

Only the red nodes pay premiums.
0.7 H the # of Hs=0.7(0.7)=0.49
0.1 S the # of Ss=0.7(0.1)=0.07
0.7 H 0.2 T
0.3 H the # of Hs=0.1(0.3)=0.03
1 H 0.1 S 0.6 S the # of Hs=0.1(0.6)=0.06
0.1 T
0.2 T
t=0 t=1 t=2
Time t 0 1 2 Total
# of the healthy &
sick (they pay
premiums)
1 0.7+0.1=0.8 0.49+0.07+0.03+0.06
=0.65

Premiums paid $800 $800(.08)=$640 $800(0.65)=$520 800+640+520
=$1,960
Next, well calculate the expected cost of the health care during the first three years.
Once again we use the above diagram. Please note that only the red nodes incur costs.

Time t 0 1 2 Total
# of the healthy 1 0.7 0.49+0.03=0.52 1+0.7+0.52=2.22
# of the sick 0 0.1 0.07+0.06=0.13 0.1+0.13=0.23
The cost for one healthy person per year is $500. The cost for one sick person per year is
$3,000.
Page 209 of 285 Deeper Understanding: Exam M November 7, 2006 2006 Yufeng Guo
http:/ / www.guo.coursehost.com

So the total cost during the 1
st
three years is: 500(2.22)+3,000(0.23)=$1,800

So the expected excess of contract premiums over cost during the 1
st
3 years is:

1,960-1,800=$160

Example 2 SOA M Spring 2005 #11

For a Markov model with three states, Healthy (0), Disabled (1), and Dead (2):

The annual transition matrix is given by

0 1 2
0 0.70 0.20 0.10
1 0.10 0.65 0.25
2 0 0 1
| |
|
|
|
|
\ .
There are 100 lives at the start, all Healthy. Their future states are independent.

Calculate the variance of the number of the original 100 lives who die within the first two
years.

Solution

First, lets figure out what we are supposed to solve. We are given 100 healthy people at
t=0. We are asked to find the variance of the # of deaths from t=0 to t=2.

Let

p = the probability that a healthy person dies in two years.
X =the # of the 100 healthy people who die in two years.

X has a binomial distribution with parameter 100 = n and p . ( ) ( ) 1 Var X np p =
To find p , lets assume we have one healthy person at t=0. Then this person goes
through the annual splitting process at t=1 and t=2. If we sum up the total number of
deaths generated by the two splits, we should get p .
First, we draw a diagram. Let H=healthy, S=disabled, D=dead.

Page 210 of 285 Deeper Understanding: Exam M November 7, 2006 2006 Yufeng Guo
http:/ / www.guo.coursehost.com

0.7 H
0.2 S
0.7 H 0.1 D # of deaths=0.7(0.1)=0.07
0.10 H
1 H 0.2 S 0.65 S
0.25 D # of deaths=0.2(0.25)=0.05
0.1 D
t=0 t=1 t=2
So the total # of deaths in two years is 0.1+0.07+0.05=0.22 (just add up the red numbers).
So 0.22 p = . The variance is 100(0.22)(1-0.22)=17.16

Example 3 SOA M Spring 2005 #12

An insurance company issues a special 3-year insurance to a high risk individual. You are
given the following homogenous Mark chain model:

State 1: active
State 2: disabled
State 3: withdrawn
Sate 4: dead

Transition probability matrix:

1 2 3 4
1 0.4 0.2 0.3 0.1
2 0.2 0.5 0 0.3
3 0 0 1 0
4 0 0 0 1
Changes in sate occur at the end of the year.
The death benefit is 1,000, payable at the end of the year of death.
0.05 i =
Page 211 of 285 Deeper Understanding: Exam M November 7, 2006 2006 Yufeng Guo
http:/ / www.guo.coursehost.com

The insured is disabled at the end of year 2.

Calculate the actuarial present value of the prospective death benefits at the beginning of
the year 2.

Solution

Here the insurance is a three-year term, but one year has already elapsed. So the policy
has two more years to go. We are asked to calculate the present value of the death
benefits for the remaining two year term of insurance.

Calculating PV of death benefits for a two-year term of insurance is easy. The tricky part
is to sort out the death rate in each of the remaining 2 years. Once again, well draw a
diagram. Note that we start off with a disabled person at t=1.

Let A=active, S=disabled, W=withdraw, D=dead. Please note that W and D are absorbing
nodes and we should not split them any more.

0.4 A
0.2 S
0.2 A 0.3 W
0.1 D # of Ds=0.2(0.1)=0.02
0.2 A
0.5 S
0.5 S 0 W
1 S 0.3 D # of Ds=0.5(0.3)=0.15
0 W
0.3 D
t=1 t=2 t=3
Page 212 of 285 Deeper Understanding: Exam M November 7, 2006 2006 Yufeng Guo
http:/ / www.guo.coursehost.com

So the # of death at t=1 is 0.3. This is the death rate for the first year.

The # of deaths at t=2 is 0.02+0.15=0.17. This is the death rate for the second year.

PV of the death benefit is:

( )
2
2
0.3 0.17
1, 000 0.3 0.17 1, 000 439.91
1.05 1.05
v v + = + =
| |
|
\ .

Learn how to manipulate matrix

Now you know how to draw a splitting process to derive probabilities. Drawing a
splitting diagram gives us an intuitive feel, but doing so can be cumbersome if there are
three or more splitting processes. For example, if the term insurance policy in May 2005
Problem #12 has 3 or 4 years remaining, then the splitting diagram becomes complex.
We need to find a better approach to track down probabilities when a transformation
takes place three or more times.

General rule:

If a beginning state goes through n transitions (i.e. if a beginning state splits n times),
then the final state is
( ) Final State=Beginning State * Markov Transition Matrix
n
To memorize this rule, imagine that you are in the beginning state (ex. you are a healthy
person). Now you stand in front of the Markov Transition Matrix. On the top right corner
of the matrix is a Transform button. If you push this button, you get transformed. If you
push once, you get transformed once. If you push twice, you get transformed twice. To
transform n times, you need to push the Transform button n times. So your final state
is

( ) Final State=Beginning State * Markov Transition Matrix
n
Example 4 SOA M Spring 2005 #12 (alternative solution)

An insurance company issues a special 3-year insurance to a high risk individual. You are
given the following homogenous Mark chain model:

State 1: active
State 2: disabled
State 3: withdrawn
Sate 4: dead
Page 213 of 285 Deeper Understanding: Exam M November 7, 2006 2006 Yufeng Guo
http:/ / www.guo.coursehost.com

Transition probability matrix:

1 2 3 4
1 0.4 0.2 0.3 0.1
2 0.2 0.5 0 0.3
3 0 0 1 0
4 0 0 0 1
Changes in sate occur at the end of the year.

The death benefit is 1,000, payable at the end of the year of death.

0.05 i =
The insured is disabled at the end of year 2.

Calculate the actuarial present value of the prospective death benefits at the beginning of
the year 2.

Solution

This time, well manipulate the matrix without drawing a diagram.

The beginning state at 2 t = is ( ) 0 1 0 0 . The ending state after one transformation is:

( ) ( )
0.4 0.2 0.3 0.1
0.2 0.5 0 0.3
0 1 0 0 0.2 0.5 0 0.3
0 0 1 0
0 0 0 1
(
(
(
=
(
(


So @ 3 t = the # of death is 0.3 (i.e. the probability of death from 2 t = to 3 t = is 0.3).

The ending state after two transformations is:

( ) ( )
2
0.4 0.2 0.3 0.1 0.4 0.2 0.3 0.1
0.2 0.5 0 0.3 0.2 0.5 0 0.3
0 1 0 0 0.2 0.5 0 0.3
0 0 1 0 0 0 1 0
0 0 0 1 0 0 0 1
( (
( (
( (
=
( (
( (

( ) 0.18 0.29 0.06 0.47 =
Page 214 of 285 Deeper Understanding: Exam M November 7, 2006 2006 Yufeng Guo
http:/ / www.guo.coursehost.com

Please note that the # of deaths from 3 t = to 4 t = is 0.47-0.3=0.17, not 0.47. The figure
0.47 is the cumulative # of deaths from the beginning state at 2 t = to the ending state at
4 t = . In other words, 0.47 is the total probability of death from 2 t = to 4 t = . This is a
very important point. Not knowing this leads to an error.

General rule:
Whenever you use the formula
( ) Final State=Beginning State * Markov Transition Matrix
n
to calculate the final state, the calculated final state matrix reflects the cumulative
probability (from 0 t = to t n = ) for the absorbing states. Then the probability of an
absorbing state occurring from 1 t n = to t n = is calculated as follows:
The cumulative probability from 0 t = to t n = minus the cumulative probability from
0 t = to 1 t n =
For an un-absorbing state, the formula ( ) Final State=Beginning State * Markov Transition Matrix
n
correctly produces the probability that an un-absorbing state occurs from 1 t n = to t n = .
For example, if you want to find out the probability that a disabled person at 2 t = dies
from 3 t = to 4 t = , the calculation is as follows:

Step 1 Find the cumulative probability of deaths from 2 t = to 4 t = (total 2
transformations):

( ) ( )
2
0.4 0.2 0.3 0.1
0.2 0.5 0 0.3
0 1 0 0 0.18 0.29 0.06
0 0 1 0
0 0 0
0.47
1
(
(
(
=
(
(


This gives 0.47.

Step 2 Find the cumulative probability of deaths from 2 t = to 3 t = (only 1
transformation):

( ) ( )
0.4 0.2 0.3 0.1
0.2 0.5 0 0.3
0 1 0 0 0.2 0.5 0
0 0 1
0
0
0 0 0 1
.3
(
(
(
=
(
(


This gives 0.3
Page 215 of 285 Deeper Understanding: Exam M November 7, 2006 2006 Yufeng Guo
http:/ / www.guo.coursehost.com

Step 3 Find the difference.

0.47 0.3 = 0.17

This is the probability that a disabled person at 2 t = dies from 3 t = to 4 t = .
Another example. If you want to find the probability that a disabled person at 2 t = dies
from 4 t = to 5 t = , the calculation process is as follows:

Step 1 Find the cumulative probability of deaths from 2 t = to 5 t = (total 3
transformations):

( ) ( )
3
0.4 0.2 0.3 0.1
0.2 0.5 0 0.3
0 1 0 0 0.13 0.181 0.114
0 0 1 0
0 0 0 1
0.575
(
(
(
=
(
(


This gives 0.575.

Step 2 Find the cumulative probability of deaths from 2 t = to 4 t = (total 2
transformations):

( ) ( )
2
0.4 0.2 0.3 0.1
0.2 0.5 0 0.3
0 1 0 0 0.18 0.29 0.06
0 0 1 0
0 0 0
0.47
1
(
(
(
=
(
(


This gives 0.47.

Step 3 Find the difference.

0.575- 0.47 = 0.105

This is the probability that a disabled person at 2 t = dies from 4 t = to 5 t = .
This process is cumbersome, but its the correct way to keep track of the probability of
having additional deaths. If you dont like this approach, you have to draw a diagram to
keep track of additional deaths each year.

Please note that if a state is not absorbing, then the formula

( ) Final State=Beginning State * Markov Transition Matrix
n
correctly produces the probability that an un-absorbing state occurs from 1 t n = to t n = .
Page 216 of 285 Deeper Understanding: Exam M November 7, 2006 2006 Yufeng Guo
http:/ / www.guo.coursehost.com

Example. Whats the probability that a disabled person at 2 t = is active at 3 t = (only 1
transformation)?

( ) ( )
0.4 0.2 0.3 0.1
0.2 0.5 0 0.3
0 1 0 0 0.5 0 0.3
0 0 1 0
0 0
.
1
0 2
0
(
(
(
=
(
(


So the probability is 0.2

Example. Whats the probability that a disabled person at 2 t = is active at 4 t = (2
transformations are needed to go from 2 t = to 4 t = )?

( ) ( )
2
0.4 0.2 0.3 0.1
0.2 0.5 0 0.3
0 1 0 0 0.29 0.06 0.47
0 0 1 0
0 0 0 1
0.18
(
(
(
=
(
(


The probability is 0.18.

Come back to the problem. Now we know that the probability of death from 2 t = to 3 t =
is 0.3. The probability of death from 3 t = to 4 t = is 0.17.

PV of the death benefit is:

( )
2
2
0.3 0.17
1, 000 0.3 0.17 1, 000 439.91
1.05 1.05
v v + = + =
| |
|
\ .

Example 5

A policyholders status in disability insurance is classified as Healthy (H), Temporarily
Disabled (T), Permanently Disabled (P), and Dead (D). A policyholders status is updated
at the end of each year according to a time-discrete Markov Chain process with the
following transition matrix:

0.74 0.1 0.06 0.1
0.6 0.2 0.1 0.1
0 0 0.7 0.3
0 0 0 1
H T P D
H
T
P
D
This policy pays the policyholder when he is temporarily or permanently disabled.

A healthy policyholder bought the insurance policy at 0 t = .
Page 217 of 285 Deeper Understanding: Exam M November 7, 2006 2006 Yufeng Guo
http:/ / www.guo.coursehost.com

Calculate the probability for the following two scenarios:
The policyholder is disabled (temporarily or permanently) three years later
The policyholder will never get any benefits from this insurance policy

Solution

The beginning state at 0 t = is ( ) 10 0 0 . This state goes through the transformation process
3 times. Drawing a transformation diagram is cumbersome when the transformation takes
place 3 ore more times. As a result, well directly manipulate the matrix without a
diagram.

The ending state after 3 transformations is ( )
3
10 0 0 P , where P is the transition matrix.
The hard way to calculate ( )
3
10 0 0 P is to calculate
3
P . An easier way is to calculate as
follows:

( ) ( ) { }
3
10 0 0 10 0 0 P P P P = (


( ) ( ) 10 0 0 0.74 0.1 0.06 0.1 P =
( ) ( ) 0.74 0.1 0.06 0.1 0.6076 0.094 0.0964 0.202 P =
( ) ( ) 0.6076 0.094 0.0964 0.202 0.50602 0.07956 0.11334 0.30108 P =
( ) ( )
3
10 0 0 0.50602 0.07956 0.11334 0.30108 P =
( ) 0.50602 0.07956 0.11334 0.30108 is actually
0.50602 0.07956 0.11334 0.30108
H T P D | |
|
\ .
The probability that a healthy policyholder at 0 t = becomes disabled (temporarily or
permanently) at 3 t = is

0.07956 0.11334 0.1929 19.29% + = =
The next question is a little tricky. If the policyholder will never get any benefits from
this insurance policy, then he shouldnt be transformed into either T or P . He can be
transformed into either H (stay healthy) or D (die) in each future year while hes alive.
The probability is:

2
die @ 1 healthy @ 1 healthy @ 1,2 healthy @ 1,2,...,
die @ 2 die @ 3 die @ 1
0.1 0.74 0.1 0.74 0.1 ... 0.74 0.1 ...
n
t t t t n
t t t n
= = = =
= = = =
+ + + + +


( )
2
0.1 1 0.74 0.74 ... 0.74

= + + + +
Page 218 of 285 Deeper Understanding: Exam M November 7, 2006 2006 Yufeng Guo
http:/ / www.guo.coursehost.com

0.1
= 0.38462
1 0.74
=

Example 6

A policyholders status in a 3-year term insurance policy is classified as Healthy (H),
Temporarily Disabled (T), Permanently Disabled (P), and Dead (D). A policyholders
status is updated at the end of each year according to a time-discrete Markov Chain
process with the following transition matrix:

0.74 0.1 0.06 0.1
0.6 0.2 0.1 0.1
0 0 0.7 0.3
0 0 0 1
H T P D
H
T T
P
D
(
(
(
( =
(
(
(

A healthy policyholder bought the insurance policy at 0 t = .
This policy pays the policyholder the following benefits:
$1,000 paid at the end of the year if the policyholder is temporarily disabled.
$5,000 paid at the end of the year if the policyholder is permanently disabled.
$10,000 paid at the end of the year if the policyholder is dead.

6% i =
Premiums are collected at the beginning of the year.

You are also given:

2
0.608 0.094 0.096 0.202
0.564 0.1 0.126 0.21
0 0 0.49 0.51
0 0 0 1
T
(
(
(
=
(
(

3
0.506 0.080 0.113 0.301
0.477 0.077 0.132 0.314
0 0 0.343 0.657
0 0 0 1
T
(
(
(
=
(
(

Calculate the benefit premium.

Solution

We need to track down, year by year, how many people are healthy, temporarily disabled,
permanently disabled, or dead.

The starting point is ( ) 1 0 0 0 at 0 t = . We have 1 healthy insured.

Page 219 of 285 Deeper Understanding: Exam M November 7, 2006 2006 Yufeng Guo
http:/ / www.guo.coursehost.com

At 1 t = , the state is ( ) ( ) 1 0 0 0 0.74 0.1 0.06 0.1 T = . So we have 0.74 person healthy,
0.1 temporarily disabled, 0.06 permanently disabled, and 0.1 dead. The incremental death
is 0.1- 0=0.1.

At 2, t = the state is ( ) ( )
2
1 0 0 0 0.608 0.094 0.096 0.202 T = . So we have 0.608 person
healthy, 0.094 temporarily disabled, 0.096 permanently disabled, and 0.202 dead. The
incremental death is 0.202-0.1=0.102.

At 3 t = , the state is ( ) ( )
3
1 0 0 0 0.506 0.08 0.113 0.301 T = . So we have 0.506 person
healthy, 0.008 temporarily disabled, 0.113 permanently disabled, and 0.301 dead. The
incremental death is 0.301-0.202=0.099.

Next, we come up with table to keep track of policyholders in different states:

t 0 1 2 3
Healthy (pay premium) 1 0.74 0.608 0.506
Temp Disabled (pay premium
and collect disability benefits)
0.10 0.094 0.080
Perm Disabled (pay premium
and collect disability benefits)
0.06 0.096 0.113
Dead (get death benefits) 0.10 0.102 0.099
Total 1 1.00 0.900
(1)
0.798
(2)
Note:
(1) The total # of remaining policyholders doesnt add up to 1. This is because we have
0.1 person dead at 1 t = . Thus the total population remaining at 2 t = is 1- 0.1= 0.9.

(2) The total # of remaining policyholders doesnt add up to 1. This is because we have
0.1 person dead at 1 t = and 0.102 dead at 2 t = . Thus the total population remaining at
3 t = is 1- 0.1- 0.102= 0.798.

PV of the premiums collected:

( ) ( )
2
1 0.74 0.10 0.06 0.608 0.094 0.096 2. 5593 P v v P ( + + + + + + =


PV of the benefits paid (using $1,000 as one unit of money):

( ) ( ) ( )
2 3
1 0.1 5 0.06 10 0.1 1 0.094 5 0.096 10 0.102 1 0.08 5 0.113 10 0.099 v v v + + + + + + + +
=6. 6594
Equivalence principle PV of premiums = PV of benefits

2. 5593 6. 6594 2.60204 $2, 602.04 P P = = =
Page 220 of 285 Deeper Understanding: Exam M November 7, 2006 2006 Yufeng Guo
http:/ / www.guo.coursehost.com

Example 7

An auto insurance company offers the following discount program to its insureds car
insurance premiums:

Level 0 Zero discount
Level 1 15% discount
Level 2 25% discount
Level 3 35% discount

Rules:

If an insured is accident-free in a year, his discount level increases by 1, unless hes
already in Level 3.

If an insured has at least one accident in a year, his discount level decreases by 1, unless
hes in Level 0.

Given:

Each year, an insured has 30% chance of having at least one accident.

Explain why this discount program can be modeled by a homogeneous Markov chain.
Write the transition matrix.

Solution

The discount level at 1 n + depends only on the discount level at n and doesnt depend on
the discount level prior to n . In addition, the chance of discount levels doesnt depend on
time n . So well use a homogeneous Markov chain to model how discount levels change
over time.

Transition matrix:

0 1 2 3
0 0.3 0.7 0 0
1 0.3 0 0.7 0
2 0 0.3 0 0.7
3 0 0 0.3 0.7
(
(
(
(
(
(
(

Page 221 of 285 Deeper Understanding: Exam M November 7, 2006 2006 Yufeng Guo
http:/ / www.guo.coursehost.com

Example 8
An auto insurer offers four levels of discounts.
Discount Annual Premiums
0 $1,000
1 $800
2 $600
3 $400
Rules for assigning discount levels:
A new insured is assigned to Level 0
If an insured is accident-free in a year, his discount level goes up by 1 (unless hes
already at Level 3)
If an insured has at least one accident, his discount level goes down by 2 (unless
hes already at Level 0)

The probability that an insured has at least one accident in a year is 0.25, independent of
whether he had any accident in prior years.

Calculate the expected premium to be paid by a newly insured in his 2
nd
policy year.

Solution
The evolution of discount levels can be modeled by a homogeneous Markov chain with
the following transition matrix:

0 1 2 3
0 0.25 0.75 0 0
1 0.25 0 0.75 0
2 0.25 0 0 0.75
3 0 0.25 0 0.75
(
(
(
(
(
(
(

For a newly added insured, his beginning state is ( ) 1 0 0 0 . His expected state next
year is

( ) ( )
0.25 0.75 0 0
0.25 0 0.75 0
1 0 0 0 0.25 0.75 0 0
0.25 0 0 0.75
0 0.25 0 0.75
(
(
(
=
(
(


The expected premium paid during the 2
nd
year is:

0.25(1,000)+0.75(800)+0(600)+0(400)=$850

Page 222 of 285 Deeper Understanding: Exam M November 7, 2006 2006 Yufeng Guo
http:/ / www.guo.coursehost.com

Chapter 13 Stochastic process, counting process,
Poisson process

Stochastic process

Problem 1

Whats a stochastic process? Give me an example.

Solution

A stochastic process is a collection of random variables whose values are indexed with
time. A stochastic process { } ( ), X X t t T = e is a collection of random variables ( ) X t . For
each t in the index set T , ( ) X t is a random variable.

The index set T can be discrete (in that case we have a discrete stochastic process) or
continuous (a continuous stochastic process).

In a stochastic process, the random variable ( ) X t associated with any time t is partly
deterministic and partly random.

For example, the interest rate is a stochastic process. For any particular year t , theres an
interest rate ( ) I t . If you have kept track of how the interest rate changes over many
years, youll have a collection of random variables { } ( ), I I t t T = e .
( ) I t is partly deterministic; we know that ( ) I t must have a lower bound (for example
( ) 0 I t > because you cant have a zero or negative interest rate) and an upper bound (the
federal government wont let the interest rate go up to, for example, 50%).

( ) I t is also partly random. No one can precisely predict what the interest rate is for the
next month, next quarter, next year because interest rate fluctuates with time.

If you can consistently predict how the interest rate will change over time, youll become
rich quickly. For example, if you know that the interest rate will go up from 4% to 6%
next month, you can borrow money from a bank at 4% just one day before the new
interest rate takes off. Then the next day when the interest rate is 6%, you can turn around
and lend the borrowed money to someone at 5% -- you shouldnt have much trouble
finding an eager borrower. You can make easy money by collect 1% net interest.

In most cases, a stochastic process has a forecast value (ex. 4% expected interest rate)
and a drift term (ex. interest rate can be higher or lower than 4%). A stochastic variable
Page 223 of 285 Deeper Understanding: Exam M November 7, 2006 2006 Yufeng Guo
http:/ / www.guo.coursehost.com

( ) X t is equal to the sum of the forecast value and the drift term (ex. the interest rate
moves up or down around 4%). In other words, [ ] ( ) ( ) ( ) X t E X t Error t = + .
Other examples of a stochastic process: stock price (such as AT&T stock price); the
temperature outside your house.

Counting process
Problem 2

Whats a counting process? Give me an example.

Solution

A stochastic process { } ( ), 0 N t t > is said to be a counting process if ( ) N t represents the
total number of events that have occurred from time zero to time t.
From this definition we can see that in a counting process:

(1) ( ) 0 N t > . You cant have a negative number of events.

(2) ( ) N t is an integer. The number of events must be an integer.

(3) If s t < , then ( ) ( ) N s N t s . As time passes, the number of events cant go
down.

(4) For s t < , ( ) ( ) N s N t equals the number of events that have occurred during the
interval [ ] , s t .
Example. Let ( ) N t represent the number of customers who have arrived at a store during
the time interval [ ] 0, t where time zero represent any arbitrary starting time (such as 8:00
am). ( ) N t is a counting process.

Page 224 of 285 Deeper Understanding: Exam M November 7, 2006 2006 Yufeng Guo
http:/ / www.guo.coursehost.com

Poisson process
Problem 3

Whats a Poisson process?

Solution

We can define a Poisson process in several ways:

1
st
definition

The counting process { } ( ), 0 N t t > is a Poisson process having rate ( 0 > ) if:
(1) (0) 0 N = .
(2) Events occurring in disjoint time (i.e. non-overlapping) intervals are
independent (independent increments).
(3) The number of events in any interval of length t is Poisson distributed
with mean t :
{ }
( )
( ) ( ) , 0,1, 2...
!
n
t
t
P N t s N s n e n
n

+ = = =
2
nd
definition

The counting process { } ( ), 0 N t t > is a Poisson process having rate ( 0 > ) if
(1) (0) 0 N =
(2) Events occurring in disjoint time intervals are independent.
(3) { } ( ) ( ) 1 P N h h o h = = +
(4) { } ( ) ( ) 2 P N h o h > =
A function ( ) f h is said to be ( ) o h is
0
( )
lim 0
h
f h
h

= .
Page 225 of 285 Deeper Understanding: Exam M November 7, 2006 2006 Yufeng Guo
http:/ / www.guo.coursehost.com

The 2
nd
definition says that if you make your time interval h very very small (i.e.
0 h ), then ( ) 0 o h . In other words, in a very small interval of length h , either no
event occurs or only one event occurs; the chance of having two events or more is zero
(because { } ( ) ( ) 2 0 P N h o h > = ).

The 2
nd
definition essentially treats a Poisson process as a baby-step binomial
distribution. To see why, lets break the time interval [ ] 0, t into m intervals, each
interval being t m. Lets define having one event as success and having zero event as
failure. Then we have a binomial distribution with mtrials and a success rate
t
p
m

| |
=
|
\ .
.
Here we assume the success rate in an interval is proportional to the length of the interval
( is the success rate in an interval of length one). We have a binomial distribution
because (1) disjoint intervals have independent increments, (2) each interval can have at
most one success. Then it follows:

[ ] ( )
!
( ) 1 1
( )! !
n m n
m n
n n
m
m t t
P N t n C p p
m n n m m


| | | |
= = =
| |

\ . \ .
( ) ( 1)...( 1)
1 1
!
n m n
n
t m m m n t t
m n m m


+ | | | |
=
| |
\ . \ .
As m +,
( 1)...( 1)
1
n
m m m n
m
+
, 1 1 , 1 1
t
t
m
n
t
m
t t t
e
m m m

(
| | | | | |
(
=
| | |
(
\ . \ . \ .


So [ ]
( )
( )
!
t
n
t
P N t n e
n

=
3
rd
definition

Let
1 2
, ,...,
n
T T T represent n independent identical exponential distributions with
parameter . Let
1 2
...
n n
S T T T = + + + (we define
0
0 S = ).
Define ( ) N t as the max n such
n
S t s (i.e. n is the max number of events during the
time interval [ ] 0, t ).
Page 226 of 285 Deeper Understanding: Exam M November 7, 2006 2006 Yufeng Guo
http:/ / www.guo.coursehost.com

1 2
, , ...,
n
T T T are called the inter-arrival times.
1
T is the time until the arrival of the 1
st
event;
2
T is the time between the arrival of the 1
st
event and the arrival of the 2
nd
event.
n
T is the time between the arrival of the ( ) 1 n -th event and the arrival of the n -th
event.
n
S is the total waiting time or total sojourn time before the arrival of the n -th event.
Please note that
n
S has gamma distribution with the following pdf:

1
, 0,1, 2...
( )
( )
( 1)!
n
n
n S
n
n
s
n
s
f s e
n

= =

Many candidates have troubling memorizing the gamma pdf. To help us memorize this
difficult pdf, we rewrite the pdf into:

1
Poisson Process
having 1 events
, 0,1, 2...
( )
( )
( 1)!
n
n
n S
n
n
s
n
n
s
f s e
n

= =


In other words, the gamma pdf is a multiply of the Poisson density function for 1 n
events. Theres no intuitive explanation for this shortcut. Just memorize it.
Please also note that a gamma distribution and a Poisson process are two sides of a coin:

During the time interval [ ] 0,
n
S youve observed n random events. If you treat
n
S as
fixed and treat n as the random variable (so you are interested in the question How
many random events can I observe during the interval [ ] 0,
n
S ?), youll have a Poisson
process:

Page 227 of 285 Deeper Understanding: Exam M November 7, 2006 2006 Yufeng Guo
http:/ / www.guo.coursehost.com

[ ]
( )
( ) , 0,1, 2...
!
n
n
S
n
n
S
P N S n e n
n

= = =
During the time interval [ ] 0,
n
S youve observed n random events. If you treat n as
fixed and treat
n
S as the random variable (so you are interested in the question How
much time must elapse before I can observe n random events?), youll have a gamma
distribution:
1
( )
( ) , 0,1, 2...
( 1)!
n
n
n
s n
S n
s
f s e n
n

= =

( ) N t
4
3
2
1

1
S

2
S

3
S

4
S

1
T
2
T
3
T
4
T
1 1
S T = ,
2 1 2
S T T = + ,
3 1 2 3
S T T T = + + ,
4 1 2 3 4
S T T T T = + + + ,
1 2
...
n n
S T T T = + + +
Page 228 of 285 Deeper Understanding: Exam M November 7, 2006 2006 Yufeng Guo
http:/ / www.guo.coursehost.com

Lucky Tom finding coins

Problem 5

Lucky Tom finds coins on his way to work at a Poisson rate of 0.2 coins per minute.
Calculate the probability that he finds no coins during his walk from 7:00 am to 7:10 am.

Solution

Method 1

We can simply apply the formula:

{ }
( )
( ) ( ) , 0,1, 2...
!
n
t
t
P N t s N s n e n
n

+ = = =
We have 0.2, 0 n = = .
{ } { } (7:10 am) (7: 00 am) 0 (10 minutes) 0 P N N P N = = =
( )
0
0.2 (10) 0.2 (10)
0.2 10
0.1353
0!
e e

= = =
Method 2

{ } ( )
1
(10) 0 10 P N P T = = >
The above equation stands because if the arrival time of the 1
st
event exceeds 10 minutes,
you wont see any event during the 10 minutes interval.

1
T is exponentially distributed with parameter 0.2 = (see the 3
rd
definition of a Poisson
process). ( )
1
t
P T t e

> = .
{ } ( )
0.2(10)
1
(10) 0 10 0.1353 P N P T e

= = > = =
Problem 6

Lucky Tom finds coins on his way to work at a Poisson rate of 0.2 coins per minute.
Calculate the probability that he finds 2 coins during the first 10 minutes of his walk and
5 coins during the first 25 minutes of his walk.

Solution
Page 229 of 285 Deeper Understanding: Exam M November 7, 2006 2006 Yufeng Guo
http:/ / www.guo.coursehost.com

The required probability is { } (10) 2 (25) 5 P N N = = .
{ } (10) 2 (25) 5 P N N = =
[ ] [ ] { } { } { } (10) 2 (25) (10) 3 (10) 2 (25) (10) 3 P N N N P N P N N = = = = = =
(non-overlapping intervals have independent Poisson processes)
However, { } { } (25) (10) 3 (15) 3 P N N P N = = =
(Only the length of the interval matters. Property 3 of the 1
st
definition)

{ } { } { } (10) 2 (25) 5 (10) 2 (15) 3 P N N P N P N = = = = =
The above equality means that in order to find 2 coins in the 1
st
10 minutes and 5 coins in
the 1
st
25 minutes, Tom needs to find 2 coins in the 1
st
10 minutes and 3 additional coins
in the next 15 minutes.

{ } { }
( ) ( )
2 3
0.2 (10) 0.2 (15)
0.2 10 0.2 15
(10) 2 (15) 3 0.06064
2! 3!
P N P N e e


= = = =
Problem 7

Lucky Tom finds coins on his way to work at a Poisson rate of 0.2 coins per minute.
Calculate the probability that he finds 2 coins during the first 10 minutes of his walk,
given that he finds 5 coins during the first 25 minutes of his walk.
Solution

The required probability is
{ }
(10) 2 (25) 5 P N N = = .
{ }
{ }
{ }
{ } { }
{ }
(10) 2 (25) 5 (10) 2 (15) 3
(10) 2 (25) 5
(25) 5 (25) 5
P N N P N P N
P N N
P N P N
= = = =
= = = =
= =

{ }
( )
5
0.2 (25)
0.2 25
(25) 5 0.1755
5!
P N e


= = =
{ } { } (10) 2 (15) 3 0.06064 P N P N = = = (calculated in the last problem)

{ }
0.06064
(10) 2 (25) 5 0.3456
0.01755
P N N = = = =
Page 230 of 285 Deeper Understanding: Exam M November 7, 2006 2006 Yufeng Guo
http:/ / www.guo.coursehost.com

Problem 8

Lucky Tom finds coins on his way to work at a Poisson rate of 0.2 coins per minute.
Calculate the probability that he finds 5 coins during the first 25 minutes of his walk,
given that he finds 2 coins during the first 10 minutes of his walk.

Solution
The required probability is
{ }
(25) 5 (10) 2 P N N = = .
{ }
{ }
{ }
{ } { }
{ }
(10) 2 (25) 5 (10) 2 (15) 3
(25) 5 (10) 2
(10) 2 (10) 2
P N N P N P N
P N N
P N P N
= = = =
= = = =
= =

{ }
( )
3
0.2 (15)
0.2 15
(15) 3 0.2240
3!
P N e


= = = =
Problem 9

Claims arrive at an insurance company at a Poisson rate of one claim every five days.

Calculate the probability that

(1) at least 3 claims arrive in 4 days,
(2) no more than 3 claims arrive in 4 days.
(3) exactly 3 claims arrive in 4 days.

Solution

The number of claims that have arrived at the insurance company during the interval
[ ] 0, 4 days is a Poisson process with the parameter 1 5 = .
( )
( )
, 0,1,...
!
n
t
t
P n e n
n

= = where 1 5 = and 4 t = .
[ ] ( 3) 1 ( 0) ( 1) ( 2) P n P n P n P n > = = + = + =
( )
4 5
0
t
P n e e

= = = , ( )
( )
1
4 5
4
1
1! 5
t
t
P n e e



= = = , ( )
( )
2
4 5
8
2
2! 25
t
t
P n e e



= = =
(1)
4 5 4 5
4 8 53
( 3) 1 1 1
5 25 25
P n e e

| |
> = + + =
|
\ .

Page 231 of 285 Deeper Understanding: Exam M November 7, 2006 2006 Yufeng Guo
http:/ / www.guo.coursehost.com

(2)
4 5 4 5
4 8 53
( 3) ( 0) ( 1) ( 2) 1
5 25 25
P n P n P n P n e e

| |
s = = + = + = = + + =
|
\ .

(3) ( )
3
4 5
1 4
3
3! 5
P n e

| |
= =
|
\ .

Problem 10

Claims arrive at an insurance company at a Poisson rate of one claim every five days.

Without using the probability formula of a Poisson process and using the 3
rd
definition of
a Poisson process, calculate the probability that:

(1) at least 3 claims arrive in 4 days;
(2) no more than 3 claims arrive in 4 days;
(3) exactly 3 claims arrive in 4 days.

Solution

( )
3
3 in 4 days (0 4 days) P n P S > = < s
The above equation says that in order to have at least 3 claims in 4 days, the total arrival
time for the first three claims must be no more than 4 days. In other words, the 3
rd
claim
must take place at the end of the 4
th
day at the latest. If the total arrival time for the first 3
claims exceeds 4 days, theres no way you can have at least 3 claims in the first four
days.

( )
4
3 3 3
0
3
(0 4 days)
S
P S f s dS < s =
}
The above equation is just an application of a generic formula

( ) ( )
b
a
X
P a X b f x dx s s =
}
3
S has gamma distribution with pdf:

Page 232 of 285 Deeper Understanding: Exam M November 7, 2006 2006 Yufeng Guo
http:/ / www.guo.coursehost.com

3
3
3 1
3
3
Poisson process
having (3 1)
events during [0, ]
( )
( )
(3 1)!
S
S
t
S
f S e

where 1 5 =
3 3
3
3 1
1 1
3
2
5 5
3 3
1
( )
1 1 1
5
( )
5 (3 1)! 50 5
S S
S
S
f S e S e


| |
= =
|

\ .

3
4 1
2
5
3 3 3
0
1 1
(0 4 days)
50 5
S
P S S e dS

| |
< s =
|
\ .
}
This is a difficult integration. Fortunately, we already have a shortcut:
For any 0 u > and 0 a > :
u u
u
+

=
}
/ /
1
x a
a
e dx e
u u
u
u
+
| |
|
\ .
= +
}
/ /
1
( )
a
a
x
x dx a e e
u u
u u
u
+

| |
|
\ .
= + +
}
/ / 2 2 2
1
[( ) ]
x a
a
x e dx a e
To find the explanation for these shortcuts, go to http://www.actuarialbookstore.com and
download the sample chapter from my P manual: Deeper understanding, faster
calculation Exam P/1 Insights & Shortcuts.

Lets continue.
3 3 3 3
4 4 1 1 1 1
2 2 2 2
5 5 5 5
3 3 3 3 3 3 3 3
0 0 0 4
1 1 1 1 1 1 1
50 5 50 5 50 5 5
S S S S
S e dS S e dS S e dS S e dS

( | | | |
| |
= =
( | | |
\ . (
\ . \ .
} } } }

( ) ( )
{ }
2 2
2 0 5 2 4 5
1
0 5 5 4 5 5
50
e e

( (
= + + + +


( )
4 5 4 5
1 53
50 106 1
50 25
e e

= =
Next, well calculate the probability of having no more than 3 claims arrive in 4 days.

Page 233 of 285 Deeper Understanding: Exam M November 7, 2006 2006 Yufeng Guo
http:/ / www.guo.coursehost.com

( )
3
3 in 4 days (4 days ) P n P S s = s
The above equation says that in order to have no more than 3 claims in 4 days, the total
arrival time for the first three claims must be no earlier than 4 days. In other words, the
3
rd
claim must take place at the end of the 4
th
day at the earliest.

3 3
1 1
2 2
5 5
3 3 3 3 3
4 4
1 1 1 1
(4 days )
50 5 50 5
S S
P S S e dS S e dS


| | | |
s = =
| |
\ . \ .
} }

2 2 4 5 4 5
1 53
5 (5 4)
50 25
e e

( = + + =


Finally, well calculate the probability that exactly 3 claims arrive in 4 days.

( )
3 4 3 3 4 3 4 3
3 in 4 days ( 4 4) ( 4 4) ( 4 4 ) P n P S S P S S T P S T S = = < > = < + > = < >
The above equation says that in order to have exactly 3 claims in four days, the 3
rd
claim
must arrive before the end of the 4
th
day and 4
th
claim must arrive after the end of the 4
th

day.

To calculate
3 4 3
( 4 4 ) P S T S < > , we first set
3 3
S s = where
3
0 4 s < < . Then we let
4 3
4 T s > . Finally, we integrate
3 4 3
( 4 4 ) P S T S < > over all possible values of
3
s :
3 4 3 3 3 4 3 3
3
3
all possible
where 0< 4
( 4 4 ) ( 4 )
s
s
P S T S P S s T s ds
<
< > = = >
}
4
3 3 4 3 3
0
( 4 ) P S s T s ds = = >
}
Because
3
S and
4
T are independent (
3 1 2 3
S T T T = + + and
1 2
, ,...,
n
T T T are independent), we
have:

( )
3 3 4 3 3 3 4 3
( 4 ) ( 4 ) P S s T s P S s P T s = > = = >
( )
4
3 4 3 3 3 4 3 3
0
( 4 4 ) ( 4 ) P S T S P S s P T s ds < > = = >
}
( )
3
3
1
2
5
3 3 3 3
1 1
( )
50 5
S
S
P S s f S S e

| |
= = =
|
\ .
(we already know this)

Page 234 of 285 Deeper Understanding: Exam M November 7, 2006 2006 Yufeng Guo
http:/ / www.guo.coursehost.com

1
(4 )
5
4 3
3
( 4 )
s
P T s e

> =
4
T is exponentially distributed with parameter 1 5 =
( )
3
1 1
(4 )
2 4 5 2
5 5
3 3 4 3 3 3
3
1 1 1
( 4 )
50 5 250
S s
P S s P T s S e e e s


| |
| | | |
= > = =
| | |
\ . \ .
\ .

( )
4 3 4 4
4 5 2 4 5 3 4 5
3 3 4 3 3 3 3 3
0 0 0
1 1 1 1 4
( 4 )
250 250 3 3! 5
P S s P T s ds e s ds e s e

| | ( | |
= > = = =
| |
(
\ . \ .
} }

( )
3
4 5
1 4
3 in 4 days
3! 5
P n e

| |
= =
|
\ .

Key Points
The 3rd definition works.
{ } ( ) (0 )
n
P N t n P S t > = < s
{ } ( ) ( )
n
P N t n P S t s = >
{ }
1
( ) ( ) ( )
n n n n n
P N t n P S t S t P S t S T t
+
= = < > = < + >
( ) ( ) ( )
n n n n n n
P S t T t S P S t P T t S = < > = < >
Problem 11 ( two Poisson processes competing)

Two independent Poisson processes with parameters
1
and
2
are going on
simultaneously. Whats the probability that the 1
st
Poisson process experiences n arrivals
before the 2
nd
Poisson process experiences m arrivals?

Solution

Probability that the 1
st
Poisson process has n arrivals before the 2
nd
Poisson process has
m arrivals is:

( )
1 2
n m
P S S <
To actually derive the formula for
( )
1 2
n m
P S S < is quite complex, but well proceed
anyway because well gain many insights by embarking on this difficult task. To begin
Page 235 of 285 Deeper Understanding: Exam M November 7, 2006 2006 Yufeng Guo
http:/ / www.guo.coursehost.com

with, we first set
2
m
S t = where 0 t < < . Then we set
1
n
S t < . Finally, we integrate
( )
1 2
n m
P S S < over all possible values of t .
( ) ( ) ( )
1 2 2 1 2 1
0 all possible
values
where 0
n m m n m n
t
t
P S S P S t S t dt P S t S t dt

< <
< = = < = = <
} }

( ) ( )
2 1
0
m n
P S t P S t dt

= = <
}
(
2 1
,
m n
S S are independent)

( )
( )
2
1
2 2
2
( 1)!
m
t
m
t
P S t e
m

= =

(gamma pdf, which we already know)


( )
1
n
P S t < is a gamma cumulative distribution function (cdf), but we dont know the
gamma cdf. Trouble? It turns out that we dont need to know the gamma cdf. Remember
that gamma distribution and Poisson process are just two sides of a coin.

( ) ( )
1
The total arrival time of events is less than
n
P S t P n t < =
[ ] ( )
# of events that have occurred during 0, P t n = >
[ ] ( )
1 # of events that have occurred during 0, P t n = <
( ) ( )
( )
1
2 1
1 1
1
1 1 ...
2! 1 !
n
t
t t
e t
n

(
= + + + + (

(

The last equality follows because the number of events that have occurred during [ ] 0, t is
a Poisson process.

( ) ( )
( ) ( )
( )
( )
1 2
2 1 1
1 1 2 2 1
1 2
0 0
1 1 ...
2! 1 ! ( 1)!
n m
t t
m n
t t t
P S t P S t dt e t e dt
n m






(

= < = + + + + (
`

(

)
} }

From the above equation you can clearly see that
( )
1 2
n m
P S S < means that you have at least n successes before you have m failures.
( )
1 2
n m
P S S < does NOT mean that you must have exactly n successes before m failures.
Come back to the problem. To solve the messy integrations on the right hand side, well
need a shortcut:

Page 236 of 285 Deeper Understanding: Exam M November 7, 2006 2006 Yufeng Guo
http:/ / www.guo.coursehost.com

1
0
!
t n
n
n
e t dt
o
o

+
=
}
where 0,1, 2,... n =
Youll need to memorize this because its very useful for Exam M and C.
To understand why the above equation holds, notice

( )
( )
( )
( 1) 1
1 1 1
0 0
gamma pdf with
parameters 1 and
1 1 ! 1 1 !
( ) !
1 1 !
n
t n t
n n n
n
n n
t n
e t dt e dt
n
o o
o
o
o
o o o
+

+ + +
+
+ + ( (

= = =
+ (

} }


We can also check that the equation holds for n=0 and n=1.
If n=0, then
0
0! 1
t
e dt
o
o o

= =
}
(A perpetual continuous annuity
1
a
o
+
= )
If n=1, then
2
0
1
t
e t dt
o
o

=
}
(A perpetual continuous increasing annuity
( )
2
1
Ia
o
+
= )
Lets get back to the problem.

( ) ( )
( )
( )
1 2
2 1 1
1 1 2
1 2
0
1 1 ...
2! 1 ! ( 1)!
n m
t t
t t t
e t e dt
n m




(

+ + + + (
`

(

)
}
( ) ( )
( )
1 2
2 1
1 1 1 2
1
0
1 1 ...
2! 1 ! ( 1)!
n
m
t t m
t t
e t e t dt
n m


(

= + + + + (
`

(

)
}
( ) ( )
( )
2 1 2
2 1
1 1 ( ) 1 1 2 2
1
0 0
1 ...
( 1)! ( 1)! 2! 1 !
n
m m
t t m m
t t
e t dt e t t dt
m m n






+
(
= + + + + (

(

} }

Well integrate the right hand side item by item:

Page 237 of 285 Deeper Understanding: Exam M November 7, 2006 2006 Yufeng Guo
http:/ / www.guo.coursehost.com

2 2
1 1 2 2 2
2 0 0
( 1)!
1
( 1)! ( 1)! ( 1)!
m m m
t t m m
m
m
e t dt e t dt
m m m




= = =

} }

1 2 1 2
( ) ( ) 1 1 2 2 2
2
1 2 1 2 0 0
( 1)! 1
( 1)! ( 1)! ( 1)! ( )
m
m m m
t t m m m
m
m
e t dt e t dt
m m m





+ +
| |
= = =
|
+ +
\ .
} }

[ ]
1 2 1 2
( ) ( ) 1 2 1 2
1
0 0
( 1)! ( 1)!
m m
t t m m
e t t dt e t dt
m m


+ +
=

} }

( )
1 2 1
2 1 1
1 2
1 2
!
( 1)! ( )
m
m
m m
m
m



+ +
= =
+
+
( )
1 2 1 2
2
2
1 ( ) ( ) 1 1 2 1 2
0 0
( 1)! 2! 2!( 1)!
m m
t t m m
t
e t dt e t dt
m m



+ + +
=

} }

2 2
1 2 1
2 2 2
1 2 1 2
( 1) ( 1)!
2!( 1)! ( ) 2!( )
m
m
m m
m m m
m


+ +
+ +
= =
+ +

After doing a bunch of integrations, we get:

( ) ( )
2 1
0
m n
P S t P S t dt

= <
}
( ) ( ) ( ) ( ) ( )
2 1
1 1 1
2 1 2 1
1 2 1 2 1 2 1 2
( 1) ( 1)...( 2) 1
1 ...
2! 1 !
n
m
m m m m n
m m m m m m n
n

+ + +
(
+ + +
= + + + + (
+ + + +
(

To simplify the above expression, we set

1
1 2
p


=
+
(probability of success),
2
1 2
1 q p


= =
+
(probability of failure)

( ) ( )
2 1
0
m n
P S t P S t dt

= <
}
0 0 1 1 1 2 2 1 1 1 1
1 1 2
1 ...
m m m n n m
m m m m n
q C p q C pq C p q C p q

+ +
( = + + + +

1 (having failures before sucesses) P m n =
If you have trouble understanding why

Page 238 of 285 Deeper Understanding: Exam M November 7, 2006 2006 Yufeng Guo
http:/ / www.guo.coursehost.com

0 0 1 1 1 2 2 1 1 1 1
1 1 2
...
m m m n n m
m m m m n
q C p q C pq C p q C p q

+ +
( + + + +

(having failures before sucesses) P m n =
well go through some concrete examples later to explain this.

Alternative method to calculate
( )
1 2
n m
P S S < .
( ) ( ) ( )
1 2 1 2 1 2
0 all possible
values
where 0
n m n m n m
t
t
P S S P S t S t dt P S t S t dt

< <
< = = > = = >
} }

( ) ( )
1 2
0
n m
P S t P S t dt

= = >
}
(
1 2
,
n m
S S are independent)

( )
( )
1
1
1 1
1
( 1)!
n
t
n
t
P S t e
n

= =

( ) ( )
2
The total arrival time of events is greatet than
m
P S t P m t > =
[ ] ( )
# of events that have occurred during 0, 1 P t m = s
( ) ( )
( )
2
2 1
2 2
2
1 ...
2! 1 !
m
t
t t
e t
m

(
= + + + + (

(

( ) ( )
( ) ( ) ( )
( )
1 2
1 2 1
1 2 2 1 2
1 2
0 0
1 ...
( 1)! 2! 1 !
n m
t t
n m
t t t
P S t P S t dt e e t dt
n m






(
= > = + + + + (

(

} }

Using the formula
1
0
!
t n
n
n
e t dt
o
o

+
=
}
and doing integration item by item, we get:
( ) ( )
1 2 0 1 1 1 1 2 1 2 1 1 1
1 1 2
0
...
n n n m n m
n m n n n n m
P S t P S t dt p C p q C p q C p q C p q


+ +
( = > = + + + +
}
(having sucesses before failures) P n m =
Where
1
1 2
p


=
+
(probability of success),
2
1 2
1 q p


= =
+
(probability of failure)

Page 239 of 285 Deeper Understanding: Exam M November 7, 2006 2006 Yufeng Guo
http:/ / www.guo.coursehost.com

Formula you want to memorize:

( )
1 2
n m
P S S <
(having sucesses before failures) P n m =
1 (having failures before sucesses) P m n =
Where
1
1 2
p


=
+
(success),
2
1 2
q


=
+
(failure)
Try to redo Problem 9 by yourself. This exercise forces you to understand the
relationship among a Poisson process, exponential distribution, and gamma distribution.
Problem 12

(# 6 Nov 2000) An insurance company has two insurance portfolios. Claims in Portfolio
P occur in accordance with a Poisson process with mean of 3 per year. Claims in
Portfolio Q occur in accordance with a Poisson process with mean 5 per year. The two
processes are independent.

Calculate the probability that 3 claims occur in Portfolio P before 3 claims occur in
Portfolio Q.
(A) 0.28 (B) 0.33 (C) 0.38 (D) 0.43 (E) 0.48

Solution

The required probability is
( )
3 3
P Q
P S S < .
3 3
3 5 8
P
P Q
p


= = =
+ +
(success),
5
1
8
q p = = (failure)

( )
3 3
(having 3 successes before 3 failures)
P Q
P S S P < =
Let S=success, F=failure

Page 240 of 285 Deeper Understanding: Exam M November 7, 2006 2006 Yufeng Guo
http:/ / www.guo.coursehost.com

1
st
method
Scenarios # of success # of failures Outcomes Probability Result
1 3 0 SSS
3
3
3
8
p
| |
=
|
\ . 0.05273
2 3 1 FSSS
3
3
5 3
8 8
p q
| |
=
|
\ . 0.03296
3 3 1 SFSS
3
3
5 3
8 8
p q
| |
=
|
\ . 0.03296
4 3 1 SSFS
3
3
5 3
8 8
p q
| |
=
|
\ . 0.03296
5 3 2 SSFFS
2 3
3 2
5 3
8 8
p q
| | | |
=
| |
\ . \ . 0.02060
6 3 2 SFFSS
2 3
3 2
5 3
8 8
p q
| | | |
=
| |
\ . \ . 0.02060
7 3 2 FFSSS
2 3
3 2
5 3
8 8
p q
| | | |
=
| |
\ . \ . 0.02060
8 3 2 FSSFS
2 3
3 2
5 3
8 8
p q
| | | |
=
| |
\ . \ . 0.02060
9 3 2 FSFSS
2 3
3 2
5 3
8 8
p q
| | | |
=
| |
\ . \ . 0.02060
10 3 2 FSSFS
2 3
3 2
5 3
8 8
p q
| | | |
=
| |
\ . \ . 0.02060
Total3
successes
before 3
failures 0.27521
Lets look at the above table carefully to see why it produces the probability of having 3
successes before 2 failures. Look at Scenario 1. If in the first 3 trials you get 3
consecutive successes (SSS), then no matter what outcomes you get in the 4
th
and 5
th
trials, you are guaranteed to have at least 3 successes before you ever get 3 failures. For
example, if the 4
th
and 5
th
trials give you 2 additional successes, youll have 5 successes
in a row in the first 5 trials. Having 5 successes in a row is still having 3 successes
before having 3 failures.

Look at Scenarios 5 through 10. In each of these scenarios, you get 3 successes and 2
failures in the first 5 trials. No matter what happens in the 6
th
trial, you are again
guaranteed to have at least 3 successes before ever having 3 failures. If the 6
th
trial is a
success, then you get 4 successes and 2 failures in 6 trials and this meets the requirement
Page 241 of 285 Deeper Understanding: Exam M November 7, 2006 2006 Yufeng Guo
http:/ / www.guo.coursehost.com

of having 3 trials before 3 failures. If the 6
th
trial is a failure, then you have exactly 3
successes before 3 failures.

2
nd
solution.
( )
3 3
1 (having 3 failures before 3 sucesses)
P Q
P S S P < =
Scenarios # of failures # of success Outcome Probability Result
1 3 0 FFF
3
3
5
8
q
| |
=
|
\ . 0.24414
2 3 1 SFFF
3
3
3 5
8 8
pq
| |
=
|
\ . 0.09155
3 3 1 FSFF
3
3
3 5
8 8
pq
| |
=
|
\ . 0.09155
4 3 1 FFSF
3
3
3 5
8 8
pq
| |
=
|
\ . 0.09155
5 3 2 FFSSF
2 3
2 3
3 5
8 8
p q
| | | |
=
| |
\ . \ . 0.03433
6 3 2 FSSFF
2 3
2 3
3 5
8 8
p q
| | | |
=
| |
\ . \ . 0.03433
7 3 2 SSFFF
2 3
2 3
3 5
8 8
p q
| | | |
=
| |
\ . \ . 0.03433
8 3 2 SFFSF
2 3
2 3
3 5
8 8
p q
| | | |
=
| |
\ . \ . 0.03433
9 3 2 SFSFF
2 3
2 3
3 5
8 8
p q
| | | |
=
| |
\ . \ . 0.03433
10 3 2 SFFSF
2 3
2 3
3 5
8 8
p q
| | | |
=
| |
\ . \ . 0.03433
Total---3 failures
before 3
successes 0.72479
( )
3 3
1 (having 3 failures before 3 sucesses)
P Q
P S S P < = =1-0.72479=0.27521

3
rd
solution method in Ross Introduction to Probability Models:
To ensure that we have at least 3 successes before possibly getting 3 failures, we need to
have at least 3 successes in the first 5 trials. This way, no matter what happens to the 6
th

trial, we are guaranteed to have at minimum 3 trials before having 3 failures.

Page 242 of 285 Deeper Understanding: Exam M November 7, 2006 2006 Yufeng Guo
http:/ / www.guo.coursehost.com

Scenarios # of success # of failures Outcome Probability Result
1 5 0 SSSSS
5
p
0.00742
2 4 1 FSSSS
4
p q
0.01236
3 4 1 SFSSS
4
p q
0.01236
4 4 1 SSFSS
4
p q
0.01236
5 4 1 SSSFS
4
p q
0.01236
6 4 1 SSSSF
4
p q
0.01236
7 3 2 SSSFF
3 2
p q
0.02060
8 3 2 SSFFS
3 2
p q
0.02060
9 3 2 SFFSS
3 2
p q
0.02060
10 3 2 FFSSS
3 2
p q
0.02060
11 3 2 FSFSS
3 2
p q
0.02060
12 3 2 FSSFS
3 2
p q
0.02060
13 3 2 FSSSF
3 2
p q
0.02060
14 3 2 SFSFS
3 2
p q
0.02060
15 3 2 SFSSF
3 2
p q
0.02060
16 3 2 SSFSF
3 2
p q
0.02060
Total 0.27521
If you compare Method 1 with Method 3 (Ross method), youll notice that under
Method 3, the total probabilities for Scenarios 1,5, 6, and 7 is equal to the probability for
Scenario 1 under Method 1. The total probability for Scenario 4 and 13 under Method 3
is equal to the probability for Scenario 2 under Method 1.

Problem 13

Policyholders can reach the customer service department of an insurance company by
calling in or clicking in (sending emails). Assume the number of calls and the number of
email that arrive at the customer service department are two independent Poisson
processes with a rate of 20 and 5 per hour respectively.

Whats the probability that the customer service department receives 2 calls before
receiving 1 email?

Solution

The required probability is

( )
3 1
(having 3 successes before 1 failure)
Call Email
P S S P < =
Page 243 of 285 Deeper Understanding: Exam M November 7, 2006 2006 Yufeng Guo
http:/ / www.guo.coursehost.com

20
0.8
20 5
Call
Call Email
p


= = =
+ +
(success), 1 0.2 q p = = (failure)

( )
( )
3 1
3
3
(having at least 3 successes before 1 failure)
= = 0.8 0.512
Call Email
P S S P
p
< =
=
Alternatively,
( )
3 1
1 (having 1 failure before 3 successes)
Call Email
P S S P < =
Scenarios # of failures # of successes Outcome Probability Result
1 3 0 F
0.2 q =
0.200
2 3 1 SF
0.8(0.2) pq =
0.160
3 3 1 SSF
( )
2
2
0.8 0.2 p q =
0.128
Total 0.488
( )
3 1
1 (having 1 failure before 3 successes)=1-0.448=0.512
Call Email
P S S P < =
Page 244 of 285 Deeper Understanding: Exam M November 7, 2006 2006 Yufeng Guo
http:/ / www.guo.coursehost.com

Chapter 14 Claim payment per payment

2005 May #32
For an insurance:
Losses can be 100, 200 or 300 with respective probabilities 0.2, 0.2, and 0.6.
The insurance has an ordinary deductible of 150 per loss.
P
Y is the claim payment per payment random variable.
Calculate
( )
P
Var Y .
(A) 1500 (B) 1875 (C) 2250 (D) 2625 (E) 3000
Core concepts:
Ground up loss
Ordinary deductible
Claim payment
Claim payment per payment

Explanation

Let X represent the ground up loss amount (ground up loss amount is the actual loss
incurred by the policyholder). Let d where 0 d > represent the deductible.
Amount paid the insurer (called claim payment):
( ) ( )
0 if
max , 0
if
X d
X d X d
X d X d
+
s
= =

>

Amount the insured needs to pay out of his own pocket:


( ) ( )
if
min ,
if
X X d
X d X d
d X d
s
. = =

>

Please note that


( ) ( )
ground up loss
amount paid by the insured amount paid by the
out of his own pocket insurance company
X X d X d
+
= + .



Page 245 of 285 Deeper Understanding: Exam M November 7, 2006 2006 Yufeng Guo
http:/ / www.guo.coursehost.com

Example. Your deductible for your car insurance is $500. If you have an accident and the
loss is $600, you pay $500 out of your own pocket and your insurance company pays you
$100. In this case,
ground up loss amount paid by the amount paid by the insured
insurance company out of his own pocket
600 100 500 = +


However, if the loss is $400, then you pay all the loss and the insurance company pays
zero.

amount paid by the


ground up loss amount paid by the insured
insurance company
out of his own pocket
400 0 400 = +


Claim payment per payment
Let Y represent the claim payment. Then ( ) Y X d
+
= . Claim payment per payment
means
( )
0 Y Y > . Evidently, if X d s , then 0 Y s . In this case, the insured will cover all
the loss with his money and wont need to report the loss to the insurance company. So
the insurance company may not even know that a loss has incurred. So for the insurance
company to pay any claim, Y must be positive. This is why the claim payment per
payment is
( )
0 Y Y > .
Full solution

Let X represent the ground up loss. Let Y represent the claim payment. The deductible is
150 d = .
( ) ( ) 150 max 150, 0 Y X X
+
= =
0
P
Y Y Y = >
We are asked to find
( )
P
Var Y .
( ) ( ) ( ) 0 150 150
P
Var Y Var Y Y Var X X
+
(
= > = >

( ) ( )
2
2
150 150 150 150 E X X E X X
+
(
= > >


Please note that
Page 246 of 285 Deeper Understanding: Exam M November 7, 2006 2006 Yufeng Guo
http:/ / www.guo.coursehost.com

( ) ( ) ( )
2
2
150 150 150 150 150 150 Var X X E X X E X X > = > >
This is because
( )
2
150 150 E X X > is not an appropriate symbol.

X 100 200 300
( ) 150 X
+

0 50 150
( ) P X
0.2 0.2 0.6
( ) ( ) ( ) 150 200 300 0.8 P X P X P X > = = + = =
( )
( ) 150
P X
P X >
0.2
0.8
0.2
0.8
0.6
0.8
( )
0.2 0.2 0.6
150 150 0 50 150 125
0.8 0.8 0.8
E X X
| | | | | |
> = + + =
| | |
\ . \ . \ .

( )
2
2 2 2
0.2 0.2 0.6
150 150 0 50 150 17, 500
0.8 0.8 0.8
E X X
+
| | | | | |
(
> = + + =
| | |

\ . \ . \ .

( )
2
150 150 17, 500 125 1, 875 Var X X
+
(
> = =


Well use BA II Plus or BA II Plus Professional 1-V Statistics Worksheet to calculate
( ) 150 150 Var X X
+
(
>

.
As explained in the chapter on calculators, when using BA II Plus or BA II Plus
Professional 1-V Statistics Worksheet, we can simply discard the data that falls out of the
conditional probability and calculate the mean/variance on the remaining data.

X 100 200 300
Is 150 X > ? No, so discard
this data
Yes. Keep this
data.
Yes. Keep this
data.
After we discarded 100 X = , the remaining data is:
X 200 300
( ) 150 X
+

50 150
( ) P X
0.2 0.6
( ) 10P X -- Scaled up
probability
2 6
Page 247 of 285 Deeper Understanding: Exam M November 7, 2006 2006 Yufeng Guo
http:/ / www.guo.coursehost.com

Enter the following into Statistics Worksheet:

X01=200, Y01=2; X02=150, Y02=6

BA II Plus or BA II Plus Professional should give you:

8, 125, 43.30127019
X
n X o = = =
2
1, 875 Var o = =
Additional practice problems

#1 For an insurance policy:

Losses can be 100, 200, 300, and 400 with respective probabilities 0.1, 0.2, 0.3, and 0.4.

The insurance has an ordinary deductible of 250 per loss.

P
Y is the claim payment per payment random variable.

Calculate
( )
P
Var Y .
Solution

Fast solution

Ground up loss X 100 200 300 400
Is 250 X > ? No. Discard No. Discard. Yes. Keep. Yes. Keep.
New table after discarding 250 X s :
X 300 400
( ) 250 X
+

50 150
( ) P X
0.3 0.4
( ) 10P X -- scaled up probability
3 4
Enter the following into 1-V Statistics Worksheet:

X01=50, Y01=3; X02=150, Y02=4

BA II Plus or BA II Plus Professional should give you:

Page 248 of 285 Deeper Understanding: Exam M November 7, 2006 2006 Yufeng Guo
http:/ / www.guo.coursehost.com

7, 107.14, 49.48716593
X
n X o = = =
2
2, 4489.98 Var o = =
Standard solution

X 100 200 300 400
( ) 250 X
+

0 0 50 150
( ) P X
0.1 0.2 0.3 0.4
( ) ( ) ( ) 250 300 400 0.3 0.4 0.7 P X P X P X > = = + = = + =
( )
( ) 250
P X
P X >
0.1
0.7
0.2
0.7
0.3
0.7
0.4
0.7
( )
1 2 3 4
250 250 0 0 50 150 107.1428571
7 7 7 7
E X X
| | | | | | | |
> = + + + =
| | | |
\ . \ . \ . \ .

( )
2
2 2 2 2
1 2 3 4
150 150 0 0 50 150 13, 928.57143
7 7 7 7
E X X
+
| | | | | | | |
(
> = + + + =
| | | |

\ . \ . \ . \ .

( )
2
150 150 13, 928.57143 107.1428571 2, 448.99 Var X X
+
(
> = =


#2 For an insurance policy:

Losses can be 1,000, 4,000, 5,000, 9,000, and 12,000 with respective probabilities 0.11,
0.17, 0.24, 0.36, and 0.12.

The insurance has an ordinary deductible of 900 per loss.

P
Y is the claim payment per payment random variable.

Calculate
( )
P
Var Y .
Solution

Page 249 of 285 Deeper Understanding: Exam M November 7, 2006 2006 Yufeng Guo
http:/ / www.guo.coursehost.com

To speed up calculations, we set one unit of money equal to $1,000.

Ground up loss X 1 4 5 9 12
Is 0.9 X > ? Yes.
Keep.
Yes.
Keep.
Yes.
Keep.
Yes.
Keep.
Yes.
Keep.
( ) 0.9 X
+

0.1 3.1 4.1 8.1 11.1


( ) P X
0.11

0.17 0.24 0.36 0.12
( ) 100P X -- scaled
up probability
11 17 24 36 12
Enter the following into 1-V Statistics Worksheet:

X01=0.1, Y01=11; X02=3.1, Y02=17;
X03=4.1, Y03=24; X04=8.1, Y04=36;
X04=11.1, Y04=12

BA II Plus or BA II Plus Professional should give you:

100, 5.77, 3.28345854
X
n X o = = =
( )
2
2 2
10.781 10.781 $1, 000 10, 781,100$ Var o = = = =
Page 250 of 285 Deeper Understanding: Exam M November 7, 2006 2006 Yufeng Guo
http:/ / www.guo.coursehost.com

Chapter 15 Aggregate model, normal approximation

SOA problems M Sample # 37, # 71

Explanation

The model is
1
N
i
i
Y X
=
=

.
Assumptions:
1 2
, ,...,
N
X X X are independent and identically distributed with a common pdf ( )
X
f x ;
N (a non-negative integer).
You are asked to find ( ) P Y y s using normal approximation.
Please note this model is different from Z n X = .
In the model
1
N
i
i
Y X
=
=

,
1 2
, ,...,
N
X X X may take different values. Some values are big
and others are small. Big values and small cancel each other out and
1
N
i
i
Y X
=
=

is
approximately normal.
In contrast, if Y n X = , then the value of X is magnified n times. As a result, Y is not
normally distributed.
So for
1
N
i
i
Y X
=
=

, dont use the following variance formula:
( ) ( ) ( )
2
Var Y Var nX n Var X = =
The above formula would have been correct if your model is Y n X =
To use normal approximation for
1
N
i
i
Y X
=
=

, youll need to use the following formulas:

Page 251 of 285 Deeper Understanding: Exam M November 7, 2006 2006 Yufeng Guo
http:/ / www.guo.coursehost.com

( ) ( ) ( ) E Y E N E X =
( ) ( ) ( ) ( ) ( )
2
Var Y E N Var X Var N E X = +
( )
( )
( )
y E Y
P Y y
Y o
(
s = u
(

One potential mistake is to use the following wrong variance formula:

( ) ( ) ( ) ( ) ( ) Var Y E N Var X Var N E X = + (Wrong)

To remember to use ( )
2
E X in the right hand side of the variance formula, assume that
1 2
, ,...,
N
X X X are loss amounts in dollars. Then Y is the total dollar loss amount; ( ) Var Y
is dollar squared.

Then look at the right hand side of the formula. The 1
st
item ( ) ( ) E N Var X is dollar
squared. ( ) E N is a pure number. ( ) Var X is dollar squared. So ( ) ( ) E N Var X is dollar
squared.

The 2
nd
item needs to be dollar squared as well to make the equation work. As a result,
you need to use ( ) ( )
2
Var N E X , which is dollar squared. In contrast, ( ) ( ) Var N E X is
dollar, not dollar squared.

To avoid using ( ) ( ) Var N E X and other mistakes, in the heat of the exam, you might
want to draw the following diagram:

N X
E ( ) E N ( )
2
E X
Var Var(N) Var(X)
( ) ( ) ( ) ( ) ( )
2
Var Y E N Var X Var N E X = +
Sample M #37

The number of auto vandalism claims reported per month at Sunny Daze Insurance
Company (SDIC) has mean 110 and variance 750. Individual losses have mean 1101 and
standard deviation 70. The number of claims and the amounts of individual losses are
Page 252 of 285 Deeper Understanding: Exam M November 7, 2006 2006 Yufeng Guo
http:/ / www.guo.coursehost.com

independent. Using the normal approximation, calculate the probability that SDICs
aggregate auto vandalism losses reported for a month will be less than 100,000.

(A) 0.24 (B) 0.31 (C) 0.36 (D) 0.39 (E) 0.49

Solution

Set
X = individual loss amount in dollars
N = # of losses reported per month
Y = total dollar loss amount per month

N
i
i i
Y X
=
=

N X
E 110
2
1,101
Var 750
2
70
( ) ( ) ( )
2 2
100 70 750 1,101 909, 689, 750 Var Y = + =
( ) ( ) ( ) ( ) 110 1,101 121,110 E Y E N E X = = =
( )
( )
( )
100, 000
100, 000 121,110
100, 000
909, 689, 750
E Y
P Y
Y o
( (

s = u = u
( (


( ) ( ) 0.7 1 0.7 24.2% u = u =
Please note that

( ) ( ) 1 z z u + u = holds for any z . This equation comes in handy in the exam when you
have a negative z and you cant look up ( ) z u from the normal distribution table.

Page 253 of 285 Deeper Understanding: Exam M November 7, 2006 2006 Yufeng Guo
http:/ / www.guo.coursehost.com

Sample M #71

You own a fancy light bulb factory. Your workforce is a bit clumsy they keep dropping
boxes of light bulbs. The boxes have varying numbers of light bulbs in them, and when
dropped, the entire box is destroyed.

You are given:

Expected number of boxes dropped per month: 50
Variance of the number of boxes dropped per month: 100
Expected value per box: 200
Variance of the value per box: 400

You pay your employees a bonus if the value of light bulbs destroyed in a month is less
than 8,000.

Assuming independence and using the normal approximation, calculate the probability
that you will pay your employees a bonus next month.

(A) 0.16 (B) 0.19 (C) 0.23 (D) 0.27 (E) 0.31

Solution

Set
X = individual dollar value per box
N = # of boxes dropped per month
Y = total dollar loss per month

1
N
i
i
Y X
=
=

N X
E 50
2
200
Var 100 400
( ) ( ) ( )
2
50 400 100 200 4, 020, 000 Var Y = + =
( ) ( ) ( ) ( ) 50 200 10, 000 E Y E N E X = = =
( )
( )
( )
( )
8, 000
8, 000 10, 000
8, 000 0.998 16%
4, 020, 000
E Y
P Y
Y o
( (

s = u = u = u =
( (


Page 254 of 285 Deeper Understanding: Exam M November 7, 2006 2006 Yufeng Guo
http:/ / www.guo.coursehost.com

Chapter 16 Double aggregate model, normal
approximation

SOA Sample #4

The model is:

1
N
i
i
Y X
=
=

(here N is a random variable taking on non-negative integer values)
1
m
j
j
S Y
=
=

(here m is a constant taking on non-negative integer values)



Then

( ) ( ) ( ) E Y E N E X =
( ) ( ) ( ) ( ) ( )
2
Var Y E N Var X Var N E X = +
( ) ( ) E S mE Y =
( ) ( ) ( ) ( ) ( ) ( )
2
Var S mVar Y m E N Var X Var N E X ( = = +

( ) ( ) ( ) ( ) ( )
2
S m E N Var X Var N E X o ( = +

( )
( )
( )
s E S
P S s
S o
(
s = u
(

Special case --If N has a Poisson distribution with the parameter
( ) ( ) E N Var N = =
( ) ( ) ( ) ( ) E Y E N E X E X = = , ( ) ( ) ( )
2
Var Y Var X E X ( = +

Then it follows that

( ) ( ) ( ) E S mE Y m E X = =
( ) ( ) ( ) ( )
2
Var S mVar Y m Var X E X ( = = +

( ) ( ) ( )
2
S m Var X E X o ( = +

Page 255 of 285 Deeper Understanding: Exam M November 7, 2006 2006 Yufeng Guo
http:/ / www.guo.coursehost.com

( )
( ) ( )
( )
( )
( )
( )
1
k E S E S E S
P S k E S k
S S o o
( (
s = u = u (
( (


( )
( )
( )
( ) ( )
( )
( )
( )
( )
2 2
2 2
1 1
E S m E X m m
S
Var X X m Var X E X
E X E X

o
o
= = =
( +

+ +
( ) ( )
( )
( )
2
2
1
1
m
P S k E S k
X
E X

o
(
(
(
s = u (
(
(
+
(

Sample M #4

Computer maintenance costs for a department are modeled as follows:

(i) The distribution of the number of maintenance calls each machine will need in
a year is Poisson with mean 3.

(ii) The cost for a maintenance call has mean 80 and standard deviation 200.

(iii) The number of maintenance calls and the costs of the maintenance calls are all
mutually independent.

The department must buy a maintenance contract to cover repairs if there is at least a
10% probability that aggregate maintenance costs in a given year will exceed 120% of
the expected costs.

Using the normal approximation for the distribution of the aggregate maintenance costs,
calculate the minimum number of computers needed to avoid purchasing a maintenance
contract.

(A) 80 (B) 90 (C) 100 (D) 110 (E) 120

Solution

X = dollar cost of one maintenance call

N = # of maintenance call per machine

m= # of machines
Page 256 of 285 Deeper Understanding: Exam M November 7, 2006 2006 Yufeng Guo
http:/ / www.guo.coursehost.com

S = total maintenance cost for m machines

1
N
i
i
Y X
=
=

,
1
m
j
j
S Y
=
=

N is a Poisson distribution with mean 3 =


The department wont need to buy the contract if

( ) 1.2 10% P S E S > < (

or ( ) 1.2 90% P S E S s > (


( ) ( )
( )
( )
( )
2 2
2
3
1.2 1.2 1 1.2 1
200
1 1
80
m m
P S E S
X
E X

o
( (
( (
( (
s = u = u (
( (
| |
(
(
+ +
|
( (
\ .


( )
0.12865 m = u
( )
( ) 0.12865 90% 1.282 m u > = u
0.12865 1.282 m > , 99.3 m >
Additional problems:

#1 Computer maintenance costs for a department are modeled as follows:

The distribution of the number of maintenance calls each machine will need in
a year is Poisson with mean 5.

The cost for a maintenance call has mean 50 and standard deviation 300.

The number of maintenance calls and the costs of the maintenance calls are all
mutually independent.

The department must buy a maintenance contract to cover repairs if there is at least a
10% probability that aggregate maintenance costs in a given year will exceed 125% of
the expected costs.

Page 257 of 285 Deeper Understanding: Exam M November 7, 2006 2006 Yufeng Guo
http:/ / www.guo.coursehost.com

Using the normal approximation for the distribution of the aggregate maintenance costs,
calculate the minimum number of computers needed to avoid purchasing a maintenance
contract.

Solution

The department wont need to buy the contract if

( ) 1.25 10% P S E S > < (

or ( ) 1.25 90% P S E S s > (


( ) ( )
( )
( )
2 2
2
0.25 5
1.25 1.25 1
300
1 1
50
m m
P S E S
X
E X

o
( (
( (
( (
s = u = u (
( (
| |
(
(
+ +
|
( (
\ .


( )
0.0919 m = u
( )
( ) 0.0919 90% 1.282 m u > = u
0.0919 1.282 m > , 194.59 m >
#2 Computer maintenance costs for a department are modeled as follows:

The distribution of the number of maintenance calls each machine will need in
a year has a mean 5 and standard deviation 10.

The cost for a maintenance call has mean 50 and variance 300.

The number of maintenance calls and the costs of the maintenance calls are all
mutually independent.

The department must buy a maintenance contract to cover repairs if there is at least a
10% probability that aggregate maintenance costs in a given year will exceed 125% of
the expected costs.

Using the normal approximation for the distribution of the aggregate maintenance costs,
calculate the minimum number of computers needed to avoid purchasing a maintenance
contract.

Solution

Page 258 of 285 Deeper Understanding: Exam M November 7, 2006 2006 Yufeng Guo
http:/ / www.guo.coursehost.com

The model is:

1
N
i
i
Y X
=
=

,
1
m
j
j
S Y
=
=

Then ( ) ( ) ( ) ( ) 5 50 250 E Y E N E X = = =
N X
E 5
2
50
Var
2
10 300
( ) ( ) ( )
2 2
5 300 10 50 251, 500 Var Y = + =
( ) ( ) 250 E S mE Y m = =
( ) ( ) 251, 500 Var S mVar Y m = =
The department wont need to buy the contract if

( ) 1.25 10% P S E S > < (

or ( ) 1.25 90% P S E S s > (


( )
( ) ( )
( )
( )
( )
1.25 0.25
1.25
E S E S E S
P S E S
S S o o
( (
s = u = u (
( (


( ) 0.25 250
251, 500
m
m
(
= u
(

( )
( )
0.25 250
90% 1.282
251, 500
m
m
(
u > = u
(


( ) ( ) 0.25 250 0.25 250
1.282, 1.282,
251, 500 251, 500
m
m
m
> > 105.82 m >
Page 259 of 285 Deeper Understanding: Exam M November 7, 2006 2006 Yufeng Guo
http:/ / www.guo.coursehost.com

Chapter 17 Aggregate loss model

Sample M #14

Aggregate losses for a portfolio of policies are modeled as follows:
The number of losses before any coverage modifications follows a Poisson distribution
with mean .
The severity of each loss before any coverage modifications is uniformly distributed
between 0 and b .
The insurer would like to model the impact of imposing an ordinary deductible, d
( ) 0 d b < < , on each loss and reimburse only a percentage, ( ) 0 1 c c < , of each loss in
excess of the deductible.
It is assumed that the coverage modifications will not affect the loss distribution. The
insurer models its claims with modified frequency and severity distributions. The
modified claim amount is uniformly distributed on the interval ( ) 0, c b d

.
Determine the mean of the modified frequency distribution.
Core concepts:
Loss frequency
Loss severity
Aggregate loss
Poisson distribution is scalable

Assume that a policyholder has incurred N ( N is a non-negative integer) losses in a
year (or two years, quarter, month, whatever time unit). N is a random variable.
Also assume that
the dollar amount of the 1
st
loss is
1
X ;
the dollar amount of the 2
nd
loss is
2
X ;

the dollar amount of the N th is


N
X .
Then the total dollar loss amount incurred by the policyholder in a year is:
1 2 2
1
...
N
N i
i
S X X X X X
=
= + + + + =

Page 260 of 285 Deeper Understanding: Exam M November 7, 2006 2006 Yufeng Guo
http:/ / www.guo.coursehost.com

1
X ,
2
X , ,
N
X are called loss severities.
1
X ,
2
X , ,
N
X can be discrete or
continuous; they are non-negative. Most likely, we assume that
1
X ,
2
X , , and
N
X are
independent and identically distributed with a common pdf and cdf.
N is called loss frequency; N represents the total # of losses incurred in a year. N
follows a discrete distribution (such as Poisson distribution or binomial distribution).
S is the aggregate (i.e. total) loss amount in dollars. S is a random sum of N individual
losses.
The equation
1 2 2
1
...
N
N i
i
S X X X X X
=
= + + + + =

is called the aggregate loss model.


Its often assumed that N and
i
X are independent. However, some times a question will
tell you that N and
i
X are dependent.
Key formulas for the aggregate loss model:

Assume (1) N and
i
X are independent, (2)
1
X ,
2
X , , and
N
X are independent and
identically distributed. Well use X to represent the common loss amount random
variable with pdf ( )
X
f x and cdf ( )
X
F x . Then
1
X ,
2
X , , and
N
X have the same pdf
( )
X
f x and cdf ( )
X
F x .
( ) ( ) ( )
1 2 2
...
N
E S X X X X E N E X = + + + + =
( ) ( ) ( ) ( ) ( )
2
1 2 2
...
N
Var S X X X X E N Var X Var N E X = + + + + = +
Formulas for the aggregate loss if N has Poisson distribution with mean :
If N has Poisson distribution with mean , then ( ) ( ) E N Var N = = .
( ) ( )
1 2 2
...
N
E S X X X X E X = + + + + =
( ) ( ) ( ) ( ) ( )
2
1 2 2
...
N
Var S X X X X E N Var X Var N E X = + + + + = +
( ) ( )
2
Var X E X = +
( ) ( ) ( )
2 2
Var X E X E X
= + =

See SOA Solution to Sample M #14
Page 261 of 285 Deeper Understanding: Exam M November 7, 2006 2006 Yufeng Guo
http:/ / www.guo.coursehost.com

Chapter 18 LER (loss elimination ratio)

MSample #27

You are given:
Losses follow an exponential distribution with the same mean in all years.
The loss elimination ratio this year is 70%.
The ordinary deductible for the coming year is 4/3 of the current deductible.
Compute the loss elimination ratio for the coming year.
Core concept:

Loss elimination ratio (LER)

( )
( )
Expected loss amount paid by the insured
Expected loss amount
E X d
LER
E X
.
= =
LER answers the question, What % of the expected loss amount is absorbed by the
policyholder due to the deductible?
How to calculate LER.

( ) ( ) ( )
0 0
E X xf x dx s x dx
+ +
= =
} }

( ) ( )
if
min ,
if
X X d
X d X d
d X d
s
. = =

>

( ) ( ) ( )
0
d
d
E X d x f x dx d f x dx
+
. = +
} }
(Intuitive formula)
Alternatively,
( ) ( ) ( ) 1
X
d d
E X d s x dx F x dx
+ +
. = = (
} }

You can find the proof of the 2
nd
formula from Loss Models.
Page 262 of 285 Deeper Understanding: Exam M November 7, 2006 2006 Yufeng Guo
http:/ / www.guo.coursehost.com

To help memorize the above formulas, notice that if we set 0 d = , then

( ) ( ) ( )
0
E X E X s x dx
+
= .0 =
}
Solution to Sample #27

Ground up loss X has exponential distribution with mean u :
( )
1
x
f x e
u
u

= , ( ) ( ) 1 1 1
x x
s x F x e e
u u
| |
= = =
|
\ .
, ( ) E X u =
( ) ( ) 1
x d
d d
E X d s x dx e dx e
u u
u
+ +
| |
. = = =
|
\ .
} }

( )
( )
1
d
E X d
LER e
E X
u
.
= = (you might want to memorize this result)

Under the original deductible, 70% LER =
1 0.7, 0.3
d d
e e
u u

= =
Under the new deductible (which is
4
3
of the original deductible),

4
4
4 3
' 3
3
1 1 1 0.3 0.799
d d
LER e e
u u
| | | |

| |
\ . \ .
(
= = = =
(
(


Page 263 of 285 Deeper Understanding: Exam M November 7, 2006 2006 Yufeng Guo
http:/ / www.guo.coursehost.com

Chapter 19 Find E(Y-M)
+
( ) ( ) ( ) ( ) ( ) ( ) ( ) ( ) 0 1 1 2 2 ...1 1
Y Y Y Y
E Y m E Y m mf m f m f f m
+
= + + + +
Where Y and m are non-negative integers.

The above formula works whether Y is a simple random variable or a compound random
variable
1
n
i
i
Y X
=
=

. If
1
n
i
i
Y X
=
=

, make sure you write


( ) ( ) ( ) ( ) ( ) ( ) ( ) ( ) 0 1 1 2 2 ...1 1
Y Y Y Y
E Y m E Y m mf m f m f f m
+
= + + + +
Dont write
( ) ( ) ( ) ( ) ( ) ( ) ( ) ( ) 0 1 1 2 2 ...1 1
X X X X
E Y m E Y m mf m f m f f m
+
= + + + +
In other words, the pdf in the right hand side must match up with the random variable in
the left hand side. If the random variable in the left hand side
1
n
i
i
Y X
=
=

, you need to use


( )
Y
f y in the right hand and write the following equation:
( ) ( ) ( ) ( ) ( ) ( ) ( ) ( ) 0 1 1 2 2 ...1 1
Y Y Y Y
E Y m E Y m mf m f m f f m
+
= + + + +
If your random variable in the left hand side is X , then you need to write
( ) ( ) ( ) ( ) ( ) ( ) ( ) ( ) 0 1 1 2 2 ...1 1
X X X X
E X m E X m mf m f m f f m
+
= + + + +
To use the above formula in the heat of the exam, we rewrite the above formula into:

( ) ( )
( )
( )
( )
( )
0
1 1
2 2
... ...
1 1
Y
Y
Y
Y
f m
f m
E Y m E Y m f m
f m
+
(
(
(
(

(
(
(
( = +
(
(
(
(
(
(




In the above formula,

Page 264 of 285 Deeper Understanding: Exam M November 7, 2006 2006 Yufeng Guo
http:/ / www.guo.coursehost.com

( )
( )
( )
( )
( ) ( ) ( ) ( ) ( ) ( )
0
1 1
2 0 1 1 2 2 ...1 1 2
... ...
1 1
Y
Y
Y Y Y Y Y
Y
f m
f m
f mf m f m f f m m
f m
(
(
(
(

(
(
(
( = + + +
(
(
(
(
(
(




This is not a standard notation. However, we use it anyway to help us memorize the
formula. In the exam, you just write these 2 matrixes. Then you simply take out each
element in the 1
st
matrix and multiply it with a corresponding element in the 2
nd
matrix.
Next, sum everything up.

Please note that if you take out an element ( )
Y
f k (where 0 1 k m s s ) from the 1
st

matrix, then you need to multiple it with m k from the 2
nd
matrix so ( ) m k k m + =
stands.
The proof of this formula is simple.

The standard formula is:

( ) ( ) ( )
1
0
1
d
S
s
E S d E S F s

+
=
= (

Please note that I didnt write the formula as



( ) ( ) ( )
1
0
1
d
S
s
x E S d E S F

+
=
= (

The above formula is confusing because ( )


S
f x is not a good notation because S and x
dont match. The right notation should be ( )
S
f s .
Lets move on from the formula ( ) ( ) ( )
1
0
1
d
S
s
E S d E S F s

+
=
= (

. To make our proof


simple, lets set 3 d = . The proof is the same if d is bigger.

( ) ( ) ( )
2
0
3 1
S
s
E S E S F s
+
=
= (

( ) ( ) ( ) ( ) ( ) ( ) ( )
2
0
1 1 0 1 1 1 2 3 0 1 2
S S S S S S S
s
F s F F F F F F
=
= + + = + + ( ( ( ( (

( ) ( ) ( ) ( ) 0 0 0 0
S S
F P S P S f = s = = =
Page 265 of 285 Deeper Understanding: Exam M November 7, 2006 2006 Yufeng Guo
http:/ / www.guo.coursehost.com

( ) ( ) ( ) ( ) ( ) ( ) 1 1 0 1 0 1
S S S
F P S P S P S f f = s = = + = = +
( ) ( ) ( ) ( ) ( ) ( ) ( ) ( ) 2 2 0 1 2 0 1 2
S S S S
F P S P S P S P S f f f = s = = + = + = = + +
( ) ( ) ( ) ( ) ( ) ( ) 0 1 2 3 0 2 1 2
S S S S S S
F F F f f f + + = + + (


( ) ( ) ( ) ( ) ( ) 3 3 3 0 2 1 2
S S S
E S E S f f f
+
= + + + (

Now you should be convinced that the following formula is correct:

( ) ( )
( )
( )
( )
( )
0
1 1
2 2
... ...
1 1
Y
Y
Y
Y
f m
f m
E Y m E Y m f m
f m
+
(
(
(
(

(
(
(
( = +
(
(
(
(
(
(




Problem 1 # 11 May 2000 Course 3

A company provides insurance to a concert hall for losses due to power failure. You are
given:

The number of power failures in a year has a Poisson distribution with mean 1.

The distribution of ground up losses due to a single power failure is

The number of power failures and the amounts of losses are independent.

There is an annual deductible of 30.

Calculate the expected amount of claims paid by the insurer in one year.

Solution

Let N = # of power failures, S = total claim dollar amount before deductible.

x Probability of x
10 0.3
20 0.3
50 0.4
Page 266 of 285 Deeper Understanding: Exam M November 7, 2006 2006 Yufeng Guo
http:/ / www.guo.coursehost.com

Then
1
N
i
i
S X
=
=

.
The total claim dollar amount after the deductible of $30 is:

( )
1
30 30
N
i
i
S X
+
=
+
(
=
`
(
)

Applying the formula, we have:



( ) ( )
( )
( )
( )
( )
0 30
1 29
30 30 2 28
... ...
29 1
S
S
S
S
f
f
E S E S f
f
+
(
(
(
(
(
(
(
( = +
(
(
(
(
(
(



It seems like we have awful lot of work to do about the two matrixes. Before you start to
panic, please note that many of the values ( ) ( ) ( ) 0 , 1 ,..., 29
S S S
f f f will be zero. This is
because X has only 3 distinct values: 10, 20, and 50 with probability of 0.3, 0.3, and 0.4
respectively. Evidently, we can throw away 50 X = . If 50 X = , then S is at least 50 and
is out of the range 29 S s .
Please also note that
1
N
i
i
S X
=
=

where N is a Poisson random variable with mean 1 = .


( )
1
1
!
P N n e
n

= =
So for 29 S s , the possible values of S are:
N
( ) P N
X
( )
1 2
, ,...,
N
P X X X
1
N
i
i
S X
=
=

( ) P S
0
1
e

0
1
e

10 X = 0.3 10
1
0.3e

1
1
e

20 X = 0.3 20
1
0.3e

2
1
1
2
e
( ) ( )
1 2
, 10,10 X X =
2
0.3
20
( )
1 2
1
0.3
2
e

Page 267 of 285 Deeper Understanding: Exam M November 7, 2006 2006 Yufeng Guo
http:/ / www.guo.coursehost.com

Next, we consolidate the probabilities:

1
N
i
i
S X
=
=

( ) P S
0
1
e

10
1
0.3e

20
1
0.3e

20
( )
1 2
1
0.3
2
e

After consolidation:

1
N
i
i
S X
=
=

( ) P S
0
1
e

10
1
0.3e

20
( )
1 1 2 1
1
0.3 0.3 0.345
2
e e e

+ =
( ) ( )
( )
( )
( )
0 30
30 30 10 20
20 10
S
S
S
f
E S E S f
f
+
( (
( (
= +
( (
( (


In the actual exam, to help remember the two matrixes, you can write only the 1
st
matrix:

( )
( )
( )
0
10
20
S
S
S
f a
f b
f c
( (
( (

( (
( (


As said early, the sum of the two elements in each row needs to be m (or 30 in this
problem). As a result,

0 30 30 a a + = =
10 30 20 b b + = =
20 30 10 c c + = =
Then, you can fill out the 2
nd
matrix:

Page 268 of 285 Deeper Understanding: Exam M November 7, 2006 2006 Yufeng Guo
http:/ / www.guo.coursehost.com

( )
( )
( )
( )
( )
( )
0 0 30
10 10 20
20 20 10
S S
S S
S S
f a f
f b f
f c f
( ( ( (
( ( ( (
=
( ( ( (
( ( ( (


( )
( )
( )
1
1 1 1
1
0 30 30 1 30
10 20 0.3 20 0.3 20 39.45
20 10 0.345 10 0.345 10
S
S
S
f e
f e e e
f e

( ( ( ( ( (
( ( ( ( ( (
= = =
( ( ( ( ( (
( ( ( ( ( (


( ) ( ) ( )
1
N
i
i
S X E S E N E X
=
= =

( ) 1 E N = , ( ) ( ) ( ) ( ) 10 0.3 20 0.3 50 0.4 29 E X = + + =


( ) ( ) ( ) 29 E S E N E X = =
( ) ( )
1
30 30 39.45 13.5128 E S E S e

+
= + =
Problem 2 #18 May M, 2005

For a collective risk model:

The number of losses has a Poisson distribution with 2 =
The common distribution of the individual losses is:

x
( )
X
f x
1 0.6
2 0.4
An insurance covers aggregate losses subject to a deductible of 3.

Calculate the expected aggregate payments of the insurance.

Solution

Page 269 of 285 Deeper Understanding: Exam M November 7, 2006 2006 Yufeng Guo
http:/ / www.guo.coursehost.com

1
N
i
i
S X
=
=

where S is the aggregate loss and X is individual loss dollar amount.



We are asked to find ( ) 3 E S
+
.
( ) ( )
( )
( )
( )
0 3
3 3 1 2
2 1
S
S
S
f
E S E S f
f
+
( (
( (
= +
( (
( (


Where ( ) ( ) ( ) ( ) ( ) 2 1 0.6 2 0.4 2.8 E S E N E X = = + = (


Next, we need to find
( )
( )
( )
0
1
2
S
S
S
f
f
f
(
(
(
(

.
N
( ) P N
X
( )
1 2
, ,...,
N
P X X X
1
N
i
i
S X
=
=

( ) P S
0
2
e

0
2
e

1 X = 0.6 1
( )
2
0.6 2e

1
2
2e

2 X = 0.4 2
( )
2
0.4 2e

2
2
2 2
2
2
2!
e e

=
( ) ( )
1 2
, 1,1 X X =
2
0.6
2
( )
2 2
0.6 2e

Next, we consolidate the table into:



1
N
i
i
S X
=
=

( ) P S
0
2
e

1
( )
2 2
0.6 2 1.2 e e

=
2
( ) ( )
2 2 2 2
0.4 2 0.6 2 1.52 e e e

+ =
( ) ( )
( )
( )
( )
2
2
2
0 3 3
3 3 1 2 2.8 3 1.2 2
2 1 1.52 1
S
S
S
f e
E S E S f e
f e

( ( ( (
( ( ( (
= + = +
( ( ( (
( ( ( (


Page 270 of 285 Deeper Understanding: Exam M November 7, 2006 2006 Yufeng Guo
http:/ / www.guo.coursehost.com

2 2
1 3
2.8 3 1.2 2 2.8 3 6.92 0.73652
1.52 1
e e

( (
( (
= + = + =
( (
( (


Problem 3 Sample M #45

Prescription drug losses, S, are modeled assuming the number of claims has a geometric
distribution with mean 4, and the amount of each prescription is 40.

Calculate ( ) 100 E S
+

Ill leave this problem for you to solve.



Page 271 of 285 Deeper Understanding: Exam M November 7, 2006 2006 Yufeng Guo
http:/ / www.guo.coursehost.com

Chapter 20 Other commonly used shortcuts

Convert discount rate to interest rate or vice versa

Often in the exam, you need to convert d to i or i to d .
To find d (interest paid in the beginning of the year), discount i (interest paid at the end
of the year) back one period:

1
i
d i DiscountFactor i v
i
= = =
+
To find i from d , accumulate d over one period:

1
1
i d AccumulatingFactor d
d
| |
= =
|

\ .
Memorize the relationship between annuity due and life insurance

1
x x
A d a + =
Remember this story. You want to borrow $1 from a bank. You tell the bank owner, If
you lend me $1, Ill pay you interest in advance as long as I live. To make sure I can pay
back your principal of $1, I will purchase a whole life insurance of $1. Should I die, you
get the $1 death benefit. Anyway, if Im alive, Ill always pay you interest in advance as
long as I live. If I die, you get the $1 back from my insurance company.

Clearly, the PV of the whole life insurance and the PV of the interest payment annuity
should sum up to $1. So 1
x x
A d a + = .
The same logic apples to 1 x x A a o + = ,
: :
1
x n x n
A d a + = , and : : 1 x n x n A a o + = .
Page 272 of 285 Deeper Understanding: Exam M November 7, 2006 2006 Yufeng Guo
http:/ / www.guo.coursehost.com

De Moivres Law

1
:
1 1 1
n
x n n i
v
A a
x x i e e
| |
= =
|

\ .
1
: :
1 1
1
n
n
n x
x n x n
v n
A A E v
x i x e e
| | | |
= + = +
| |

\ .
\ .
, where 1
n
n x
n
E v
x e
| |
=
|

\ .
1
:
1 1 1
n
x n n i
v
A a
x x e e o
| |
= =
|

\ .
If you need to calculate
: x n
a , its easier to calculate
1
: :
n x
x n x n
A A E = + and then

:
:
1
x n
x n
A
a
d

=
Similarly, to calculate : x n a , first calculate
1
: : x n x n
n x
A A E = + . Then

:
:
1 x n
x n
A
a
d

=
Recursive formulas life insurance and annuities

Treat
n x
E as the big discount factor. In Exam FM, if you need to discount a cash flow,
you use v . In Exam M, however, you need to use
n x
E in most cases.

( )
1 1 x x x x n x x
A vq DisctFactor A vq E A
+ +
= + = +
( )
1 1 1
: 1: 1 1: 1
x x n x
x n x n x n
A vq DisctFactor A vq E A
+ +
= + = +
( )
: 1: 1 1: 1
x x n x
x n x n x n
A vq DisctFactor A vq E A
+ +
= + = +
The above three formulas work too if you change A to A.
Page 273 of 285 Deeper Understanding: Exam M November 7, 2006 2006 Yufeng Guo
http:/ / www.guo.coursehost.com

If the death benefits are non-level:

( ) ( )
1 1
1
: 1: 1
x x
x n x n
IA A E IA
+
= +
( ) ( )
1 1
1
: 1: 1
x x
x n x n
DA nvq E DA
+
= +
1 1
1
x x x
a E a
+
= +
1
: 1: 1
1
x
x n x n
a E a
+
= +
1
1
1 x x
x
a E a + = +
: 1: 1
1
1 x n x n
x
a E a + = +
UDD

x
x
i
A A
o
=
1
1
:
:
x n
x n
i
A A
o
=
Please note that :
:
x n
x n
i
A A
o
=
1
1
: :
:
x n x n
n x n x
x n
i
A A E A E
o
= + = +
( )
( )
x
x
i
I A IA
o
=
( )
( )
1
1
:
:
x n
x n
i
I A IA
o
=
Other UDD shortcuts (Use May 2000 #1 as the example)

Page 274 of 285 Deeper Understanding: Exam M November 7, 2006 2006 Yufeng Guo
http:/ / www.guo.coursehost.com

t x
p
B(0,1)

O A ( x e , 0) ( ) T x
0
0
x
t x
e p dt

=
}
= Area of the graph
t x
p .
Shortcut #1
If
t x
p is a linear function of t , we can quickly calculate
0
0
x
t x
e p dt

=
}
by finding the
area below
t x
p .
In this problem, 1
x t
t x
x
l x t t
p
l x x
e
e e
+

= = =

. So
t x
p is a straight line AB in the 2-D
plane of [ ] ,
t x
t p . The area of the triangle AOB is
2
x e
.
0
0 0
2
x
x
t x t x
x
e p dt p dt
e
e

= = =
} }

Alternately,

( )
( )
( )
( )
2
2
0
0 0
0
1 1
1
2 2 2
x
x
x
t x
x t t x
e p dt dt t x
x x x
e
e
e e
e
e e e


( | |
= = = = =
( |

\ .

} }

Page 275 of 285 Deeper Understanding: Exam M November 7, 2006 2006 Yufeng Guo
http:/ / www.guo.coursehost.com

Shortcut #2
If
x
l x e = or 1
t x
t
p
x e
=

, then ( ) x follows De Moivres Law.


0
2
x
x
e
e
= , ( )
2
x
E T x
e
= (

, ( )
( )
2
12
x
Var T x
e
= (


To see why ( )
2
x
E T x
e
= (

and ( )
( )
2
12
x
Var T x
e
= (

, notice that

( )
( )
( )
0
T x
E T x f t t dt

= (
}
Under De Moivres Law

( )
( )
1
1
t x T x
d d t
f t p
dt dt x x e e
| |
= = =
|

\ .

( ) T x is uniformly distributed over [ ] 0, x e
To find ( ) E T x (

and ( ) Var T x (

without doing integration, you might want to
memorize the following formulas for uniform distributions:

If the random variable Y is uniformly distributed over [ ] , a b where b a > , then
=

1
( ) f y
b a
,
+
= ( )
2
b a
E Y , o

=
2 3
Y
b a
( )
( ) 0
2 2
x
x
E T x
e
e
+

= = (


( )
( )
2 2
0
12 2 3
x
x
Var T x
e
e

| |
= = (
|
\ .

Using all the shortcuts:

0
0
0
25, 50
2
e
e
e

= = =
Page 276 of 285 Deeper Understanding: Exam M November 7, 2006 2006 Yufeng Guo
http:/ / www.guo.coursehost.com

( )
( ) ( )
2 2
10 50 10
10 133.33
12 12
Var T
e
= = = (


Insurance and annuity when If and o are constant

If and o are constant, then

( )
'
1
:
0 0
n n
t t t
x n n A e e dt e dt a
o o
o o
+
= + = = =
} }

( )
'
:
0 0
n n
t t t
x n n a e e dt e dt a
o o
o o
+
= + = = =
} }

( )
'
1
n
n
e
a
o
o o
o
+
= +

=
+
(an annuity symbol in Exam FM)

If n :
'
x A a o o

o
= + = =
+
'
1
x a a o o
o
= + = =
+
Memorize first to die and last to die

Many candidates have trouble remembering the difference between x y and x y . Think
of the bar in x y as a roof of a house and remember the following:

If you have a roof over your head, you maximize your life and you are the second to die.
If you dont have a roof over your head, then you minimize your life and you are the first
to die.

x y is 1
st
to die. ( ) ( ) ( ) min , T x y T x T y = (

x y is 2nd to die.
( )
( ) ( ) max , T x y T x T y = (

Remember the following two sentences:

If the 1
st
-to-die is still alive, then both are alive. First alive, both alive.
x y x y
p p p =
Page 277 of 285 Deeper Understanding: Exam M November 7, 2006 2006 Yufeng Guo
http:/ / www.guo.coursehost.com

If the 2
nd
-to-die is dead, then both are dead. 2
nd
dead, both dead.
x y x y
q q q =
Calculate PV of insurance or annuity if mortality is updated

This type of problems occurs a lot in the exam. You need to know two things:
Write out the recursive relationship of annuity or death benefits

If the mortality is off in one year, we can issue a brand new insurance or annuity
policy next year. The brand new policy issue next year is NOT affected by the
mortality error in the previous year (because we are issuing a brand new policy to
a brand new policyholder).

Example. #29 May 2000 Course 3

In this problem,
1 x
p
+
is increased. Then we issue a brand new whole life annuity next
year to brand new policyholder age ( ) 2 x + . The PV of this brand new annuity at issue is
2 x
a
+
.
The recursive formula is:

2
1 2
1
x x x x x
a vp v p p a
+ +
= + +
If we decided to increase
1 x
p
+
and recalculate
x
a , we just need to put the new
1 x
p
+
value
in the above formula and recalculate
x
a . Everything else in the above formula, v ,
x
p ,
2
v , and
2 x
a
+
, stay intact as if we didnt increase
1 x
p
+
.
Common shock

You might want to draw a Venn Diagram. From the diagram above, you see that
z

( ) * T y
y

( ) * T y
y

Page 278 of 285 Deeper Understanding: Exam M November 7, 2006 2006 Yufeng Guo
http:/ / www.guo.coursehost.com

( ) ( ) * T y T y z
x x
= + ,
( ) ( ) * T y T y z
y y
= + ,
( ) ( ) * * T y T y z
x y x y
= + +
Poisson/gamma model

If N O is Poisson with mean u and Ohas a gamma distribution with parameters o and
u , then the unconditional distribution of N is negative binomial with parameter r o =
and | u = .
Formally, if given u O = , X is Poisson distributed with mean u , that is,
( )
!
x
X
f x e
x
u
u
u

O
= , 0,1, 2,... x =
And O is gamma distributed with pdf
( )
( )
1
e
f
o u |
o
u
u
o |

O
=
I
, 0 u >
Where 0 | > and 0 o > are parameters, then the margin probability function of X is:
( ) ( )
1
1 , 0,1, 2,...
x
X
x
f x p p x
x
o
o + | |
= =
|
\ .

See #2 May 2000 Course 3

Memorize Weibull Distribution

We-ibull = Wild Exponential

Exponential: ( )
x
s x e
u

=
Wild exponential: Raise
x
u
to t power. So ( )
x
s x e
t
u
| |

|
\ .
=
Page 279 of 285 Deeper Understanding: Exam M November 7, 2006 2006 Yufeng Guo
http:/ / www.guo.coursehost.com

In the exam, whenever you see Weibull distribution, you immediately write the survival
function of exponential distribution ( )
x
s x e
u

= . Next, you make the exponential


distribution go wild by raising
x
u
to t power.

Stop loss

( ) ( ) a b a b a
+
. + =
To see why this is true, please note that

( )
if
if
a a b
a b
b a b
s
. =

>

, ( )
0 if
if
a b
a b
a b a b
+
s
=

>

( ) ( ) a b a b a
+
. + =
If a X = (loss) and b d = (deductible), the above equation becomes

( ) ( )

total loss
policyhoder insurer pays
pays this part this part
X d X d X
+
. + =


#25 May 2000 Course 3 solution

( ) ( ) ( )
1 1 1
500, 000 0.7 500, 000 0.7 500, 000
3 500, 000 3 500, 000
L
bonus L
+
+
| | | |
= = (
| |
\ . \ .

( ) ( )
1 1
350, 000 350, 000 350, 000
3 3
L L
+
= = (

.
( ) ( )
1
350, 000 350, 000
3
E bonus E L = (

.
We go through this trouble because we can readily look up ( ) E X x . in the Tables for
Exam M. The rest of the work should be easy. You simply look up the ( ) E X x .
formula for Pareto distribution.

Page 280 of 285 Deeper Understanding: Exam M November 7, 2006 2006 Yufeng Guo
http:/ / www.guo.coursehost.com

Claim payment under policy limit, deductible, and inflation

You need to memorize Theorem 5.13 in Loss Models (Page 126):

( ) ( ) 1
1 1
L
u d
E Y r E X E X
r r
o
( | | | |
= +
| | (
+ +
\ . \ .
. .
If we set 1 o = (i.e. no co-insurance) and 0 r = , then

( ) ( ) ( )
L
E Y E X u E X d = . . .
This formula is very useful for the exam.

See #30 May 2000 Course 3.

Split a Poisson process
Example May 2000 #1

This is a split-Poisson type of problems.

If the total # of coins found is a Poisson distribution with parameter and

1
% x of the coins found are Type I

2
% x of the coins found are Type II

%
n
x of the coins found are Type N

Then we can split the original big Poisson distribution into the following N mutually
independent Poisson distributions:

# of Type I coins found is a Poisson distribution with parameter ( )
1
% x
# of Type II coins found is a Poisson distribution with parameter ( )
2
% x

# of Type N coins found is a Poisson distribution with parameter ( ) %


n
x
In this problem, we have three sub-Poisson distributions:

In one minute, the # of coins found worth 1 each has a Poisson distribution with
parameter (60%)0.5=0.3.
Page 281 of 285 Deeper Understanding: Exam M November 7, 2006 2006 Yufeng Guo
http:/ / www.guo.coursehost.com

In one minute, the # of coins found worth 5 each has a Poisson distribution with
parameter (20%)0.5=0.1.

In one minute, the # of coins found worth 10 each has a Poisson distribution with
parameter (20%)0.5=0.1.

Because we are concerned with the # of coins found in 60 minutes, we need to scale up
the parameters of the above three Poisson distributions:

In 60 minutes, the # of coins found worth 1 each (denoted as
1
Y ) has a Poisson
distribution with parameter
1
=0.3(60)=18 (1
st
Poisson distribution)

In one minute, the # of coins found worth 5 each (denoted as
2
Y ) has a Poisson
distribution with parameter
2
=0.1(60)=6 (2
nd
Poisson distribution)

In one minute, the # of coins found worth 10 each (denoted as
3
Y ) has a Poisson
distribution with parameter
3
=0.1(60)=6 (3
rd
Poisson distribution)

We are told that of the total coins found in 60 minutes, 10 were worth 5 each. As a result,
we need to update the parameter of the 2
nd
Poisson distribution to
'
2
10 = .
( )
1 1
18 E Y = =
( )
'
2 2
10 E Y = =
( )
3 3
6 E Y = =
The total value is: 18(1)+10(5)+6(10)=128

Page 282 of 285 Deeper Understanding: Exam M November 7, 2006 2006 Yufeng Guo
http:/ / www.guo.coursehost.com

About the author

Yufeng Guo was born in central China. After receiving his Bachelors degree in physics
at Zhengzhou University, he attended Beijing Law School and received his Masters of
law. He was an attorney and law school lecturer in China before immigrating to the
United States. He received his Masters of accounting at Indiana University. He has
pursued a life actuarial career and passed exams 1, 2, 3, 4, 5, 6, and 7 in rapid succession
after discovering a successful study strategy.

Mr. Guos exam records are as follows:
Fall 2002 Passed Course 1
Spring 2003 Passed Courses 2, 3
Fall 2003 Passed Course 4
Spring 2004 Passed Course 6
Fall 2004 Passed Course 5
Spring 2005 Passed Course 7
Mr. Guo currently teaches an online prep course for 2006 Exam P, FM, and M. For more
information, visit http://guo.coursehost.com.
If you have any comments or suggestions, you can contact Mr. Guo at
yufeng_guo@msn.com.
Please note that if I find any errors, I will post the errata at
http://guo.coursehost.com.
Page 283 of 285 Deeper Understanding: Exam M November 7, 2006 2006 Yufeng Guo
http:/ / www.guo.coursehost.com

User review of this manual

See users feedback at actuarialoutpost forum:
Pros & Cons of the M manual by Guo
What users said

Pros:
Offers a different approach to preparing for Exam M

Has good shortcuts and teaches you how to solve problems quickly and efficiently

The depth is great.

Cons
The author should have stated what COM stands for upfront. Why is the secrecy?
(OK; I told you what COM stands for in this 2
nd
edition and the 3
rd
edition

Has many typos (OK; I fixed the typos in this 3
rd
edition)

Page 284 of 285 Deeper Understanding: Exam M November 7, 2006 2006 Yufeng Guo
http:/ / www.guo.coursehost.com

Value of this PDF study manual

1. Dont pay the shipping fee (can cost $5 to $10 for U.S. shipping and over $30 for
international shipping). Big saving for Canadian candidates and other
international exam takers.

2. Dont wait a week for the manual to arrive. You download the study manual
instantly from the web and begin studying right away.

3. Load the PDF in your laptop. Study as you go. Or if you prefer a printed copy,
you can print the manual yourself.

4. Use the study manual as flash cards. Click on bookmarks to choose a chapter and
quiz yourself.

5. Search any topic by keywords. From the Adobe Acrobat reader toolbar, click Edit
->Search or Edit ->Find. Then type in a key word.

Page 285 of 285 Deeper Understanding: Exam M November 7, 2006 2006 Yufeng Guo

You might also like